Das kleine Einmaleins des Leib-Seele-Problems

Der Mensch als Kulturwesen oder Naturgegenstand?

Das Leib-Seele-Problem hat in unserer Kultur eine lange Geschichte. Viele werden hier zuerst an den Philosophen, Mathematiker und Physiologen René Descartes (1596-1650) denken. Dieser trennte Körper (beziehungsweise ausgedehnte Dinge) und Seelen (beziehungsweise denkende Dinge) begrifflich in zwei Domänen. Diese stünden im Menschen – und auch nur in ihm – vor allem in der Zirbeldrüse (Epiphyse) miteinander in Verbindung. Dieses Organ schien Descartes wegen seiner zentralen Lage im Gehirn der passende Ort für die Leib-Seele-Wechselwirkung.

Das Titelbild ist ein Gemälde des deutschen Malers und Bauhaus-Meisters Oskar Schlemmer (1888-1943) aus dem Jahr 1937. Es ist eine Hommage an den emeritierten Magdeburger Professor für Theoretische Philosophie Arno Ros, auf den mich bei einem früheren Artikel zum Leib-Seele-Problem ein Leser aufmerksam machte. Ros verwendete auf seinem Buch “Materie und Geist: Eine philosophische Untersuchung”, in dem er ähnliche Ziele verfolgt wie ich im folgenden Text, ein Gemälde desselben Künstlers. Schlemmer ist übrigens auch ein trauriges Beispiel dafür, zu welchen Verwerfungen der 70-jährige Urheberrechtsschutz führen kann. Denn seine Erben verhinderten mit vielen Klagen, dass seine Kunst ausgestellt werden konnte, bis der Schutz 2014 ein für alle Mal auslief. Bild: Sailko/CC BY-3.0

Ich möchte hier aber noch einmal ziemlich genau 2000 Jahre länger in unserer Kulturgeschichte zurückgehen, nämlich zu keinem Geringeren als Sokrates (469-399 v.Chr.). Dieser saß nach seiner Verurteilung wegen Gottesfrevels und verderblichen Einflusses auf die Jugend im Gefängnis und wartete auf seinen Tod. Mit seinen kritischen Fragen hatte er zu viele Athener gegen sich aufgebracht.

Seine Schüler und Freunde, darunter der Simmias aus wohlhabendem Hause, hatten ihm die Flucht ermöglichen wollen. Doch Sokrates blieb. Und er blieb mit Fassung, seinen Idealen treu, obwohl er den Urteilsspruch für ungerecht hielt. So will es die literarische Überlieferung seines Schülers Platon. Und dort im Gefängnis besuchten ihn noch einmal seine Schüler Phaidon, nach dem Sokrates’ letzte Unterweisungen benannt sind, Simmias und Kebes.

Der Tod des Sokrates, nach dem französischen Historienmaler Jacques-Louis David (1748-1825). Bild: The Met/public domain

Ein Grund für die Gefasstheit des Philosophen, während viele seiner Freunde schon um ihn trauerten, war dessen unerschütterlicher Glaube an die Unsterblichkeit der Seele. Auch heute, fast 2500 Jahre später, verwundert es uns nicht, dass Menschen sich im Angesicht des Todes Gedanken über das Jenseits machen. Hier soll es aber nicht um Sokrates’ Seelenlehre gehen.

Erklärung sozialer Sachverhalte

Für diejenigen unter uns, die noch für unbestimmte Zeit im Diesseits verharren, stellt sich eine andere interessante Frage: Warum war Sokrates im Gefängnis? Oder allgemeiner gefragt: Was gilt als Erklärung für Sachverhalte wie dem, dass der Philosoph dortblieb – und nicht etwa mit seinen Freunden geflohen war?

Sokrates erzählt, dass er sich schon in seiner Jugend für die Ursachen aller Phänomene interessierte. Dabei sei er auf das Werk des Naturphilosophen – Naturwissenschaft im heutigen Sinne gab es noch keine – Anaxagoras (500-428 v. Chr) gestoßen. Laut dessen Lehre sei die Antwort auf die Frage, warum Sokrates im Gefängnis ist,

“weil mein Leib aus Knochen und Sehnen besteht und die Knochen dicht sind und durch Gelenke voneinander geschieden, die Sehnen aber so eingerichtet, dass sie angezogen und nachgelassen werden können und die Knochen umgeben von dem Fleisch und der Haut, welche sie zusammenhält. Da sich nun die Knochen in ihren Gelenken drehen, so machten die Sehnen, wenn ich sie nachlasse und anziehe, dass ich jetzt imstande sei, meine Glieder zu bewegen, und aus diesem Grund säße ich jetzt hier mit gebogenen Knien.”

Phaidon, 98c-e, nach der Übersetzung von Friedrich Schleiermacher von 1809

Kurzum, aus heutiger Sicht würden wir sagen, dass Anaxagoras den Sachverhalt auf die Physiologie reduzieren wollte. Ähnliche sei der Versuch, erklärt Sokrates weiter, das Gespräch zwischen ihm und seinen Schülern auf die Töne, die Luft und das Gehör zu reduzieren. Solchen Bestrebungen widerspricht er aber entschieden und hält es stattdessen für die “wahren Ursachen”,

“dass nämlich, weil es den Athenern besser gefallen hat, mich zu verdammen, deshalb es auch mir besser geschienen hat, hier sitzen zu bleiben, und gerechter geschienen hat, hier zu bleiben und die Strafe geduldig auf mich zu nehmen, welche sie angeordnet haben. Denn, beim Hunde, schon lange, glaube ich wenigstens, wären diese Sehnen und Knochen in [der Hafenstadt] Megara oder bei den Boiotiern …, hätte ich es nicht für gerechter und schöner gehalten, lieber als dass ich fliehen und davongehen sollte, dem Staate die Strafe zu büßen, die er mir anordnet.”

Phaidon, 98e-99a, nach der Übersetzung von Friedrich Schleiermacher von 1809

Mit anderen Worten: Sokrates säße wegen des Urteils der Athener im Gefängnis und wegen seiner eigenen Entscheidung, weil er das für gerechter und schöner gehalten habe, als wegzulaufen. Und man muss auch fast 2500 Jahre später einräumen, dass dieser Erklärung eine gewisse Plausibilität innewohnt. Auf physiologischer Ebene aber erklären zu wollen, warum der Philosoph dort saß und wenig später aus dem Giftbecher trinken würde, anstatt in Megara seine Freiheit zu genießen, wäre auch heute noch eine unlösbare Herausforderung.

Willkommen inmitten des Leib-Seele-Problems.

Das formale Leib-Seele-Problem

Zwar reden wir inzwischen weniger von Seelen als zu Sokrates’ Zeiten – nicht in der Philosophie und schon gar nicht in den Naturwissenschaften. Doch, wie ich im Folgenden zeigen möchte, ist auch die Rede vom “Geist” (englisch mind, von lateinisch mens und Sanskrit manas), der uns in der Philosophie oder auch den “Geisteswissenschaften” häufiger begegnet, nicht unproblematisch. Bis hierhin sollte aber klar geworden sein, dass es bis heute eine Herausforderung geblieben ist, den Menschen als Kulturwesen oder als Naturgegenstand zu beschreiben; und diese Herausforderung begleitet uns schon mindestens seit der Antike.

Der britische Philosoph Jonathan Westphal hat 2016 ein neues Buch über das Leib-Seele-Problem – oder in seiner aktuelleren Variante sollte man es besser “Körper-Geist-Problem” nennen – geschrieben (Westphal, J. (2016). The Mind-Body Problem. Cambridge, MA: MIT Press). Darin formuliert er es als ein logisches Problem mit den folgenden vier Prämissen:

(1) Der Geist ist ein nichtphysikalisches Ding.
(2) Der Körper ist ein physikalisches Ding.
(3) Geist und Körper interagieren miteinander.
(4) Physikalische und nichtphysikalische Dinge können nicht miteinander interagieren.

Die vier Annahmen führen zu einem Widerspruch und können daher nicht gleichzeitig wahr sein. Auf den Sinn der einzelnen Prämissen und den Widerspruch werde ich bei einer ähnlichen Formulierung des Problems durch den Schweizer Philosophen Peter Bieri gleich ausführlicher eingehen. Hier will ich erst einmal die Aufmerksamkeit darauf richten, den Geist als “Ding” zu sehen. In der Fachsprache nennt man dies auch Reifikation, wortwörtlich “Verdinglichung”, von lateinisch res für “Ding”.

Ob wir nun im Deutschen vom “Geist” sprechen oder das englische “mind” verwenden, wie Westphal es in seinem Buch tut, sollten wir kurz innehalten, was wir damit überhaupt meinen. Der Duden erklärt das Wort als “denkendes Bewusstsein des Menschen, Verstandeskraft, Verstand”, das Grimmsche Wörterbuch setzt es in ganzen 119 Spalten unter anderem mit der Seele und dem Atem in Beziehung. Mit über 500.000 Zeichen wäre der Eintrag für ein einziges Wort genug für ein eigenes Buch! Das wäre aber allenfalls für ein paar Sprachwissenschaftler und Philosophen von Interesse.

Für unsere Zwecke ist die Feststellung hinreichend, dass man den Geist erst zu einem Ding macht und dann später Probleme bekommt, weil physikalische und nichtphysikalische Dinge nicht miteinander interagieren könnten. Man erwartet von dem so verstandenen Geist etwas, was er, gemäß wissenschaftlichen Überlegungen über beispielsweise Energieerhaltung oder Ursache-Wirkungs-Beziehungen, nicht leisten kann. Kein Wunder, dass das zu Schwierigkeiten führt!

Rätselhafte Wechselwirkung

Das Weltgeschehen ist komplexer als das, was sich auf einem Billardtisch abspielt. Dort trifft eine Kugel auf eine andere, überträgt ihre Bewegungsenergie und das Spiel geht weiter. Wenn man jetzt für die erste Kugel “den Geist” setzt, dann ist nicht ersichtlich, wo und wie die Interaktion, der Stoß, stattfinden soll. Mit diesem Problem war schon Descartes konfrontiert, wie in einem Briefwechsel mit Elisabeth von der Pfalz (1618-1680) dokumentiert ist. So schrieb die Adlige dem Philosophen im Mai 1643:

“Wie kann die Seele des Menschen die Lebensgeister [deutsch für lateinisch spiriti animali, wie man damals die Kraft nannte, die den Körper bewegt; Anm. St. S.] dazu veranlassen, die Willkürhandlungen auszuführen (da sie doch nur eine denkende Substanz ist)? Denn es scheint, daß jede Bewegung durch einen Stoß verursacht wird, wobei die Art des Stoßes von den Eigenschaften und der Form der Oberfläche abhängt, durch den der Stoß ausgeführt wird. In den ersten beiden Fällen wird Berührung vorausgesetzt und beim dritten räumliche Ausdehnung. Sie schließen aber diese vollständig aus dem Begriff aus, den Sie von der Seele haben, und jene erscheint mir unvereinbar mit einem immateriellen Gegenstand. Deshalb bitte ich Sie um eine spezifischere Definition der Seele als in ihrer Metaphysik.”

Zitiert nach Hammling, S. (2018). Mentale Verursachung. Problemaufriss und Darstellung einer Debatte innerhalb der zeitgenössischen Philosophie des Geistes. Ludwig-Maximilians-Universität München: Inauguraldissertation, S. 47.

Elisabeth machte Descartes darauf aufmerksam, dass er die Wechselwirkung von Leib und Seele nicht erklären konnte. Ja, der Philosoph könne nicht einmal plausibel machen, wie etwas Immaterielles die Teilchen im Körper anstoßen und so in Bewegung setzen könne! Auch Descartes’ Antwort stellte die Adlige nicht zufrieden und sie zeigte ihm weitere Probleme in seinem Denkmodell auf.

Wenn wir heute, bald 400 Jahre später, “Seele” durch “Geist” ersetzen, dann wird die Lösung keinesfalls naheliegender. Trotzdem wird es oft stillschweigend hingenommen, wenn jemand Sätze wie den folgenden formuliert, um hier noch einmal Westphal zu zitieren: “Materie oder das Physikalische kann irgendwie den Geist beeinflussen; und der Geist kann irgendwie den physikalischen Körper bewegen” (Westphal, 2016, a.a.O., S. 12; meine Übersetzung). Ein anderes Beispiel lieferte der kanadische Philosoph Walter Gannon erst kürzlich in einem Fachartikel über psychiatrische Behandlungen:

“Eine normale Geist-Gehirn-Interaktion ermöglicht es Personen, sich an die Welt anzupassen. Bei schwereren psychiatrischen Störungen gibt es sowohl auf der mentalen als auch auf der neuronalen Ebene eine Dysfunktion. Ein angemessenes Erklärungsmodell für diese Störungen und Interventionen zu ihrer Behandlung erfordert nicht nur ein Verständnis der Interaktion zwischen dem Geist und Gehirn, sondern auch wie genetische, epigenetische, hormonale, Immunsystem- und Umweltfaktoren diese Interaktion beeinflussen.”

Glannon, W. (2020). Mind-Brain Dualism in Psychiatry: Ethical Implications. Frontiers in Psychiatry, 11, 85, S. 2; meine Übersetzung.

Probleme dualistischer Sprache

Glannon meint, den Leib-Seele-Dualismus von Descartes überwunden zu haben, verwendet aber weiter dualistische Sprache, in der die Seele schlicht durch den Geist ersetzt wird. Westphal und Glannon sind damit nicht alleine; und es sind auch nicht nur Philosophen, die so reden. Eine schnelle Suche auf der Google-Seite für akademische Texte, Google Scholar, nach “the mind” ergibt beispielsweise fast vier Millionen Treffer. Und auf dem Web of Science, einer Datenbank für wissenschaftliche Fachartikel, liefert eine entsprechende Themensuche tausende Publikationen. Dabei stehen neben der Philosophie auch Psychiatrie, Psychologie, Neurowissenschaften, Geistes-, Literatur- und Geschichtswissenschaften ganz oben auf der Liste.

In der frühen Menschheitsgeschichte – und in manchen Religionen bis heute – war es üblich, unerklärlichen Naturkräfte als Götter zu beschreiben: Da gab es einen Wasser-, Feuer-, Regen- oder Donnergott und so weiter. Diesen Prozess nennt man auch “Deifikation”, Vergöttlichung. Machen wir dasselbe, wenn wir unsere psychischen Prozesse wie Wahrnehmungs-, Gefühls-, Denk-, Entscheidungs- und Handlungsprozesse einem Geist zuschreiben, Reifikation?

Wie sollten wir die Existenz so eines Dings nachweisen? Wir können es nicht anfassen, nicht hin- und herschieben oder mit physikalischen Teilchen beschießen. Um eine berühmte Wendung aus der hinduistischen Bhagavad Gita (Kapitel 2, Vers 23), einem der wichtigsten Bücher dieser Religion, zu zitieren, mit der das Selbst charakterisiert wird: “Waffen können ihm nichts anhaben, Feuer verbrennt es nicht, Wasser macht es nicht nass und Wind macht es nicht trocken.” So scheint es sich auch mit dem Geist zu verhalten. Reden Philosophen und Wissenschaftler also vielfach über eine Illusion?

Das Bieri-Trilemma

Ich will mich meinem Lösungsvorschlag mit einem Zwischenschritt annähern. Oben versprach ich, mich noch mit der Variante des Körper-Geist-Problems von Peter Bieri zu beschäftigen. Diese wurde mir in meinem Studium beigebracht und verwende ich in meiner eigenen Lehre noch heute. Im Folgenden habe ich allerdings “mentale Zustände” durch “psychische Prozesse” ersetzt:

(1) Mentaler Realismus: Es gibt genuin mentale/psychische Prozesse und diese sind nicht-physikalische Prozesse.
(2) Mentale Verursachung: Mentale/psychische Prozesse sind für den Verlauf der Welt kausal relevant.
(3) Kausale Geschlossenheit: Der physikalische Bereich ist kausal geschlossen; jedes physikalische Ereignis hat eine hinreichende physikalische Ursache.

Diese Formulierung ähnelt der von Westphal, kommt aber mit nur drei Prämissen aus. Das liegt im Wesentlichen daran, dass der britische Philosoph den Körper explizit als “physikalisches Ding” definierte, um den logischen Widerspruch leichter erkennbar zu machen. Bevor ich auf den Widerspruch komme, also das Körper-Geist-Problem im engeren Sinn, sei nun etwas mehr über den Inhalt der drei Annahmen gesagt:

Der mentale Realismus räumt den psychischen Prozessen ein eigenes Bestehensrecht ein. Sie sind etwas prinzipiell Anderes als physikalische Vorgänge. Das entspricht jedenfalls unserer Intuition, dass unser Wahrnehmen, Fühlen, Denken, Entscheiden und Handeln etwas ist, das nur im belebten Teil der Natur vorkommt, in einem anspruchsvolleren Sinne nur in den höheren Lebewesen und in anspruchsvollster Weise vielleicht sogar nur in uns Menschen. (Vertreter von Computerintelligenz oder künstlichem Bewusstsein mögen mir verzeihen, dass ich dieses Thema ausklammere, um den Aufsatz nicht zu lang werden zu lassen.)

Die Idee der mentalen Verursachung drückt aus, dass psychische Prozesse nicht nur als Beiprodukte entstehen, sondern auch etwas in der Welt bewirken. Das entspricht auch unserer Intuition, denn wir erfahren unsere Entscheidungen und Handlungen als Ergebnisse unserer Wahrnehmungs-, Gefühls- und Denkprozesse. Dass unsere Handlungen, etwa jetzt mein Schreiben dieses Artikels, etwas in der Welt bewirken, beispielsweise Ihr Lesen desselben, lässt sich kaum bezweifeln. Die Frage ist hier aber, ob die Handlungen wirklich kausale Folgen psychischer Prozesse sind. Vielleicht findet die ganze kausale Wechselwirkung doch nur auf der physikalischen Ebene statt?

Jetzt bleibt noch die dritte Annahme, die der kausalen Geschlossenheit. Über die Frage, was Kausalität ist, wurden ganze Bände geschrieben. In dieser Tiefe können wir das Thema hier nicht behandeln; das brauchen wir aber auch nicht. Wir können es hier bei der Intuition belassen, dass das Netz von Ursachen und Wirkungen in der Natur und dann insbesondere in der Physik keine Lücken aufweist. Jedes physikalische Ereignis hat dann eine vollständige physikalische Ursache. (Dass das Prinzip der Kausalität in der Physik auch durch Entdeckungen der Quantenmechanik an Bedeutung verloren hat, wäre hier ein berechtigter Einwand, bedeutet für das Körper-Geist-Problem erst aber einmal nichts.)

Über jede der drei Prämissen kann man lange philosophische Diskussionen führen und dabei auch auf wissenschaftliche Befunde verweisen. Für uns genügt es hier erst einmal, dass sie nicht völlig abwegig sind, und die Annahme von zweien jeweils die Dritte ausschließt: Wenn psychische Prozesse nicht-physikalisch und doch kausal wirksam sind, wie kann der physikalische Bereich dann kausal geschlossen sein? Immerhin sind die Folgen unserer psychischen Prozesse, beispielsweise die Tippbewegungen meiner Finger auf der Tastatur, ja auch physikalische Vorgänge, nämlich letztlich Bewegung von Atomen.

Nimmt man stattdessen mentale Verursachung an und hält den physikalischen Bereich für kausal geschlossen, wie können psychische Prozesse dann noch nicht-physikalisch sein? Und schließlich, drittens und letztens, wenn psychische Vorgänge nicht-physikalisch sind und der Bereich des Physikalischen kausal geschlossen ist, wie können sie dann noch kausal wirken? Es scheint aussichtslos: Jedes Mal kommen wir auf einen Widerspruch. In Anlehnung an das Wort “Dilemma” und den Namen des Autors nannte man diese Formulierung des Körper-Geist-Problems dann auch das “Bieri-Trilemma”.

Drei klassische Lösungswege

Interessanterweise – und das macht das Bieri-Trilemma auch so geeignet für die Lehre – kann man jeden Versuch, sich einer der Prämissen zu entledigen, einem bestimmten philosophischen Standpunkt zuordnen: Wenn man den mentalen Realismus ablehnt, dann landet man bei einer reduktionistischen Position, die darauf hinausläuft, dass psychische Vorgänge letztlich doch nur physikalische Prozesse sind. Formulierungen wie: “Der Geist ist nichts Anderes als das Gehirn.”, sind hierfür beispielhaft. Extrempositionen bestreiten ganz generell den Sinn der Redeweise vom Geist oder psychischen Prozessen. Diese nennt man darum auch “Eliminativismus”, worin das Wort “eliminieren” steckt.

Wenn man die These der kausalen Geschlossenheit ablehnt, dann landet man üblicherweise bei dualistischen Standpunkten. Damit stellt sich vor allem die Frage, inwieweit psychische Prozesse noch in ein naturwissenschaftliches Weltmodell hereinpassen. Wie wir gesehen haben, räumte beispielsweise Descartes der Seele eine eigene Seinsweise ein, die der denkenden Substanz. Zudem ging er davon aus, dass sie auf den Körper wirkt (und umgekehrt), und zwar vor allem über die Zirbeldrüse. Er konnte aber den postulierten Wirkmechanismus nicht erklären, nicht einmal plausibel machen.

Schließlich kann man noch die These der mentalen Verursachung ablehnen. Diesen Standpunkt nennt man dann Epiphänomenalismus. Ein Epiphänomen ist eine bloße Randerscheinung, eine Folge von anderen Prozessen ohne eigene Wirkung auf weitere Vorgänge. Ein klassischer Vertreter dieses Standpunkts war der Biologe und Philosoph Thomas H. Huxley (1825-1895), der dabei übrigens Descartes’ physiologische Gedanken weiterführte und auf den Menschen übertrug.

Auch heute noch vertreten manche Philosophen, Biologen und Neurowissenschaftler zwar den Standpunkt, dass es in einem starken Sinne psychische Prozesse gibt, diese aber keinen Einfluss auf der physikalischen Ebene haben können. Das degradiert den Geist oder die Psyche, wenn man es so nennen will, zum bloßen Zuschauer des Weltgeschehens.

Huxley war übrigens nicht nur der Großvater des Biologen Julian und des Schriftstellers Aldous Huxley, sondern auch ein großer Verteidiger von Darwins Evolutionstheorie. Während sich Charles R. Darwin (1809-1882) aus öffentlichen Debatten eher heraushielt, ging Thomas Huxley der Konfrontation auch mit einflussreichen Kirchenvertretern nicht aus dem Weg. Das brachte ihm den Spitznamen “Darwins Bulldogge” ein, für den er heute wohl bekannter sein dürfte als für den Epiphänomenalismus.

Nähere Analyse der Sprache

Wir haben jetzt schon viel gelernt – aber immer noch keine Lösung für das Körper-Geist-Problem. Ich denke, dass wir uns noch einmal genauer mit der Sprache beschäftigen müssen, die wir hier verwenden.

Die Meisten von uns würden sich wahrscheinlich der These anschließen, dass wir Wahrnehmungs-, Gefühls-, Denk-, Entscheidungs- und Handlungsprozesse haben. Ich sehe jetzt beispielsweise den Text auf dem Bildschirm, höre draußen ein Auto vorbeifahren, sehe durch das große Fenster aber auch die grünen Blätter vieler Bäume und höre Vogelzwitschern. Es ist ein sonniger Tag im Mai.

Ich weiß, dass ich zu viel am Computer sitze und spüre deshalb sogar schon Schmerzen im Nacken, in der Schulter und im Arm. Ich habe mir aber vorgenommen, unter anderem diesen Text noch vorm Juni fertigzustellen. Eine erste Version hatte ich schon vor über einem Jahr angefangen. Da ich zu diesem Thema aber schon meine Magisterarbeit im Jahr 2005 geschrieben habe, sind meine Erwartungen an mich selbst hoch: Ich will wirklich einen Schritt weiterkommen. Und es soll auch ein schöner Text werden.

Diese Faktoren – so erlebe ich es jedenfalls – spielen alle eine Rolle bei der Entscheidung dafür, jetzt diese Zeilen zu schreiben. Ich will nicht aufhören, bevor ich fertig bin, und denke mit Widersinn an den universitären Online-Kurs, den ich heute Abend noch leiten muss. Ich hatte ihn erst schon vergessen; danach kam mir der Gedanke, ob ich ihn schwänzen könne. Meine Studierenden machen das manchmal. Ich kann mir das als Kursleiter aber nicht erlauben.

Auf einen Teil dieser Vorgänge beziehe ich mich sprachlich, indem ich von der Entscheidung spreche, den Text zu schreiben. Die meisten Leserinnen und Leser werden vermutlich auch problemlos verstehen, was ich damit meine. Wenn ich jetzt aber einen genauen Zeitpunkt angeben soll, an dem diese Entscheidung stattfindet, gerate ich in Probleme. Ich würde gar sagen, dass sich dieser Vorgang schon tagelang hinzieht. Es ist ein Ringen mit mir selbst, das erst dann sein Ende finden wird, wenn dieser Text fertig ist.

Wissenschaftliche Untersuchung

Wer solche Phänomene, die ja zweifellos Teil unserer Welt sind, nun wissenschaftlich untersuchen will, steht vor vielen Problemen: Die sprachliche Formulierung, die ich hier vorgeschlagen habe, ist nämlich viel zu grob. Mit einer Beschreibung wie “der Entscheidungsprozess ist ein innerer Kampf, der sich schon tagelang hinzieht”, kann kein Experimentalpsychologe oder Neurowissenschaftler etwas anfangen. Früher habe ich ja selbst solche Versuche durchgeführt.

Um mit den gängigen psychologischen oder neurowissenschaftlichen Verfahren untersucht zu werden, muss ein Vorgang standardisiert und zeitlich eingegrenzt werden, idealerweise in einem Zeitfenster von wenigen Sekunden. Aufgrund der Variabilität von Messergebnissen muss er idealerweise auf Anweisung produzierbar und viele Male wiederholbar sein. Solche Herausforderungen gilt es zu meistern, bevor überhaupt mit einem Versuch angefangen wird.

Dann stellt sich die Frage, mit was für einem Messverfahren man das Phänomen untersuchen will. Die Elektroenzephalographie (EEG) wird seit fast 100 Jahren verwendet, misst elektrische Ströme an der Kopfhaut und ist zwar sehr schnell, dafür räumlich aber recht ungenau. Neuere Verfahren wie etwa die funktionelle Magnetresonanztomographie (fMRT) sind zwar räumlich genauer, dafür zeitlich aber sehr träge. Zudem misst sie ein Durchblutungssignal, das nur indirekt mit der Zellaktivierung im Gehirn zusammenhängt.

Niemand wird wohl bezweifeln, dass das Nervensystem und insbesondere das Gehirn eine bedeutende Rolle bei den genannten Vorgängen spielt. Das Nervensystem befindet sich aber auch in einem Körper und dieser in einer Umwelt. Für den Zustand von Körper und Umwelt sind außerdem Vorgänge in der Vergangenheit entscheidend. So weit dürfte Descartes auch schon gewesen sein und sogar Sokrates hat eingeräumt, wie wir am Anfang des Artikels gesehen haben, dass ihn in einem gewissen Sinne seine Gebeine ins Gefängnis gebracht haben.

Was wollen wir erklären?

Was gilt es aber denn zu erklären? Stellen wir uns ein aktuelleres Beispiel vor, dass ein Bankräuber vor Gericht steht und dort gefragt wird, warum er an einem bestimmten Tag die örtliche Sparkasse überfallen hat. Darauf seine Antwort: “Weil vom motorischen Kortex meiner Großhirnrinde Signale an die Muskeln gesendet wurden, die mich in die Sparkasse gehen ließen; Signale an meine Hände ließen mich dort mit der Pistole herumfuchteln und Signale an mein Zwerchfell, meine Stimmbänder und meinen Mund ließen mich schreien: ‘Geld her, oder es knallt!'”

In einem gewissen Sinne stimmt die Erklärung des Räubers, denn wir wissen aus Zeugenaussagen und Videoaufnahmen, dass sich die Tat so zugetragen hat. In einem gewissen Sinn wäre die Beschreibung auch wissenschaftlich richtig, denn all diese Vorgänge müssen im Körper stattgefunden haben, damit der Bankraub so ablaufen konnte. Das Gericht würde wahrscheinlich trotzdem glauben, dass der Bankräuber Witze macht oder die Befragung nicht ernst nimmt.

Dieses Beispiel macht deutlich, dass unsere Alltagsphänomene und auch die Erklärungen in unserer Gesellschaft von ganz anderer Art sind als die Phänomene, die Forscher im Labor untersuchen, und ihre Erklärungen. Im Einzelfall, wie hier, können die letztlich akzeptierten Erklärungen große Auswirkungen haben, etwa auf das Strafmaß. Psychologen und Hirnforscher können aufgrund der Beschränkungen ihres experimentellen Ansatzes und ihrer Methoden aber immer nur ein kleines Steinchen des großen und bunten menschlichen Mosaiks untersuchen. Und das ist oft schon herausfordernd genug!

Denken wir zum Vergleich an das Dreikörperproblem aus der Physik: Schon bei der Vorhersage der Bahn von drei Himmelskörpern, auf die die Schwerkraft wirkt, stößt unsere Mathematik (bislang) an ihre Grenzen. Das Verhalten dieser Objekte erscheint uns chaotisch, selbst wenn die Kräfte, die auf sie wirken, deterministisch sind. Durch eine Simulation lassen sich, sozusagen durch Herumprobieren, lediglich Näherungslösungen finden.

Komplexität

Nun hat das Gehirn eines erwachsenen Menschen ungefähr 86 Milliarden Neuronen, die oftmals viele tausende, mitunter gar zehntausende Verbindungen zu anderen Nervenzellen haben. Dazu kommen andere Zelltypen, deren Funktion heute noch gar nicht ganz klar ist. Außerdem gibt es noch komplexe Regelkreise, die Zellen aktivieren oder auch unterdrücken können. Bereits wenige Nervenzellen verhalten sich aber chaotisch, also so, dass selbst die besten Wissenschaftler ihre Reaktionen nicht mehr vorhersagen können. Bis auf Weiteres ist hier keine Änderung in Sicht.

Also selbst wenn Sokrates’ Entscheidung für den Giftbecher und gegen die Flucht, selbst wenn meine Entscheidung für das Schreiben des Textes und selbst wenn die Entscheidung des Bankräubers für den Überfall der Sparkasse vollständig im Gehirn determiniert war, können wir dafür mit wissenschaftlichen Mitteln schlicht keine Ursache-Wirkungs-Kette auf neuronaler Ebene angeben. Ob wir es jemals können werden, ist reine Spekulation.

Fest steht aber, dass wir wichtige Informationen verlieren würden, wenn wir darum die alltäglichen Erklärungen aufgäben: Sokrates wollte seinen Idealen treu bleiben, ich wollte die Abgabefrist einhalten und der Bankräuber wollte mehr Geld haben, um damit bestimmte Ziele zu verwirklichen.

Es ist eine Eigenschaft unserer Alltagssprache, mit einem Ausdruck wie “die Entscheidung, X zu tun” einen Sachverhalt ausdrücken und in einen Sinnzusammenhang stellen zu können, der tatsächlich stattgefunden und zudem eine erklärende Funktion hat. Die Wissenschaftssprache ist von ihrem Anspruch her zwar genauer, in diesem erklärenden Kontext aber unterlegen, weil die unterliegenden Ursache-Wirkungs-Zusammenhänge zumindest bis auf Weiteres unerreichbar sind.

Ein Computervergleich

Stellen wir uns zum Vergleich einen modernen Computer vor: Damit ein Programm vom Prozessor ausgeführt werden kann, muss es an einen bestimmten Punkt in Maschinensprache vorliegen. Das ist sozusagen die Sprache, die der Prozessor versteht. Für uns Menschen ist diese Sprache aber eher ungeeignet, sodass wir, zumindest bei komplexeren Programmen, eine bestimmte Programmiersprache verwenden müssen. Ein bestimmtes Programm, ein sogenannter Compiler, übersetzt diese für uns hinterher in Maschinensprache.

Wenn nun bei der Ausführung des Programms etwas schiefläuft, wurden vom Prozessor zwar die Anweisungen in Maschinensprache ausgeführt. Ab einem gewissen Komplexitätsgrad ist es aber für Menschen schlicht nicht mehr realistisch, den Fehler auf dieser Ebene zu erklären. Dafür muss die Anwendung in der Programmiersprache untersucht, gegebenenfalls geändert und dann wieder neu in Maschinensprache compiliert (übersetzt) werden. Die Programmiersprache hat also eine bestimmte erklärende und nützliche Funktion, die durch die Maschinensprache nicht realistischerweise ersetzt werden kann, selbst wenn der Prozessor tatsächlich immer nur Maschinensprache ausführt.

Extremer Reduktionismus

Vergleichen wir das mit dem, was der theoretische Physiker Sean Carroll vor ein paar Jahren schrieb:

“Alles, was wir brauchen, um alles zu erklären, was wir in unserer Alltagswelt sehen, sind eine Handvoll Partikel – Elektronen, Protonen und Neutronen –, die mittels einiger Kräfte – der nuklearen Kraft, Gravitation und dem Elektromagnetismus – und gemäß der grundlegenden Regeln der Quantenmechanik und der allgemeinen Relativität miteinander Interagieren.”

Der theoretsiche Physiker Sean Carroll

Carroll arbeitet am renommierten California Institute of Technology, ist einer der führenden Physiker auf seinem Gebiet und zudem sehr aktiv in der Wissenschaftskommunikation tätig. Hier behauptet er nun, man könne unsere gesamte Alltagswelt in der Sprache der Physik, mit nur drei Arten von Teilchen, drei Arten von Kräften und ein paar weiteren Regeln erklären. Wenn das stimmte, wären mit einem Schlag nicht nur die gesamten Geistes-, Kultur- und Sozialwissenschaften sowie die Psychologie, sondern gar Neurowissenschaften, Biologie und Chemie überflüssig.

Der Physiker vertritt hier einen extremen Reduktionismus, der in dieser Stärke beim heutigen Kenntnisstand offensichtlich falsch und mit Blick auf den Fortgang der Wissenschaften eher unwahrscheinlich ist. Doch auch verschiedene Hirnforscher äußerten sich ähnlich, als sie behaupteten, den gesamten Menschen vom Gehirn aus erklären zu können. Interessanterweise würden auch diese Wissenschaftler arbeitslos, wenn die noch radikalere Sichtweise des theoretischen Physikers Carroll zuträfe.

Überlegen Sie einmal selbst, wie weit Sie in Ihrem Leben noch kämen, wenn Sie nur noch über Elektronen, Protonen und Neutronen sowie deren Wechselwirkungen sprechen könnten. Und das gilt in Analogie auch für die Hirnforschung und die Psychologie: Diese untersuchen und erklären zwar bestimmte Teilsysteme des Menschen, denken wir an Bewegung, Hören, Sehen oder Sprechen. Eine Erklärung des großen Ganzen scheitert aber sowohl an experimentellen als auch an methodischen Einschränkungen.

Nützliche Alltagssprache

Unsere Alltagssprache aber ist gerade so nützlich, weil sie in einem gewissen Maß ungenau ist. Es gibt physiologisch unendlich viele Weisen, das Mobiltelefon auf meinem Schreibtisch zu nehmen: mehr von links, von rechts oder von oben, mal schneller mal langsamer und so weiter. Diese fallen alle unter die eine Beschreibung: “Stephan nahm das Mobiltelefon auf dem Schreibtisch.”

Für den Sinnzusammenhang und das Verständnis dessen, warum ich das tat, nämlich um nach neuen Kurznachrichten zu schauen, sind die Unterschiede auf der physiologischen Ebene irrelevant. In ähnlicher Weise kann die Untersuchung von Sokrates’ Gebeinen keine Antwort auf die Frage geben, warum der Philosoph im Gefängnis blieb und nicht in die Hafenstadt Megara floh.

Unsere Alltagssprache ist vielleicht unscharf. Sie ist darum aber nicht falsch. Und diese Unschärfe macht sie erst so nützlich. Es spricht nichts dagegen, die Sprache präzisieren zu wollen. Man sollte aber nicht aus den Augen verlieren, was man damit bezweckt. Ebenso kann jemand die Fehler eines Computerprogramms in Maschinensprache zu erklären versuchen. Es mag ein paar Spezialfälle geben, in denen das sinnvoll ist. In aller Regel wird man damit aber Zeit verschwenden. Und welchen Erkenntnisgewinn bringt das? Ebenso wissen wir doch heute schon, dass das Nervensystem eine entscheidende Rolle beim Verarbeiten von Wahrnehmungen, Gefühlen, Gedanken, beim Entscheiden und Handeln spielt.

Kommen wir am Ende zum Körper-Geist-Problem zurück: Die Suche nach Leib-Seele- oder, moderner gesprochen, Körper-Geist-Interaktionen ist unsinnig, weil es sich hierbei um keine zwei unterschiedlichen Dinge wie zwei Kugeln auf dem Billardtisch handelt. Die seit Jahrhunderten andauernde Suche nach dieser Interaktion war sinnlos, weil die Frage falsch formuliert wurde. Nicht wenige Philosophen, Psychologen und Neurowissenschaftler glauben, den Leib-Seele-Dualismus hinter sich gelassen zu haben, sprechen aber tatsächlich immer noch in dualistischer Sprache.

Mein Lösungsvorschlag

Das Problem lässt sich wie folgt auflösen: Psychische Vorgänge gibt es, weil psychische Beschreibungen zwar nicht immer, doch oft genug funktionieren und sinnvoll Sachverhalte dieser Welt beschreiben. Natürlich sind diesen Vorgängen im konkreten Einzelfall körperliche, hirnphysiologische, ja sogar physikalische Prozesse zuzuordnen.

Das hilft uns in der Praxis aber nichts, weil wir die Vorgänge auf dieser Ebene gar nicht beschreiben können. Insbesondere können wir damit keine Sinnzusammenhänge herstellen, mit denen sich Fragen wie: “Warum blieb Sokrates im Gefängnis, anstatt zu fliehen?” Oder “Warum tippte Stephan am Computer, anstatt sich im Park die Sonne auf den Bauch scheinen zu lassen?”, beantworten ließen.

Wenn das nächste Mal ein Hirnforscher kommt und sagt: “Du bist nichts als dein Gehirn!”, dann sollten wir erwidern: “Ach ja? Dann erklären Sie doch mal dieses und jenes.” Und ergänzend: “Haben Sie noch nicht gehört, was führende Physiker sagen? Sie sind nichts als Elektronen, Protonen und Neutronen! Mehr Präzision darf ich von einem Wissenschaftler schon erwarten.”

Philosophen haben sich auch um mehr Genauigkeit bemüht und beispielsweise intentionalen sowie phänomenalen Gehalt als entscheidende Merkmale psychischer Vorgänge identifiziert. Ersterer bedeutet, dass sie von etwas handeln, beispielsweise Sokrates’ Strafe, meinem Text oder Ihren Wünschen; letzterer bezieht sich darauf, wie es sich anfühlt, diesen Vorgang zu haben, den subjektiven Erlebnisgehalt. Manche Philosophen haben das wieder reifiziert und sprechen von “Qualia”, als ob es sich um kleine Bewusstseinsatome handeln würde, die irgendwo im Raum-Zeit-Kontinuum existieren. Ich fürchte, dass uns diese verdinglichte Sprache wieder mehr Probleme einhandelt, als dass sie etwas erklärt.

Das fehlende Subjekt

Einen wichtigen Punkt unterstreichen die philosophischen Überlegungen aber doch: Psychische Vorgänge – jedenfalls diejenigen in einem reichhaltigeren Sinne – sind immer Vorgänge von jemandem, einem Subjekt. Darum halte ich es für problematisch, dass mit der Psychologie und der Hirnforschung führende Wissenschaftszweige im Laufe des 20. Jahrhunderts die Phänomenologie und Introspektion als “unwissenschaftlich” aus ihrem Reich verbannt haben. Aus der Außenansicht lässt sich vielleicht ein Teil des Menschen erklären, aber eben nicht alles.

Viele Laien haben das auch verstanden und wählen darum lieber Literatur über östliche Philosophien, die die Subjekt-Objekt-Trennung nicht so vollzogen haben. Oder sie präferieren Lebensphilosophie und Phänomenologie gegenüber standardisierten quantitativen Studien, die zwar nach den heute herkömmlichen Regeln der Wissenschaft vorgehen, doch letztlich für das Menschenleben irrelevante Sachverhalte aufklären.

Die antike Philosophie war übrigens noch nicht so zerrissen und stand mitten im Leben. Für Sokrates wurde das allerdings zum Verhängnis. Hätte er darum aber besser in hochspezialisierten Fachzeitschriften publizieren sollen, deren Sinn kaum noch jemand versteht? So bleibt mir nur noch, auf das Bieri-Trilemma zurückzukommen und es wie folgt aufzulösen:

(1) Mentaler Instrumentalismus: Es gibt genuin mentale/psychische Prozesse, sofern sich deren Beschreibungen auf Sachverhalte in der Welt beziehen.
(2) Mentale Sinnstiftung: Mentale/psychische Prozesse sind für das Verständnis von Sinnzusammenhängen in der Welt notwendig.
(3) Kausale Irrelevanz: Kausale Erklärungen sind überbewertet und für den Fortschritt vieler Wissenschaftszweige von geringerer Relevanz.

Damit lautet die Schlussfolgerung: Der Mensch ist sowohl Kulturwesen als auch Naturgegenstand. Hier gibt es keinen Widerspruch. Und es bleibt ein Faszinosum, wie die Bewusstseinserlebnisse und Kulturleistungen des Menschen in einem Körper und insbesondere den 1,5kg Zellgewebe und Verbindungen des Gehirns entstehen können. Anstatt “den Geist” auf körperliche Prozesse reduzieren zu wollen, könnte man es auch umdrehen: Körper ist Geist.

Die Welt kann einen philosophisch wie wissenschaftlich faszinieren.

Hinweis: Dieser Beitrag erscheint auch auf Telepolis – Magazin für Netzkultur.

Avatar-Foto

Die Diskussionen hier sind frei und werden grundsätzlich nicht moderiert. Gehen Sie respektvoll miteinander um, orientieren Sie sich am Thema der Blogbeiträge und vermeiden Sie Wiederholungen oder Monologe. Bei Zuwiderhandlung können Kommentare gekürzt, gelöscht und/oder die Diskussion gesperrt werden. Nähere Details finden Sie in "Über das Blog". Stephan Schleim ist studierter Philosoph und promovierter Kognitionswissenschaftler. Seit 2009 ist er an der Universität Groningen in den Niederlanden tätig, zurzeit als Assoziierter Professor für Theorie und Geschichte der Psychologie.

195 Kommentare

  1. [Bitte äußern Sie sich nur zum Thema, anstatt hier immer wieder dieselben politischen Äußerungen zu wiederholen. Danke. St. Schleim]

  2. Obwohl ich trotz meiner AKE Seele nicht genau beschreiben kann (der Zustand von fusioniertes/geistig-heilendes Bewusstsein im Freien Willen / im Vollbesitz der Kraft des Geistes), bin ich ziemlich sicher, dass bisher kein Verstorbener Seele im Sinne menschlich-reinkarnativen Lebens erlangt hat, denn MENSCH bedeutet nach wahrhaftiger Definition der biblischen/spirituellen Texte immer ALLE, “Einzelne/Individualbewusstsein” wird dort nicht angesprochen.

  3. @Schleim

    Ja, das war eben nicht nur das kleine Einmaleins, dass war Erfahrung gelebten Lebens, was Dir völlig abgeht, denn Du bist offensichtlich noch nie aus der Bildungsblase zu systemrationaler Suppenkaspermentalität heraus gekommen.

    In einer Diskussion von Auge zu Auge, live und mit viel Publikum, würdest Du sicher zerbrechen, denn da würde ich mir nicht mehr die Mühe mit sinngemäßen Wiederholungen machen 😏

  4. @hto: Ich lasse das jetzt einmal so stehen, damit andere Menschen sehen können, was Sie hier so fabrizieren. Sie haben jetzt aber für diesen Artikel Diskussionsverbot. Ich habe Sie oft genug gewarnt.

    Mir ist dieses Thema (das Leib-Seele-Problem) ein ernsthaftes Anliegen und ich würde mich freuen, wenn eine sinnvolle Diskussion hierzu möglich wäre. Sie stören dabei.

  5. Erstmal ein Dankeschön für dieses ausführliche kleine Einmaleins!

    Einige Anmerkungen zu einigen Aussagen:

    »(1) Der Geist ist ein nichtphysikalisches Ding.
    […]

    Hier will ich erst einmal die Aufmerksamkeit darauf richten, den Geist als “Ding” zu sehen. In der Fachsprache nennt man dies auch Reifikation, wortwörtlich “Verdinglichung”, von lateinisch res für “Ding”.«

    Nur um sicherzugehen: Diese Verdinglichung geschieht nicht erst dadurch, dass der Geist ein „Ding“ genannt wird, sondern bereits dadurch, dass man überhaupt von ‚dem‘ Geist spricht. Oder sehe ich das falsch?

    Im Folgenden schreibst Du:

    »Für unsere Zwecke ist die Feststellung hinreichend, dass man den Geist erst zu einem Ding macht und dann später Probleme bekommt, weil physikalische und nichtphysikalische Dinge nicht miteinander interagieren könnten.«

    Dieses „erst zu einem Ding machen“ liegt ja nun schon ziemlich lange zurück. Ohne diese Verdinglichung wäre es wohl kaum zu der Idee gekommen, es gäbe da ein „Körper-Geist-Problem“.

    Zum Bankräuberbeispiel:

    Mir fällt auf, dass der Bankräuber auf die Warum-Frage so antwortet, als sei es eine Wie-Frage gewesen. Dann passt seine Antwort nämlich (weitgehend).

    Zum Sean Caroll-Zitat:

    Ich verstehe das Zitat so, dass es in unserer Alltagswelt keine anderen Kräfte gibt als die physikalischen (die an den genannten drei Partikeln angreifen). Es gibt demnach keine Beobachtungen, für die wir andere Kräfte als Erklärung bemühen müssten, als eben diese physikalischen.

    Wenn man das so versteht, dann werden auch nicht die anderen, nichtphysikalischen Wissenschaften überflüssig. Aber die jeweiligen Aussagen dieser Wissenschaften dürfen nicht im Widerspruch zur Physik stehen, wenn die altbekannten (Körper/Geist-, Leib/Seele-)Probleme vermieden werden sollen. Ich denke nicht, dass Sean Caroll bestreiten würde, dass es unterschiedliche Beschreibungsebenen gibt. Diese gehen aber alle mit den bekannten physikalischen Gesetzmäßigkeiten konform—sofern sie für den betrachteten Gegenstand überhaupt relevant sind.

    Abschnitt Alltagssprache:

    »Kommen wir am Ende zum Körper-Geist-Problem zurück: Die Suche nach Leib-Seele- oder, moderner gesprochen, Körper-Geist-Interaktionen ist unsinnig, weil es sich hierbei um keine zwei unterschiedlichen Dinge wie zwei Kugeln auf dem Billardtisch handelt. Die seit Jahrhunderten andauernde Suche nach dieser Interaktion war sinnlos, weil die Frage falsch formuliert wurde.«

    Das unterschreibe ich sofort.

    »Nicht wenige Philosophen, Psychologen und Neurowissenschaftler glauben, den Leib-Seele-Dualismus hinter sich gelassen zu haben, sprechen aber tatsächlich immer noch in dualistischer Sprache. «

    Da sehe ich keinen Widerspruch zwischen dem Erkannten und dem Sprechen darüber. Wenn man sich darüber im Klaren ist, was es mit Begriffen wie Geist, Seele, Psyche, Idee usw. auf sich hat, dann ist die dualistische Sprechweise einfach ein probates Mittel, sich verständlich auszudrücken. Wir können die Entstehung der Sprache mit ihren Begriffen nicht einfach rückgängig machen und all die vielen dualistisch geprägten Begriffe einfach vermeiden.

    Soweit fürs Erste. Zu den Lösungsvorschlägen später …

  6. Ich komme ja aus einem musikalischen Bereich und würde sagen: wie kann ein Pianist sein Können unter Beweis stellen, wenn ihm ein Pianoforte nicht zur Verfügung steht? Doch ist dieser Vergleich statthaft? Ist der Geist ein Nomen, ein Ding, welches das Hirn nötig hat, wie der Pianist das Pianoforte? Das wäre wahrlich ein Dualismus. Und zugegeben, der Dualismus ist uns naheliegender denn all die Verrenkungen, die nötig sind, um den Geist zu leugnen.
    Geistige Fähigkeiten sagt man heute, ist stolz darauf, aber erklärt damit nichts -da haben Sie recht. Egal. Sie wollen, wenn ich Sie richtig verstehe, freilich eigentlich mit viel Text nur eines sagen: wie werden wir dem Menschen gerecht, wenn wir ihn nur auf das Feuern der Neuronen reduzieren? Die Hämmer und Saiten des Klaviers sollen den Pianisten erzeugen? Wollen uns so die Hirnforscher die Welt plausibel machen? Ich glaube und irgendwo schrieb ich das auch schon: man hat sich vom idealistischen Weltbild ein für allemal verabschiedet, weil man nicht mehr in die Welt der Quacksalber und Wunderheiler zurück will. Der Idealismus ist tot, so wie Gott tot ist, weil sie sich beide mit der Religion ins Bett gelegt haben. Kann man es den Menschen verübeln? Ich meine nein. Die ganzen Neurosen, die ganzen sexuellen Verklemmungen. Das braucht erst mal viele Jahrzehnte vielleicht Jahrhunderte, bis die Wunden verheilt sind und man über den Idealismus wieder offen reden kann. Dazu kommt eben, daß wir mit dem Geist nichts in der Hand haben. Wir sind nun mal dringend auf die Anschauung angewiesen, um uns eine wie auch immer geartete Existenz plausibel zu machen. Sie wissen ja, daß es sinnlos ist, sich auf irgendeine dahergelaufene Autorität zu berufen, da sie sich irren kann. Und im Narzißmus unserer Zeit ist das eher die Regel denn die Ausnahme. Wer also einen wie auch immer gearteten Geist behauptet, muß ihn beweisen -das sind heute die wissenschaftlichen Regeln. Diese Regeln waren und sind notwendig, da sonst jeder Unsinn fürwahr gehalten werden kann und ein Aufweichen jener Regeln die Wissenschaften beliebig machen würden. Die Antike, die vor Ihrem geistigen Auge zu neuem Leben erwacht, ist wie das Paradies, welches mit den Schwertern der Cherubim bewacht wird. Da können wir nicht mehr hin. Das ist das von Nietzsche erkannte Herumschleichen um das Grab Gottes. Denn wir können ja nicht mehr unbefangene Suchende sein, so wie es die Philosophen der Antike noch waren. Wir stehen auf den Schultern der Giganten der Aufklärung -um die können wir keinen Bogen machen ohne uns der sonst ergebenen Lächerlichkeit preiszugeben.

  7. Sie schmeißen das Kausalprinzip über Bord, so wie Sabine Hossenfelder die Vorhersagekraft (für Experimente, an die vielleicht vorher niemand gedacht hatte) von physikalischen Theorien nicht für erforderlich hält. Koinzidenz, natürlich, aber ich bin nicht begeistert.
    Kausalketten nachzuvollziehen ist gleichsam Brot und Butter des ganzen technischen (und gesellschaftlichen) Fortschritts. Der TÜV wurde gegründet, um Dampfkesselexplosionen zu verhindern. Bessere Behandlung ethnischer wie sexueller Minderheiten beruht auf der Erwartung, dass dies Vorteile für besagte Minderheiten bedeutet, die sich für die Gesamtgesellschaft fortsetzen.
    Meine Position zum Leib-Seele-Problem ist Reduktionismus.
    Damit handele ich mir natürlich auch ein, dass es keinen freien Willen geben kann.

  8. Die buddhistische Philosophie geht seit ca. 2500 Jahren davon aus, dass unser Bewusstsein und die Empfindung von Zeit-Dauer nur eine Illusion sind – die sich aus der Art und Weise ergibt, wie wir denken.

    Wenn man nur die Philosophie unserer Kultur für seine Überlegungen in Betracht zieht, dann macht man den Fehler, dass man damit mögliche Lösungen eines Problems nicht erkennen wird.

  9. @Balanus: Verdinglichung & Sprache

    Die Reifikation setzt meiner Meinung nach dort ein, wo man das Substantiv “Geist” verwendet, ob mit oder ohne Artikel (der Geist). Das gilt ebenso fürs englische “mind”.

    Ich habe aufzuzeigen versucht, dass man nichts verliert, wenn man stattdessen von “psychischen Prozessen” spricht, aber Klarheit gewinnt: Das sind eben keine festen Dinge, sondern sich dynamisch in der Zeit verändernde Vorgänge.

    An Stelle der Frage, wie denn der Geist mit dem Körper interagieren könne (oder umgekehrt), die meiner Meinung nach schon unsinnig und irreführend ist, kann man dann eher die Frage stellen: Was ist an diesem konkreten Prozess denn psychisch?

  10. @wereatheist: kausale Irrelevanz

    Wenn Sie noch einmal genau nachschauen, wie “kausale Irrelevanz” definiert ist, dann steht dort nicht, dass kausale Erklärungen völlig irrelevant sind, sondern dass sie überbewertet sind.

    Natürlich sucht man nach kusalen Zusammenhängen – aber doch nicht nach “den Ursachen”. Z.B. wenn man untersucht, dass Rauchen das Risiko für Lungenkrebs erhöht: Dann hat man einen kausalen Zusammenhang festgestellt, nicht “die Ursache” gefunden. So einfach ist die Realität nicht.

  11. “Leib-Seele-Problem / Kulturwesen” – Ich bin angewidert von soviel ignoranter Arroganz auf den Knochen von Toten und Lebenden – das alles nur, weil die katholische Kirche ihr missbräuchliches Machtmonopol in einem ebenso stumpfsinnig-zeitgeistlichen Spaltungsprozess, über “Aufklärung”, dem nun “freiheitlichen” Wettbewerb hingeben musste (ein psychischer Prozess der mit dem Geist nichts zu tun hat!).

  12. @hto: letzte Warnung

    Ich empfehle Ihnen dringend, einmal draußen spazieren zu gehen, um auf andere Gedanken zu kommen. Sie haben hier aufgrund Ihrer beleidigenden, ja schon drohenden Äußerungen nach vielen vielen Vorwarnungen Diskussionsverbot bekommen. Einige Ihrer weiteren Äußerungen, mit denen Sie dieses Verbot ignorierten, landeten schom im Spam-Ordner.

    Wenn Sie sich jetzt nicht endlich an die Regeln halten, fliegen Sie hier zeitlich unbeschränkt bei MENSCHEN-BILDER raus.

  13. [Auf Wiedersehen und alles Gute! Sie hatten für diesen Beitrag aufgrund Ihres Verhaltens schon Diskussionsverbot. Jetzt haben Sie allgemeines Hausverbot auf MENSCHEN-BILDER. St. Schleim]

  14. Hausverbot wegen anhaltender Verstöße gegen die Diskussionsregeln für hto.

    Ich bitte Sie, auf die störenden Kommentare dieses Nutzers nicht mehr zu antworten. Das ist das dritte Hausverbot in der fast dreizehnjährigen Geschichte von MENSCHEN-BILDER.

    Wie ich in den Regeln schreibe, werden die Diskussionen hier prinzipiell nicht moderiert. Wo ich aber weiter Kommentare von hto finde, werden diese sowie Antworten darauf nun kommentarlos in den Spam-Ordner verschoben.

    Manche Menschen haben im Leben nichts Wichtigeres zu tun, als andere zu stören. Das Forum hier ist aber für diejenigen gedacht, die sich ernsthaft mit den Themen auseinandersetzen möchten.

    P.S. Ich versuche, ein Buchmanuskript fertigzustellen. Für so einen Quatsch fehlt mir die Zeit.

  15. “Ich habe aufzuzeigen versucht, dass man nichts verliert, wenn man stattdessen von “psychischen Prozessen” spricht, aber Klarheit gewinnt: Das sind eben keine festen Dinge, sondern sich dynamisch in der Zeit verändernde Vorgänge.”
    Sie schreiben zu Recht: man verliert nichts. Aber man gewinnt auch nichts. Was ändert sich, wenn man statt von Geist/Seele nun von “psychischen Prozessen” spricht?
    Auch wenn man den Geist in der Sprache substantiviert, so sehe ich nicht, daß die Altvorderen, die dem Idealismus zuneigten, den Geist als etwas statisches verstanden. Auch wenn Sie von “psychischen Prozessen” sprechen, so bleiben Sie ja dann trotzdem dem Dualismus verhaftet und stehen dann immer noch vor dem Problem, daß Sie die Interaktion zwischen den “psychischen Prozessen” und des Hirns klären müßten. Oder aber man zieht sich auf den Standpunkt zurück, daß das Feuern der Neuronen die psychischen Prozesse hervorbringt. Das wäre im Falle Sokrates’ so, daß die Determinierung seines Hirns ihn gar nichts anderes hätte empfinden lassen können als seinen Idealen treu zu bleiben und nicht wegzulaufen. Das aber würde zu kurz springen. Das Hirn ist ja nicht von der Umwelt losgelöst, sondern interagiert mit ihr, da wir ja sonst noch heute in der Höhle sitzen würden. Ist also alles Umwelt und Hirn und deren Interaktion -und nichts außerdem?

  16. @Hilsebein: Beschreibungen

    Der Prozess X ist ein psychologischer Prozess unter dieser Beschreibung. Die Frage nach der Interkation (womit? mit sich selbst?) stellt sich dann gar nicht.

    Oder um es einmal anders zusammenzufassen:

    Mein Artikel verhält sich neutral zum ontologischen Monismus, vertritt aber die Notwendigkeit eines epistemischen Pluralismus.

  17. @Dietmar Hilsebein // 01.06.2020, 10:49 Uhr

    »Ist also alles Umwelt und Hirn und deren Interaktion -und nichts außerdem?«

    Ja, das steht zu hoffen…

    Beziehungsweise, wenn man‘s genau nimmt: Agieren tut die Umwelt ja nun eher nicht (abgesehen von den „höheren“ Lebewesen, die ja auch zur Umwelt zählen).

  18. “Der Prozess X ist ein psychologischer Prozess unter dieser Beschreibung. Die Frage nach der Interkation (womit? mit sich selbst?) stellt sich dann gar nicht.”

    Einverstanden. Wenn sich aber die Frage nach der Interaktion nicht stellt, so wird auch das Leib-Seele-Problem nicht berührt. Man beschäftigt sich dann eben mit anderen Dingen, was durchaus auch seinen Reiz hat.

  19. Ein wahrhaft würdiges Thema für Pfingsten.
    50 Tage nach Ostern kamen die Jünger Jesu zusammen und der Hl. Geist kam über sie. Sie konnten auf einmal fremde Sprachen verstehen und sich in fremden Sprachen unterhalten.

    Schöner kann man doch nicht erklären, was mit Geist gemeint ist.

  20. Vielleicht sollte man vorsichtig sein, zu schnell zu fragen, ob Geist und Gehirn identisch sind. Vom Begriff des “Geistes” hat man vermutlich mehr, wenn man ihn als Verhältnis zur Welt versteht. Der “Geist”, auch seine subjektive Repräsentanz, das Psychische, sind nicht allein im Kopf, sondern auf die Welt bezogen.

    Lektüreempfehlungen, wenn auch schon etwas älter:

    1. Klaus Holzkamp: Grundlegung der Psychologie. Frankfurt/New York 1985. Das Buch rekonstruiert das Psychische aus der Phylogenese, expliziert von daher kommend den Unterschied zwischen Bedingtheit und Begründetheit menschlichen Handelns und ist auch im Hinblick auf die gesellschaftliche Natur des Menschen sehr erhellend. Hintergrund ist die Tätigkeitstheorie Leontjews.

    2. Manuel Bremer, Rationalität und Naturalisierung. Berlin 2001. Eine Habil, die sich mit der Geschichte von Vernunft und Verstand in der analytischen Philosophie beschäftigt und dabei auch gute Argumente zur Frage der Naturalisierbarkeit der Vernunft diskutiert.

  21. @Stefan Schleim Alltagspsychologie

    Wenn ich Sie richtig verstanden habe, favorisieren sie den Alltagsverstand. Nehmen wir einfach die Wahrnehmung aus unserer subjektiven Sicht, so wie wir im Alltag damit umgehen. Die Sprache und ihre Bedeutungen sind ganz gut an die Wirklichkeit angepasst, auch wenn keiner weiß, wie sich das konkret auf hirnorganischer Ebene wirklich verhält.

    Letztlich wäre eine Computersimulation eines komplettes Konnektoms eines menschlichen Gehirns vonnöten, um einen Ansatzpunkt zu haben, wie wir wirklich funktionieren. Und selbst wenn wir diese Simulation hätten, hieße das noch lange nicht, dass wir wirklich verstehen, was da abläuft.

    Aber egal: im normalen subjektiven Leben leben wir mitten in der Welt, und freuen uns hoffentlich des Lebens und der wunderbaren Natur, die uns umgibt. Und wir schaffen es auch, uns die Maschinen zu bauen, die das Leben erst so richtig einfach machen. Auch hier lernt man im Maschinenbaustudium nicht, wie wir im Detail funktionieren, aber sehr wohl wie man Kreativität so einsetzt, dass da hinterher Maschinen dabei herauskommen, die auch funktionieren.

    Was die grundsätzlichen Schwierigkeiten mit dem Geist-Seele-Problem angeht, da denke ich allerdings, das hier die gängige Interpretation der Physik vielleicht der Kern des Problems ist. Wenn es tatsächlich kosmische Geisteswelten gibt, die auch mit der Welt des Materials interagieren, so heißt das ja nicht, dass die Physiker dieses heute kennen müssten.

    Seit Newton versuchen Physiker mathematisch beschreibbare Regeln in der Natur zu finden, die möglichen Interaktionen mit einer Geisteswelt werden schlichtweg praktisch nie untersucht. Ich hätte auch eine Idee, wo genau hier eine Schnittstelle wäre: In der Quantenphysik von u.a. chemischen Reaktionen bauen sich Superpositionszustände auf, die dann regelmäßig zusammenbrechen und dann Zufallsergebnisse produzieren, von deren konkreten Verlauf dann die weitere Wirklichkeit abhängig ist. Ich vermute hier, dass die kosmische Geisteswelt diese Prozesse unter Kontrolle hat, und bei Bedarf ganz gezielte Zufälle beim Zusammenbruch dieser Wellenfunktionen realisieren kann. So nimmt dann die Geisteswelt hinreichen Einfluss auch auf Gehirnprozesse.

    Wenn jetzt unser Bewusstsein eine Synthese des kosmischen Bewusstseins mit dem lokalen Gehirn ist, hätte wir hiermit eine Schnittstelle der Interaktion der beiden Welten: der Geisteswelt wie der materiellen Welt. Und damit dann das Problem gelöst, theoretisch zumindest.

    Da ich hier keine Nachweise liefern kann, will ich aber wenigstens Werbung dafür machen, dieses vorerst als eventuell zukünftig begründbaren Mythos zu nehmen, um sich in der psychischen Existenz in einer bewusst lebendigen Welt mit persönlicher kosmisch-geistiger Perspektive zu bewegen. Im Gegensatz zu einer grundsätzlich sinnlosen Welt, in der man sich seinen Sinn in verzweifelter Suche und mit erheblicher Anstrengung erst suchen muss, mit signifikantem Anteil an Menschen, die dabei versagen und nie wirklich einen Sinn im eigenen Leben finden.

    Wer mit der eigentlich geistlosen Variante eines Modells für die menschliche Psyche zurecht kommt, der hat hier wenig Handlungsbedarf. Wer aber sowieso spirituelle Erfahrungen im Leben hat und damit ja auch irgendwie klarkommen muss, dem kann es eventuell helfen, hier ein Modell zu haben, in dem das möglich und sogar eigentlich selbstverständlich ist, dass man selbst zur Hälfte aus Geist besteht, und nur zur anderen Hälfte auf Basis von Gehirnprozessen existiert.

    Wobei ich jetzt auch nicht wirklich Werbung für religiöse Vereine machen will, aber ein Leben als teils Geist inmitten einer lebendigen und auch wunderbar geistigen Welt gefällt mir dann doch.

    @Balanus

    Agieren tut die Umwelt dann doch. Und das aus dem tiefsten Inneren heraus.

  22. Zu KRichard:
    Ich verstehe den Buddhismus anders. Das Bewusstsein ist keine Illusion sondern das eigene ICH ist die Illusion. Eine Form des Bewusstsein-nämlich das Gewahrsein- benötigen wir, um diese Illusion zu erkennen. Die Buddhisten bezeichnen diesen ” Erkenner” als den GEIST. Unser Geist hat die Fähigkeit sich selbst zu erkennen, also all die Muster aus denen dieses ICH zusammengesetzt ist. Die Hinduisten nennen diesen “Erkenner” das Brahman (Das Selbst bzw. kosmische Bewusstsein). Dieser kosmische Geist der Hindus wurde vor über 2500 Jahren bereits in den Veden beschrieben und ist unpersönlich . Erst andere Religionen die danach kamen, haben diesen GEIST in Form einer oberen Gottheit personifiziert. Buddha selbst stellt mit seinen Ansichten alle Götter in Frage bzw. auch den Begriff Seele. Für ihn sind wir nur ein Nicht-Selbst, eine sich laufend verändernde Form aus Gedanken, Gefühlen und Empfindungen.. Diese Form erkennt sich -wie beschrieben- über das reine Bewusstsein (Gewahrsein) permanent neu .

  23. Tobias Jeckenburger,
    In der Sprache unterscheidet man zwischen Psyche und Geist.
    Die Psyche ist ein wertfreier Begriff und beschreibt, wie wir denken und warum wir so denken, wie wir denken.
    Der Geist ist ein Wertbegriff, und meint das “richtige” Denken, der in Harmonie mit der Welt agiert.

    Und, wie Herr Kuhn angemerkt hat, sind Gehirn und Geist nicht identisch. Psychopathen können unerhörte Leistungen auf wissenschaftlichem Gebiet erbringen aber ihnen fehlt die Rückbindung (religio) mit den ethischen Werten.

    Der Volksmund ist oft sehr treffend, wenn sich jemand unverständlich verhält. Man sagt dann: ” Da sieht man mal wieder wessen Geistes Kind du bist “

  24. @ Schleim

    “Mentale Zustände” durch “psychische Prozesse” zu ersetzen, halte ich für eine sehr gute Idee.
    Zustände sind nicht „dynamisch“, das „lebende Gehirn“ ist hoch dynamisch, wenn man einmal vom Verwesungsprozess absieht, der eigentlich auch dynamisch verläuft.

    Ich meine, man sollte immer mehr auf das Konzept der Informatiker, „Prozessor – Prozess – Information zusteuern“. Es gibt eindeutige Kategorien, die Wechselbeziehungen können gut beschrieben werden. Es scheint sich bewährt zu haben.

    „Information“ sollte man allgemeiner als die Informatiker sehen, eher so wie die Nachrichtentechniker. Die haben z.b. mit „Sprache“ („Information“) auf vielfältige Weise zu tun.
    Mit Syntax, Semantik, wie ist Information realisiert (Wort, Schrift,….), wie kann sie „übertragen“ werden, wie kann sie technisch verarbeitet werden….

    Information ist das, was sozusagen „übrig bleibt“, wenn man alle Prozesse und jegliches an Hardware (z.B. auch die Schallwellen und natürlich die Datenträger) „extrahiert“, so dass nur noch abstrakte „Beziehungen“ übrig bleiben.

    Die Philosophen hatten nun einmal das Problem, dass sie Begriffsbedeutungen festlegen mussten, ohne die dahinter liegenden Sachverhalte wirklich zu verstehen. Man hatte keine Ahnung von der Funktionsweise des Gehirns und schon gar nicht von der technischen „Informationsverarbeitung“, die es erst seit relativ kurzer Zeit überhaupt gibt.

    In der Informatik werden die in einem Programm verwendeten Objekte „deklariert“, bedeutet mit einem Namen versehen und es wird ihnen eine besondere (auch neue) Bedeutung zugewiesen. Macht man dabei Fehler, treten Widersprüche auf und das Programm fliegt einem sozusagen um die „Ohren“.

    In der Philosophie bedeutet dies, es kommt zu Widersprüchen und man entfernt sich immer mehr vom realen Geschehen. Es scheint zweckmäßig, wenn man auf ein bewährtes Grundkonzept aufbauen kann.

    Dass „Information“ (als “Naturgesetze”) implizit am Anfang allen Geschehens stand, scheint klar.

    Auch den Reduktionismus von Carroll finde ich zweckmäßig.

    Zitat: „… Hier behauptet er nun, man könne unsere gesamte Alltagswelt in der Sprache der Physik, mit nur drei Arten von Teilchen, drei Arten von Kräften und ein paar weiteren Regeln erklären. …“

    Eine Sprache und Regeln (auch der Physik) sind mindestens implizit „Information“. Da sich die Komponenten „dynamisch“ verhalten müssen, verändern sich Zustände, man hat mit „Prozessen“ zu tun. Dass „Teilchen“ letztlich „Prozessorchen“ sind, ist auch klar, selbst wenn sie nur einen „Spin“ haben sollten.

  25. @Buddhismus/Hinduismus: Wo es Wiedergeburt gibt, hat man Dualismus. Wie interagiert mein Karma mit meinem Körper?

    @“psychische Prozesse” Ich würde lieber “biologische Prozesse” sagen, da in meinem Alltagsverständnis die Psyche, als selbständiges Konstrukt gedacht ist, das irgendwie psycho-somatisch mit meinem Körper interagiert.

    “Biologisch” weist auch eher darauf hin, dass der ganze Körper (z.B mit der Signalübertragung durch Hormone und andere Chemikalien) am Geist beteiligt ist und nicht nur das Hirn.

  26. @Stephan Schleim: kausale Irrelevanz

    Ich denke, wir sind d’accord, wenn es um die Relevanz der frühen kosmischen Inflation geht, für so ziemlich alles von dem, was der Fall ist: Interessiert niemand.
    Für psychische Prozesse (ich mag diesen Begriff) ist auch Sean Carolls ‘Only Atoms and The Void’ nicht hilfreich, aber eine (anderthalb) Ebene(n) höher, nämlich bei Biochemie und Wirkung ‘psychoaktiver’ Substanzen a.k.a. Drogen, angesetzte Betrachtung ist unvermeidlich.

  27. @Querdenker 01.06. 16:20 Buddhismus

    Das passt ja recht gut zu meinem Modell, was Sie hier über den Buddhismus schreiben. Wenn mein lokales Bewusstsein eine Synthese des kosmischen Bewusstseins mit meinem lokalen Gehirn ist, dann bleibt der kosmische Teil – als Gewahrsam – eher gleich, nur das Gehirn steuert ständig was neues bei, was oft nicht lange Bestand haben muss. Auch brauchen wir in dem kosmischen Bewusstsein als Ganzes keine personifizierten Gottheiten, nicht mal Spezialgottheiten und auch keine Seelen von Verstorbenen.

    Ich stell mir hier eine grundsätzlich durchgängige Organisation vor, die sich wie eine Art absolut gigantischer Großrechner verhält. Hier können wir als Menschenseele bzw. als psychisches Faktum nur in Verbindung mit einem laufendem und wachem Gehirn existieren. Das irdische Leben ist hier dann wie eine Art Computerspiel, das die Weltbühne als verbindlichen Spielraum nutzt und die einzelnen Akteure jeweils in ein eigenes, ziemlich langes und umfassendes Spiel des Lebens schickt.

    Dass wir gleichzeitig unseren eher universalen Geistesanteil haben, sorgt dann dafür, dass wir auch wenn möglich gut miteinander umgehen bzw. umgehen sollten. Das Leid des anderen geht uns hier durchaus was an, insbesondere wenn wir es dem Anderen antun. Aber auch Verbindung untereinander als Menschen wie auch zur restlichen Biologie des Planeten ist in diesem Zusammenhang folgerichtig.

    Insbesondere ist die Zerstörung unseres eigenen Planeten ganz und gar nicht im Sinne dieses Spiels des Lebens. Und eine gemeinsame Aktion unter uns Menschen, dies zu verhindern, ist vermutlich grundsätzlich von Seiten der kosmischen Geisteswelt her extra unterstützungswürdig.

  28. In Lehrbüchern der Philosophie of Science wird meist auf ein häufig auftretendes Missverständnis im Zusammenhang mit Erklärungen in den Wissenschaften aufmerksam gemacht.
    Im Abschnitt ‘Mein Lösungsvorschlag’ steht:
    Wenn das nächste Mal ein Hirnforscher kommt und sagt: “Du bist nichts als dein Gehirn!”, dann sollten wir erwidern: “Ach ja? Dann erklären Sie doch mal dieses und jenes.”
    Nun ist es in den Wissenschaften zwar erfreulich, jedoch keineswegs notwendig eine Erklärung zu haben. Wir kenne viele naturwissenschaftliche Phänomene für die wir keine Erklärung haben. Aber die Phänomene müssen empirisch nachweisbar sein, Intuition und Introspektion reichen nicht.

  29. @Querdenker: Buddhidmus
    mein Hinweis darauf, dass Bewusstsein und Zeit-Dauerr als Illusion gesehen werden – sollte anregen, darüber nachzudenken, wie lange ´Geist/Seele´ existieren können wenn es keine Zeit-Dauer in der Realität gibt.

    Die Frage nach der Dauer wäre wichtig für die Diskussion des Leib-Seele-Problems.

  30. @Balanus: Verdinglichung/Carroll

    Allgemein gesagt, findet Verdinglichung/Reifikation statt, wenn man über etwas spricht, als sei es ein Ding. Substantivierungen (der Geist, the mind) sind dafür Beispiele. Man muss also nicht zwangsläufig den Geist ein Ding nennen, wie das Westphal etwa tut, um ihn zu verdinglichen.

    Da Carroll ausdrücklich über die Erklärung von Alltagsproblemen spricht, müssen wir ja seinen Satz nun nicht uminterpretieren, damit er dir besser gefällt.

    Wenn man sich darüber im Klaren ist, was es mit Begriffen wie Geist, Seele, Psyche, Idee usw. auf sich hat, dann ist die dualistische Sprechweise einfach ein probates Mittel, sich verständlich auszudrücken.

    Dann ist es aber doch komisch, wenn jemand, wie Glannon, immer noch nach einer Interaktion zwischen Dingen suchen will, die es wahrscheinlich gar nicht gibt.

    Gerade in Philosophie und Wissenschaft sollten wir doch so präzise wie möglich formulieren.

  31. @wereatheist: Kausalprinzip

    Vielleicht haben Sie das mit der “kausalen Irrelevanz” falsch verstanden. Da steht ja nicht, dass nichts mehr kausal erklärt werden sollte, sondern dass kausale Erklärungen in der Praxis weniger relevant sind, als das häufig gedacht wird.

    Zum Weiterlesen möchte ich Ihnen Kants Überlegungen darüber empfehlen, dass Kausalität mehr eine Denkstruktur ist, die wir brauchen, um Sinn in der Welt zu sehen; oder Wittgensteins Überlegungen, dass Kausalzusammenhänge sprachlich immer auf Wenn-dann-Zusammenhänge reduziert werden können.

    P.S. In den Wissenschaften scheinen sich übrigens probabilistische Kausalbegriffe durchzusetzen: man spricht also nicht mehr von Einzelursachen, sondern allgemeiner und abstrakter von ursächlichen Faktoren.

  32. @Kuhn: Identität

    Hmm, wer nimmt hier denn eine Identität an? Die ontologische Frage wird doch bewusst von mir offen gelassen. Und auf der sprachlichen Ebene halte ich aus den genannten Gründen die psychischen Prozesse für hilfreicher.

    Haben Sie denn mal ein Beispiel dafür, was man nur erklären kann, indem man einen “Geist” annimmt?

    P.S. Interessant übrigens, dass noch jemand Klaus Holzkamp zitiert. Ich habe in Berlin ein paarmal alte Schüler von ihm kennengelernt. Das ist Kritische Psychologie, für diejenigen, denen das nichts sagt.

  33. @Jeckenburger: spirituelle Erfahrungen

    Ich bin gerade zu ermüdet, um darauf etwas Sinnvolles zu erwidern.

    Aber ein interessanter Gedanke ist doch, dass z.B. in den meisten Schulen der indischen Philosophie sogar die Gedanken als materiell und darum illusionär (Sanskrit mythia) angesehen werden. (So viel zum Gedanken, der westliche Reduktionismus sei neu.)

    Wenn man dann aber in tiefer Meditation erfährt, ein unsterbliches und unbeschränktes Selbst zu sein, dann sagt man auf einmal, diese Erfahrung/dieser Gedanke sei echt (vidya).

    Das scheint mir irgendwie nicht ganz konsequent zu sein, beziehungsweise schlicht vorauszusetzen, dass das wahr ist, was wahr sein soll, und alles andere falsch sein muss.

  34. @Elektroniker: danke für die freundliche Ergänzung; von Ihnen hätte ich erwartet, dass Sie etwas zum Maschinensprachebeispiel sagen würden. Tatsächlich musste ich beim Schreiben an Sie denken.

  35. @Omnivor: Karma

    Beim Karma sollte man die Möglichkeit berücksichtigen, dass sich indische Philosophen das schlicht ausgedacht haben, um eine bestimmte Vorstellung von Gerechtigkeit aufrechtzuerhalten, sprich: Wer in diesem Leben nicht für seine Sünden büßen muss, der wird das in einem folgenden Leben tun müssen.

    Das kennen wir doch auch aus dem Christentum ganz gut: Selbst wenn diese Welt ungerecht ist, erledigt das das “Jüngste Gericht” für uns.

    Ich bleibe doch schlicht zu kritisch, um mir so etwas vorstellen zu können.

  36. @wereatheist: Beschreibungen

    (Ich fürchte, ich habe Ihnen und @Balanus doppelt geantwortet; ich habe ein paar 80-Stunden-Arbeitswochen hinter mir und bitte um Verständnis.)

    Natürlich ist die neurobiologische Beschreibungsebene erforderlich. Die Frage ist aber doch, ob sie hinreichend wäre, um z.B. spirituelle Erfahrungen, wie sie etwa Herr Jeckenburger erwähnt, vollständig zu erklären, seien sie durch Meditation, Drogen/Medikamente, Rituale oder was auch immer ausgelöst.

  37. @Koller: Erklärungen

    Gut, dass Sie die Wissenschaftstheorie stärker in den Fokus rücken.

    Was würden Sie denn alles zulassen, um ein Phänomen “empirisch nachzuweisen”?

    Und Sie sprechen im Satz davor von “naturwissenschaftlichen Phänomenen”. Würden Sie das bitte in Bezug zum Leib-Seele-Problem setzen? (Oder mir anderen Worten: Inwiefern sind psychische Prozesse naturwissenschaftliche Phänomene oder auch nicht?)

    Die Frage, was denn eine “Erklärung” ist, erspare ich mir jetzt einmal, sonst werden wir hier nie fertig. 😉

  38. Zu Omnivor:
    “Wie interagiert mein Karma mit meinem Körper ?”
    Meiner Ansicht nach kann man den Begriff “Karma” sehr gut mit heutigen psychologischen Erkenntnissen bzw. überhaupt mit dem gesunden Menschenverstand erklären. KARMA ist für mich das Ergebnis all unserer Handlungen die wiederum das Ergebnis all unserer Bewertungsmuster, also unserer Gedanken und Gefühle sind. Hierzu könnte man frühkindliche Prägungen, genetische Prägungen, Lebenserfahrungen zählen. Bei den Medizinmännern hieß so etwas früher auch Ahnenreihe. Diese Prägungen-und hier beginnt die moderne Psychologie- haben Einfluss auf den Körper (Soma), da der Körper diese Gefühle gespeichert hat und somit auf das Körpergedächtnis, was sich in Form von inneren Spannungen(Empfindungen äußert),so Buddha .Wer sein KARMA also “erkennen” will, sollte seinen Körper beobachten , was Buddha in der ACHTSAMKEIT diffeniert .

  39. @Querdenker: Rettung des Karma

    Es gibt Versuche, “Karma” schlicht als Prinzip von Ursache und Wirkung zu verstehen. Dann stimmt es natürlich in einem gewissen Sinne: Wenn ich mehr und mehr Menschen schlecht behandle, dann nimmt die Wahrscheinlichkeit zu, dass diese mich ebenfalls schlecht behandeln werden.

    Bei östlichen Philosophien ist nach meinem Wissen Karma aber nicht ohne Wiedergeburt zu denken – und das übrigens selbst im Buddhismus, wo man dem “Selbst” eine eigenständige Existenz abspricht (anatman oder anatta). Das hat in buddhistischen Schulen natürlich zu ewigen Diskussionen geführt, was dann der “Träger” des Karma sein kann. Und mitunter wird dann statt einer “Seele” eine “Buddhanatur” eingeführt. Das klingt aber doch wieder sehr vage und ad hoc.

  40. @Stephan Schleim

    Mir erscheint ihre Lösung das Problem zur Seite zu schieben:
    “Es gibt genuin mentale/psychische Prozesse, sofern sich deren Beschreibungen auf Sachverhalte in der Welt beziehen.”
    Müssen die Sachverhalte bestimmter Art sein oder ist das einfach nur das, was auch immer da in der Welt ist? Die Lösung erscheint sich mir damit von der Substanz zu entfernen. Damit überdeckt man aber vielleicht das tatsächliche Problem, dass wir einfach nicht wissen, was psychische Prozesse eigentlich sind. Sind dies einfach vereinfachte Beschreibungen oder gibt es genuin andersartige Prozesse?

    Bei der kausalen Irrelevanz muss ich an die Diskussion über wissenschaftliche Erklärungen in den Geisteswissenschaften denken. Insbesondere Hempel hat hier ja eine breite Debatte hervorgerufen. Mich auf diese Debatte beziehen würde ich zu bedenken geben, dass manche Äußerungen, nur scheinbar keine Kausalität enthalten. Auch Äußerungen darüber, dass Herr Schleim sich dazu aufgerafft hat diesen durchdachten Artikel zu schreiben, enthalten zahlreiche kausale Hypothesen. Diese sind natürlich in anderer Form als physikalische Beschreibungen, aber es erscheint mir nicht so zu sein, dass man damit den Anspruch aufgibt, kausale Zusammenhänge zu beschreiben.

  41. @libertador: Erklärungen

    Nehmen wir einmal an, dass Hänsel Gretel hinterherspioniert, weil er denkt, dass sie (Gretel) ihn betrügt. Das beschreibt einen Sachverhalt in der Welt, der durch verschiedene Verhaltensweisen realisiert wird, die hier als “hinterherspionieren” beschrieben sind. Das beschreibt einen weiteren Sachverhalt in der Welt, der als “denken, dass Gretel ihn [Hänsel] betrügt” beschrieben ist. Die Sachverhalte treffen zu, wenn es denn tatsächlich stimmt, dass Hänsel sich auf diese Weise verhält und er das denkt.

    Wo ist das Problem? Damit sind die Sachverhalte und insbesondere die Frage, warum Hänsel sich verhält, wie er sich verhält, doch hinreichend erklärt. Und es sind psychologische Beschreibungen, weil Denkprozesse und so komplexe Verhaltensweisen in den Bereich der Psychologie fallen.

    (Es ließen sich natürlich weitere Fragen stellen und beantworten, etwa warum denkt Hänsel, dass Gretel ihn betrügt. Ist er in diesem Denken gerechtfertigt oder schlicht eine sehr eifersüchtige Person?)

    Wenn Carroll oder sonst irgendjemand eine bessere Erklärung mit Atomen, Protonen, Neutronen und so weiter anzubieten hat, dann bitte, lese ich mir die gerne durch.

  42. Stephan Schleim,
    eine klare Ansage : Körper ist Geist !
    So langsam dämmert es den Naturwissenschaften, die Atome bestehen aus noch kleineren Teilchen. Die noch kleineren Teilchen sind nur über die Energie erklärbar. Und die Energie als Begriff ist ein Konstrukt des Geistes, was sonst.
    So steht es schon in der Bibel: “Im Anfag war das Wort und das Wort war bei Gott und Gott war das Wort.”
    Den Mensch kann man biologisch sehen, man kann ihn psychologisch sehen und man kann ihn als Phänomen betrachten, das letztlich transzendent bleibt , zum Glück !
    Oder kann hier jemand erklären, warum er lebt ?

  43. Das was wir gerade erleben, sind Aluhut-tragende Zeitgenossen, die „kausale Erklärungen für stark überbewertet“ halten. Und ich frage mich, für welche „Wissenschaften“ nonkausale Erklärungen von Relevanz für ihren Fortschritt sind? Selbst Soziologen fragen nach Ursachen.
    Wir haben heute auf der Welt echte Probleme, Komplexe Phänomene wie der Klimawandel erfordern eine fächerübergreifende Zusammenarbeit. Meteorologen und Geophysiker entwickeln zusammen mit den Informatikern Vorhersagemodelle für das zukünftige Klima. Unsere Wirtschaft muss umgestaltet werden, um einen katastrophalen Temperaturanstieg zu verhindern. Dafür bedarf es neuer Technologien, was in das Fachgebiet der Ingenieure fällt. Politiker müssen den Umbau der Wirtschaft politisch forcieren, Juristen müssen mithelfen, internationale Verträge zu verhandeln, denn das Problem ist nur global zu lösen. Wirtschaftswissenschaftler müssen Wege aufzeigen, wie der Umbau der Wirtschaft zu finanzieren ist, die Liste ist viel länger. Um Pandemien, wie Covid-19 in Zukunft vermeiden zu können, müssen Biologen und Epidemiologen mit Sozialwissenschaftlern zusammenarbeiten. Wie soll das gehen, wenn Kulturwissenschaftler die Naturwissenschaftler für stark überbewerten halten, was ihre Vorhersagekraft anlangt?

    Im Übrigen ist das Körper-Geist-Problem viel älter und taucht mit dem Urknall auf: Warum gibt es neben der Materie auch noch immaterielle Gesetze, nach denen sich die Materie zu richten hat? Ich nenne es hier mal das „Materie – Physikalische Gesetze-Problem“.
    Ab da führt der Weg, in einer ununterbrochenen Kausalkette bis zum Menschen, ab der ersten Zelle gesteuert von der Evolution. Der Unterschied ist: Evolution ist Versuch und Irrtum, und dazu gehört auch die Alltagssprache und der hohler Bauch, Wissenschaft ist die Enträtselung von Gesetzmäßigkeiten..
    Beste Grüße
    Peter-Paul Manzel

  44. @ Stephan Schleim:

    “Haben Sie denn mal ein Beispiel dafür, was man nur erklären kann, indem man einen “Geist” annimmt?”

    Ich verstehe die Frage nicht, sie klingt auch nicht gut, fast, als ob nach einem Gespenst gefragt wird. Ich schrieb: “Vom Begriff des “Geistes” hat man vermutlich mehr, wenn man ihn als Verhältnis zur Welt versteht.” Das sollte eigentlich sehr nahe an Ihrer Position sein. Vielleicht haben Sie es nicht gesehen, weil Sie meine Anmerkung spontan an Kritik einsortiert haben und nach einem Gegenargument gesucht haben?

    Was die ontologischen Aspekte angeht: Da finde ich die Sicht Thomas Nagels sehr sympathisch, dass uns vermutlich noch die richtigen Begriffe fehlen, um ohne Annahme von Gespenstern oder ohne billigen Panpsychismus eine Beschreibung der Welt zu haben, die geistigen Phänomenen wie z.B. dem Bewusstsein (oder von mir aus subjektunabhängig auch einer logischen Beziehung) ontologisch gerecht wird.

    “Klaus Holzkamp … Ich habe in Berlin ein paarmal alte Schüler von ihm kennengelernt.”

    Also, dass Sie jetzt auch noch altersdiskriminierend auf mich reagieren, wäre nicht nötig gewesen 😉

  45. @Manzel: Ursachen

    Es kommt immer gut an, einen Autor gleich im ersten Satz als “Aluhutträger” zu beleidigen.

    Da Sie es hier ja besser wissen als alle anderen, formulieren Sie doch einmal aus, (1) was eine Ursache (vorzugsweise in einem sozialen Kontext) ist, (2) wie wir so eine Ursache feststellen und (3) was dann eine ursächliche Erklärung ist.

    Wenn Sie Ihre Hausaufgaben machen, können wir hier vielleicht ein ernsthaftes Gespräch führen.

  46. @Kuhn: Geist, zweiter Versuch

    Wir scheinen aneinander vorbeizureden. Sie schreiben vom Geist im Bezug zur Welt. Ich frage Sie, was Sie meinen. Sie wiederholen: Den Geist im Bezug zur Welt.

    Ich verstehe Sie nicht.

  47. @Stephan // 02.06.2020, 12:48 Uhr

    Ad Carroll et Glannon

    »Da Carroll ausdrücklich über die Erklärung von Alltagsproblemen spricht, müssen wir ja seinen Satz nun nicht uminterpretieren, damit er dir besser gefällt.«

    Ich interpretiere keineswegs um, sondern nehme den Begriff ‚Erklärung‘ bzw. ‚erklären‘ („to account for“) schlicht so, wie er von Carroll gemeint sein dürfte und auch im alltäglichen Sprachgebrauch üblicherweise verwendet wird. Mir ist schleierhaft, wie man das missverstehen kann. Außer man macht das absichtlich, weil man eine bestimmte Agenda verfolgt, in der Hoffnung, es merkt keiner.

    »Dann ist es aber doch komisch, wenn jemand, wie Glannon, immer noch nach einer Interaktion zwischen Dingen suchen will, die es wahrscheinlich gar nicht gibt.«

    Das ist in der Tat sehr seltsam. Einerseits.

    Andererseits aber auch wieder verständlich, weil es die Herangehensweise an die psychologischen oder psychiatrischen Fragestellungen enorm vereinfacht.

    Es erleichtert sehr die Arbeit, wenn die höheren Hirnfunktionen, die ohnehin kaum darstellbar sind, unter dem Begriff Mind/Psyche subsummiert werden.

    Glannon scheint es aber noch etwas anders zu sehen, also nicht nur pragmatisch wie z. B. ich (und die Mehrzahl der Neurobiologen). Bei ihm scheint „Geist“ ein emergentes Phänomen zu sein, basierend auf den ihm unterliegenden neuronalen Aktivitäten.

    Ich fürchte nur, mit dieser Art von „Emergenz“ (in der Biologie gibt auch andere Formen von Emergenzen) verhält es sich ganz ähnlich wie mit der „Ursache“: Das sind alles Dinge, die im Auge des Betrachters liegen (womit ich aber nicht gesagt habe, das Kausalitätsprinzip sei irrelevant und/oder werde überschätzt—das ist mitnichten so).

  48. @Balanus: Erklärungen

    Gut, dann nehmen wir Carrolls Zitat auf englisch; immerhin sind wir ja Fachleute, nicht wahr?

    All we need to account for everything we see in our everyday lives are a handful of particles — electrons, protons, and neutrons — interacting via a few forces — the nuclear forces, gravity, and electromagnetism — subject to the basic rules of quantum mechanics and general relativity. […] That’s a remarkably short list of ingredients, to account for all the marvelous diversity of things we see in the world.

    Was heißt diese Aussage jetzt anderes, als dass man zum Erklären von allem, was wir in unserem Alltagsleben sehen, nur drei Arten von Partikeln, drei Arten von Kräften und ein paar physikalische Regeln braucht?

    Jetzt bin ich gespannt. Deine Gedanken kann ich leider nicht lesen. (Hast du überhaupt Gedanken? Oder nur Neuronenfeuern?)

  49. @ Schleim

    “Deine Gedanken kann ich leider nicht lesen. (Hast du überhaupt Gedanken? Oder nur Neuronenfeuern?)”

    Eben, weil Sie die Gedanken nicht sehen. Was wir sinnlich nicht erfassen können, darüber können wir auch nichts wissen. Aber es heißt ja nicht: das, was wir nicht sehen und nicht mit der Handvoll Teilchen erklären können, gibt es nicht. Der schwarze Schwan ist immer möglich, gesetzt, es fügt sich, daß ein menschliches Wesen, meinetwegen auch mit der Verlängerung seiner Sinnesorgane (Atlas am Cern), darauf zeigen kann.
    Wir sehen also nicht einen Menschen, der denkt, sondern einen Menschen, der sich verhält und schließen auf das Denken. Auch hier könnte man hergehen und sagen: der Gedanke ist bloße Substantivierung des Denkens.

  50. @Stephan

    Carroll …

    …ist meiner Auffassung nach so zu verstehen, dass es im Alltagsleben nichts gibt, was nicht den bekannten physikalischen Gesetzmäßigkeiten gehorchen würde. Entsprechend heißt es auch in der Überschrift seines Artikels:

    The Laws Underlying The Physics of Everyday Life Are Completely Understood

    Es ist also mitnichten so, dass Carroll behaupten würde, man könne komplexe Alltagsphänomene auf dem Niveau atomarer Bausteine beschreiben.

    Wenn Du Carroll tatsächlich so interpretierst, wie Du hier vorgibst es zu tun (also nicht nur um dessen eher reduktionistische Auffassung lächerlich zu machen), dann dürftest Du mit diesem Verständnis seines Textes ziemlich alleine dastehen.

    (Hoffe ich zumindest…)

  51. @Balanus: Übersetzung

    Mit Verlaub, aber dein Vorschlag ergibt überhaupt gar keinen Sinn, davon abgesehen, dass sich das viel einfacher ausdrücken lassen würde mit Wendungen wie: “Our everyday phenomena are subject to… determined by… obey physical laws.”

    Deine “Übersetzung” passt aber gar nicht zu dem, was da steht, dass es sich nämlich auf die erkennenden Subjekte bezieht (“All we need to account for…”), was dann im zweiten Satz noch einmal aufgegriffen wird (“things we see in the world”).

    Kurzum, du sprichst (wieder einmal) balanesisch. Anstatt jetzt ein Ablenkungsmanöver zu starten und auf die Überschrift zu verweisen, übersetze doch einfach mal, was da steht. Im Übrigen verwenden wir Autoren permanent griffige Überschriften, kurz, bündig und aussagestark, und erklären dann erst im Text, wo wir mehr Platz haben, was wir genau damit meinen. Somit kann die Überschrift nur der Anfang, nicht aber das Ende der Diskussion sein.

    P.S. Und falls du es immer noch nicht verstanden hast, hier für dich aus dem Oxford Dictionary of English:

    account for
    1. give a satisfactory record of
    * provide or serve as a satisfactory explanation for: he was brought before the Board to account for his behavior.

  52. @Stephan Schleim

    “Nehmen wir einmal an, dass Hänsel Gretel hinterherspioniert, weil er denkt, dass sie (Gretel) ihn betrügt. ”
    Aber wäre das nicht eine kausale Erklärungshypothese, da man ansonsten die psychologischen Prozesse doch als Epiphänomen ansieht?
    Mir ist nicht ganz klar, was die Alternative zur Kausalität hier sein soll oder beschränken Sie hier den Kausalitätsbegriff auf physikalische Kausalität?

  53. @libertador: Kausalität

    Ich verstehe nicht, was die Diskussion über Kausalität hier bringen soll. Wir haben doch gerade einen Sinnzusammenhang hergestellt und erklärt, warum Hänsel Gretel hinterherspioniert. Was wollen Sie denn noch mehr? Und es ging ausdrücklich um Verhalten und Gedankenvorgänge – warum sollen dann ausgerechnet psychische Prozesse Epiphänomene sein?!

  54. @Stephan / Sean Carroll’s satisfactory explanations

    Der jüngst verstorbene Physik-Nobelpreisträger Philip W. Anderson war jemand von einem anderen intellektuellen Kaliber als Sean Carroll, und hier scheint mir ein Zitat aus Andersons Artikel `More is different‘ von 1972 in Science angebracht, wo er schreibt

    The arrogance of the particle physicist and his intensive research may be behind us (the discoverer of the positron said “the rest is chemistry”), but we have yet to recover from that of some molecular biologists, who seem determined to try to reduce everything about the human organism to “only” chemistry, from the common cold and all mental disease to the religious instinct.

    Was die Physiker angeht, mag er da etwas zu optimistisch gewesen sein.

    Man darf nicht vergessen, dass es eines der CMI Millenium Problems ist, dem Standardmodell der Teilchenphysik ein mathematisch rigoroses Fundament zu verpassen, und von `completely understood‘ kann beim gegenwärtigen Stand der Erkenntnis allein deswegen doch noch überhaupt keine Rede sein; vgl. dazu auch SdW 5/2009, 66-73.

  55. Der Mensch ist ein Organismus mit einfach mehr Nervenzellen, mehr Verknüpfungen zwischen diesen und mehr spezialisierten Hirnbereichen, die es ihm erlauben, mit sich selbst zu kommunizieren und sich solche Dinge auszudenken, wie sie in den Beiträgen hier zum Besten gegeben werden. Es gibt kein Leib-Seele-Problem, sondern nur verschiedene Blickwinkel durch die Brille diverser Wissenschaften. Das Stück, das wir in unserem subjektiven Bewußtseinstheater spielen, wird ständig durch die Umwelt “objektiviert”. Dies erweckt in uns den Anschein, als gäbe es eine transzendente Rationalität, eine für uns reservierte Sphäre.

  56. Dr. Wolfgang Stegemann,
    es gibt Menschen, die haben eine Gotteserfahrung gemacht und die fühlen sich eingebettet in eine freundliche Welt , die sie nicht nur materiell ansehen , sondern gefühlsmäßig, man kann sagen spirituell.

    Für solche Menschn ist Gott und Seele ganz natürlich, die gehören dazu. Gott und Seele verlieren dabei ihre Transzendenz.

    Menschen, die keine spirituellen Erfahrungen gemacht haben, die nehmen ihre Existenz nüchtern wahr, logisch, wissenschaftlich .

    Was lernen wir daraus ? Die Menschen sind verschieden und jeder sollte seinen Mitmenschen tolerieren.

  57. @Stephan // Carroll

    »Deine “Übersetzung” passt aber gar nicht zu dem, was da steht, … «

    Ich habe nichts „übersetzt“, sondern versucht Dir zu erklären, um was es Carroll in seinem kleinen Beitrag offenkundig geht, wie er, der Text, von einem verständigen Menschen zu verstehen ist. Aber Du verstehst ja wieder nur Balanesisch.

    Selbst @Chrys scheint Dir in diesem Fall nicht zu folgen (ob Carroll eine große Leuchte am Himmel der theoretischen Physik ist oder eher nicht, spielt zunächst mal keine Rolle).

    Ich verstehe auch gar nicht, was es bringen soll, Carroll zu unterstellen, er wolle Alltagsphänomene auf dem Niveau atomarer Teilchen beschreiben bzw. erklären. Das stärkt doch nicht die Argumentationskette, sondern schwächt sie nur.

    (Damit soll es nun aber gut sein, es gibt Wichtigeres zum Geist-Körper-Problem zu sagen)

  58. Anstatt “den Geist” auf körperliche Prozesse reduzieren zu wollen, könnte man es auch umdrehen: Körper ist Geist.

    Nicht schlecht, allerdings müsste der Körper zuerst (damit es so klappt : “Körper ist Geist”) in der Lage sein ‘Geist’ zu entwickeln, um dann mit den Begriffen ‘Geist’ und ‘Körper’ hantieren zu können, damit ihm ‘Geist’ zugesprochen werden kann bzw. damit er, wohlgemerkt : der Körper, dies selbst tun kann und auch tut.

    Dieses “Leib-Seele-Problem” hat den Schreiber dieser Zeilen nie sonderlich interessiert, weil es offensichtlich an Unterbestimmtheit leidet und insofern (zu) viel gequatscht wird.

    Dr. Webbaer mag die Idee, dass diese Welt gedacht (oder auch : betrieben) ist und insofern Welten Welten erzeugen können, die gedacht sind, sich sozusagen eine Menge von Gedachtem ergibt.

    Der Webbaer bearbeitet insofern nicht die Welt, sondern die Idee der Welt.

    Wer die Welt direkt bearbeiten will, hat womöglich (oder : sicherlich) nicht die “richtigen Handschuhe” an, ihm sind womöglich die “Ärmchen zu kurz” oder er verfügt, weniger poetisch formuliert, nicht über das geeignete Instrumentarium dafür.

    Wobei derart konstruktivistische Sicht auf die Welt sich umfänglich mit der modernen skeptizistischen Naturwissenschaftlichkeit deckt, kluge Physiklehrer betrachten die Welt, im Sinne der Humeschen Metaphysik, quasi als eine Art Black Box und klopfen sie bestmöglich ab, experimentieren, schaffen sogenannte Evidenz für ihre Theoretisierungen und sind dann auch fertig.

    Überlassen die kluge oder weniger kluge Metaphysik dann gerne anderen, machen “nur” das, was ihnen möglich erscheint.
    Unser Josef Honerkamp war bei den dankenswerterweise verfügbaren Scilogs.de so freundlich alles aufzuschreiben zur Funktionsweise der Erkenntnis.
    Das ‘Alles’ gerne in doppelte Anführungszeichen denken, danke.

    Mit freundlichen Grüßen und eine schöne Mittwoche noch!
    Dr. Webbaer

  59. @ Kommentatorenfreund H.Wied (so heißen Sie doch?!) und hierzu kurz :

    Menschen, die keine spirituellen Erfahrungen gemacht haben, die nehmen ihre Existenz nüchtern wahr, logisch, wissenschaftlich .

    Personen (vs. ‘Menschen’, kein Speziesismus bitte!), können die Welt im Metaphysischen nicht anders als “spirituell” bearbeiten, es liegt bekanntes Unbekanntes vor über das letztlich, sog. letzte Fragen meinend, nur spekuliert werden kann, logisch (und auch, haha, wissenschaftlich – moderne skeptizistische Naturwissenschaft und Spiritualität sind insofern keine Gegensatzpaare).
    Spiritualität ist sozusagen logisch.

    Naturwissenschaftlich Bemühte greifen insofern also nicht zu “Strohhalmen” hier gemeint religiöser Art, sondern sind sozusagen intrinsisch “spirituell”, nicht selten humanistisch und teilen viele Vorhalte, die von denjenigen vorgetragen werden, die wie hier gemeint religiöser Art sind.
    Gegensatzbildungen, wie von Ihnen vorgenommen, sind aus diesseitiger Sicht abzulehnen.
    Klar, der sogenannte Szientismus ist schlicht blöde.

    Mit freundlichen Grüßen und eine schöne Mittwoche noch
    Dr. Webbaer

  60. Dr. Webbaer,
    spirituelle Erfahrungen können auch künstlerischer oder auch esotherischer Natur sein. Auch Naturerfahrungen z.B. ein Wirbelsturm, ein Sonnenuntergang, auch eine unerwartete Rettung aus Lebensgefahr sind reale Faktoren. Humansisten sind stets willkommen !
    Die Gegensatzbildungen sind Absicht um hier Gegenrede zu erzeugen, was Sie ja dankenderweise auch getan haben.

  61. @ Kommentatorenfreund H.Wied (so heißen Sie doch?!) und hierzu kurz :

    Die Gegensatzbildungen sind Absicht um hier Gegenrede zu erzeugen, was Sie ja danken[swerter]weise auch getan haben.

    Jenau, Herr Wied, die Spiritualität entsteht als Gegenstand in der Naturlehre, wenn sog. letzte Fragen unbeantwortet zu bleiben haben, ihre Beantwortung oder zumindest ihre Bearbeitung aber nahe liegt, und im Geisteswissenschaftlichen, das leider teils verkommen ist, wird das Spirituelle auch teils hübsch bearbeitet, wenn sozusagen die Richtigen gelesen werden.
    Die Spiritualität, genau darum haben sich die Geisteswissenschaften zu kümmern, ist insofern ein ganz üblicher Gegenstand der Forschung.

    Dr. W hat sich mehrfach überlegt und ist auch eingeladen worden sozusagen zu den steifen Personen vom anderen Ufer zu gehen, die sind ja auch nicht blöde, haben ihre persönlichen Erfahrungen gemacht und sind nicht selten nett, abär im Sinne des Sapere-Aude wollte Dr. Webbaer nicht untreu werden, zumal auch sich ihm so auch nichts Neues, im Sinne von Anstrebenswerten, ergab.

    Es ist schon cool, wenn Erkenntnissubjekte (kein Speziesimus, bitte!) auf einer Kugel (streng genommen liegt ein sog. Ellipsoid vor) in Nähe eines sog. Sterns herumrollen und dieses Gesamtgebilde wiederum anderswie “herumrollt”.

    Und wenn dann sog. Szientisten oder Ökologisten (die gibt es ebenfalls, der Ökologist glaubt an einen intrinsischen Wert der Welt, vgl. mit dem Dominium Terrae, dem Sie womöglich, wie Dr. W, ebenfalls anhängen) irgendetwas vertellen.

    Spiritualität zu entwickeln ist des Erkenntnissubjekts vornehme (“vorzunehmende”) Aufgabe, es darf diesbezüglich ergebnisoffen gemeinsam erörtert werden, von Setzungen wie gemeint religiöser Art ist womöglich Abstand zu nehmen, inspirierend bleiben sie.

    Mit freundlichen Grüßen
    Dr. Webbaer (der anrät mal bei Josef Honerkamp reinzulesen, nur um mal die szientifische Methode zu verstehen zu lernen)

  62. @Balanus / Carroll & Co.

    Um nicht missverstanden zu werden, ich sehe Sean Carroll mit seinen Einlassungen schon eher als Geistesverwandten von Carl Anderson — jenem von Phil Anderson im Zitat namentlich nicht erwähnten Entdecker des Positrons, der die Parole “the rest is chemistry” ausgegeben hat. Ob die beiden Andersons familiär irgendwie verwandt waren, weiss ich nicht, aber sehr geistesverwandt waren sie augenscheinlich nicht. Im verlinkten Nachruf auf Phil Anderson heisst es noch wörtlich:

    In his famous 1972 article “More is Different,” he emphasized that complex systems may exhibit behavior that cannot be understood only in terms of laws governing their microscopic constituents, but may require hierarchical levels of science each with their own fundamental principles.

    Und dies liest sich doch ganz anders an als das, was Carroll da verkündet.

  63. @Chrys, Balanus: Carroll

    Vielen Dank für die Literaturhinweise, Chrys!

    Über Carrolls Arbeiten auf dem Gebiet der Physik kann ich nichts sagen. Das ist nicht mein Fachgebiet…

    …aber wenn ich sehe, wie er über Wissenschaft kommuniziert, dann bekomme ich schon den Eindruck: Das ist einer von denen, die nette Märchen verkaufen. Man sehe sich hier mal die Diskussion mit dem Buddhisten Alan Wallace über die Natur der Realität an. Ich habe nicht den Eindruck gewonnen, dass Carroll in der Diskussion viel zu bieten hat.

  64. @H.Wied 02.06. 17:25 Körper ist Geist

    „So langsam dämmert es den Naturwissenschaften, die Atome bestehen aus noch kleineren Teilchen. Die noch kleineren Teilchen sind nur über die Energie erklärbar. Und die Energie als Begriff ist ein Konstrukt des Geistes, was sonst.“

    Das sowieso, aber vielleicht noch etwas mehr: Wenn es ein kosmisches Bewusstsein gibt, dann ist dieses mutmaßlich auch der Weltbetreiber, der die Naturgesetze nicht nur erfunden hat, sondern sie auch in der Wirklichkeit realisiert. Man kann beobachten, dass die Feineinstellung der Naturgesetze sehr, sehr speziell sind, und ein Universum, das Sterne, Planeten und Lebewesen hervorbringen kann, eigentlich sehr unwahrscheinlich ist. Zahlreiche Naturkonstanten haben genau die richtigen Werte, die letztlich für das biologische Leben notwendig sind.

    Wenn sich dieses kosmische Bewusstsein dann an den persönlichen psychischen Prozessen von einzelnen Lebewesen direkt beteiligt, dann ist das nur folgerichtig. Offenbar ist das ganze Universum genau dafür konstruiert.

    Es ergibt sich daraus dann noch eine Erklärung dafür, wieso vieles in der Physik überhaupt mathematischen Regeln folgt: das führt zu einer hohen Ordnung aller Dinge in diesem Kosmos, und die ist für die Bewohnbarkeit mancher Planeten durch biologische Lebewesen wohl Voraussetzung. Die Mischung aus vorhersehbaren Naturgesetzen und Quantenphysikalischen Zufallsprozessen machen es erst möglich, dass sich das kosmische Bewusstsein am lokalem menschlichen Bewusstsein sinnvoll beteiligen kann.

    Mit weniger Ordnung würde das zu keinen brauchbaren Lebensräumen führen, die aber für die kleinen Lebensgeschichten der Lebewesen nötig sind, um die es hier anscheinend geht. Ohne ordentliche mathematische Naturgesetze wäre unser Lebensraum für Lebewesen mit einer eigenen inneren Lebenswelt nicht zu gebrauchen. Aber ohne eigenen inneren Spielraum wäre unsere Existenz selbst wiederum sinnlos.

  65. Wichtig bleibt die sog. Humesche Metaphysik zu verstehen, die gerne auch angenommen werden kann, die den Empirismus erklärt.
    Es ist insofern dumm [1] oder doof [1] (physikalische) Theorie, die auf (noch) empirisch adäquater Datenlage beruht, irgendwie als wahr zu bezeichnen.
    Die sogenannte Realität kann insofern nichts anderes als eine Hilfsgröße sein, derer sich die hier gemeinte Primatenschaft bedient.
    Wahrheit gibt es in der Tautologie und zwar genau dann, wenn diese auf einer Axiomatik beruhend den Wahrheitswert kennt und pflegt.
    Nicht in der Natur also.

    Mit freundlichen Grüßen und eine schöne Mittwoche noch
    Dr. Webbaer

    [1]
    Die Dummheit meint die Tumbheit und die (vergleichsweise) Unfähigkeit sich sprachlich auszudrücken (Merkel hier mal böse und beispielhaft genannt, diese Dame kann ja nicht (frei) reden) und die Doofheit die Harthörigkeit, viele sind ja nicht in der Lage, durchaus dem Shannon-Weaver-Modell folgend, erhaltende Nachricht hinreichend zu abstrahieren, zu dekodieren oder zu verstehen.

  66. Bonuskommentar @ Herr Dr. Stephan Schleim und hierzu :

    Man sehe sich hier mal die Diskussion mit dem Buddhisten Alan Wallace über die Natur der Realität an. Ich habe nicht den Eindruck gewonnen, dass Carroll in der Diskussion viel zu bieten hat.

    Dr. W hat momentan ein wenig Zeit, abär nicht so-o viel – gerne schriftlich zur Schau bringen, wenn etwas besonders und der hiesigen kleinen Erörterung beihelfend ist oder sein könnte.

    Nichts spricht gegen die gute alte Zitation.
    “Time saved …. is money earned!”

    Mit freundlichen Grüßen
    Dr. Webbaer

    PS:
    So schaut’s besser aus.

  67. Oh, sorry,
    Natürlich war der Aluhutträger auf keinen in diesem Forum gemünzt, da ging es mir eher um die derzeitigen Proteste gegen die Corona- Maßnahmen. Das das missverstanden wurde, geht klar auf meine Kappe.
    Nun, das Corona-Virus ist eine Ursache, die Wirkung ist im Moment zu bestaunen.
    Klar dürfte auch sein, dass sich diejenigen, die auf den Rat von Wissenschaftlern hören, besser durch die Kriese kommen (Deutschland – Brasilien)
    Hier eine meiner Hausaufgaben:
    Das Kausalitätsprinzip hat durch die Quantenmechanik durchaus nicht an Bedeutung verloren, allerdings wird es eher so ausgedrückt: Information kann nicht verloren gehen. Das ist z.B. ein großes Problem bezüglich Schwarzer Löcher, wo etwas hineinfallen kann, aber auch etwas daraus entkommen kann (Hawking-Strahlung) und dabei offensichtlich die Kausalkette reißt. Würden auch noch mentale Prozesse „nicht-kausal“ sein, wäre schlicht die Physik am Ende.
    In diesem Zusammenhang wäre ich neugierig auf auch nur einen wissenschaftlichen Beweis: „Es gibt genuin mentale/psychische Prozesse und diese sind nicht-physikalische Prozesse.“ Mentale Prozesse, also das Denken betreffende Prozesse, da sind sich die Naturwissenschaftler einig, spielen sich im Gehirn als physikalische Prozesse ab. Kann man schon in Bienengehirnen Neuron für Neuron beobachten.

  68. Not that bad although :

    -> https://www.youtube.com/watch?v=Bt4Xx3vX3Lg

    “Satan” ist interessant, er dient dem verständigen und allgemein im sozialen, im aufklärerisch-sozialen untergewegs seienden Subjekt als Verständigungdgröße.
    Dr. W mag “Satan” als Vrevollkommnung des Bösen, das nicht nur gelegentlich antithetisch heranzuziehen ist, der Song der “Stones” verfügt über Schönheit, Dr. W hat auch die begeleitende Hässlichkeit in “One plus One” zK nehmen können, seinerzeit und konterkarierend, die “Frösche” halt,
    reinzuschauen lohnt sich dennoch.

  69. Upps,
    als Verständigung[s]größe (war natürlich gemeint)

    ‘Verständigungsgröße’ gibt es immerhin bei “Google” etwa ein Dutzend fach, Dr, W mag es Sprachen zu verreichern.

  70. *
    Dr. W mag “Satan” als V[er]vollkommnung des Bösen

    (Sicherlich ist Dr. W nur satanisch im Sinne des Verstehens des Bösen interessiert, das Böse bleibt (auch für ihn) effing interessant – es geht aktuelle gegen das Böse, gegen die Schweine, es darf wehrhaft geblieben werden.)

  71. @Webbär: Carroll vs Wallace

    Sie sind mir ja einer, wollen sich damit nicht auseinandersetzen und machen mir daraus einen Vorwurf.

    Im Übrigens wäre es schön, wenn Sie sich mit Ihren Kommentaren etwas mäßigen un aufs Thema dieses Beitrags beschränken würden. Danke.

  72. @Manzel: Information

    Mir erschließt sich erst einmal nicht, was Information mit Kausalität zu tun hat. Wie ich, glaube ich, im Artikel erwähnte, wurden über “Kausalität” ganze Bücher geschrieben. Da sollte man schon etwas präziser formulieren.

    Und zur mentalen Verursachung… Wenn ich schreibe, dass ein psychischer Prozess ein solcher ist, weil er zutreffenderweise unter eine psychologische Beschreibung fällt, wo wird da jetzt genau ein Kausalprinzip o.ä. verletzt?

    Um zum Beispiel zurückzukehren: Warum soll die Physik am Ende sein, wenn Hänsel Gretel hinterherspioniert, weil er denkt, sie würde fremdgehen?! Meines Wissens gibt es solche Prozesse wirklich und stört das die Physik kein Iota.

  73. Die Reduktion ist selbst ein mentaler Prozess und sogar einer der elementarsten und einer der wichtigsten für die Erkenntnisgewinnung. Denn ohne Reduktion kommen wir nie zur Abgrenzung, die nötig ist um einen Begriff zu schaffen. Reduktionismus ist dann die Extremposition: sie hält die geschaffenen Begriffe oder gefundenen Teile wichtiger als das Ganze oder sie will das Ganze durch simple Komposition der Teile erklären.
    Aber nicht nur Reduktion ist ein mentaler Prozess, auch das was wir Geist, Körper oder die Welt nennen entspringen mentalen Prozessen. Allerdings existiert ein bestimmter Körper, ein bestimmter Geist (beispielsweise der Geist/Körper mit dem ich mich vor kurzem unterhalten habe) auch als reales Ding. Es gibt aber auch Dinge, die nicht real sind, sondern die nur existieren, wenn ein Geist sich damit beschäftigt. Das ist beispielsweise die Mathematik, das sind die Objekte der Mathematik. Die Zahl Pi beispielsweise ist kein reales Ding. Ohne Mathematiker gibt es sie nicht.

  74. Martin Holzherr,
    Bezug auf :
    “Es gibt aber auch Dinge, die nicht real sind, sondern die nur existieren, wenn ein Geist sich damit beschäftigt. Das ist beispielsweise die Mathematik, das sind die Objekte der Mathematik. Die Zahl Pi beispielsweise ist kein reales Ding. Ohne Mathematiker gibt es sie nicht.”
    Mit dieser Ansicht sind wir wieder bei Aristoteles, im Gegensatz zur platonischen Ideenlehre.
    Die Gesetze der Logik , die Gesetze der Mathematik gelten immer, auch wenn keine Menschen mehr unseren Planeten bewohnen. Die Gesetze der Logik sind auch die Gesetze der Physik , der Mathematik, der Natur.. Und die Natur wird auch weiter diesen Gesetzen gehorchen, wenn es keinen Geist mehr gibt, der sie erkennen kann.
    Die Naturgesetze sind nicht der Zeit und dem Raum unterworfen, sie gelten immer, auch wenn es die Zeit und den Raum nicht mehr gibt.

  75. Immer wieder gibt es Formulierungen wie: Körper ist Geist, oder auch umgekehrt.

    Ich würde davon ausgehen, Körper als auch Geist gehören verschiedenen Kategorien an. Allein weil „Körper“ z.B. direkt „angreifbar“ und „sichtbar“ sind. Das ist der Geist nicht.

    Daher können „Körper“ nicht so ohne weiteres gleich „Geist“ sein.

    Allenfalls wäre denkbar, dass sozusagen (auch wechselwirkende) geistige (von Information) gesteuerte Prozesse, im Sinne z.B. von der „alleruntersten“ Teilchen Ebene im Sinne von Carroll ausgehend, letztlich „Körper“ entstehen, was sich ja vermutlich in der Evolution auch tatsächlich so entwickelt haben dürfte.

    Dies würde letztlich bedeuten, dass Carrolls Reduktionismus im Prinzip stimmig ist, wenn er in das Konzept der Informatiker aus Prozessor(chen) (Teilchen), (physikalische, chemische….) Prozesse, (steuernde und beschreibende) Information eingebunden ist.

    „Geist“ ist zwar noch immer nicht gleich „Körper“, kann aber zur „Entstehung“ eines „Körpers“ beitragen.

    Mich würden einleuchtende kreative Gegenargumente interessieren.

  76. @ Elektroniker: Gegenargument. Die Vorstellung einer der Materie innewohnenden Information ist ein beliebtes Denkspiel, das auf einen Schöpfer oder ein ordnendes Prinzip hinweist. Die Frage ist nur, was wollen wir unter einer solchen Information verstehen? Etwa, daß am 28.3. um 12.00 Uhr Frau Müller mit dem Fahrrad eine Katze überfährt? Oder ganz allgemein und damit nichtssagend, daß im Universum von uns bezeichnete verschiedene Kräfte existieren? Wir neigen zur Kausalitätsumkehr und glauben, daß die Evolution (des Universums) zielgerichtet zum Menschen führt. Sie hätte genausogut zu nichts Lebendigem führen können. Der Gedanke einer immateriellen Information in der Materie ist letztlich Religionsersatz.

  77. @Chrys // Carroll vs. Philip/Carl Anderson

    Wer da mit wem geistesverwandt ist, sei mal dahingestellt.

    Mit dem Zitat aus dem Nachruf auf Philip Anderson soll wohl suggeriert werden, dass Carroll der Auffassung ist, dass das Verhalten von komplexen Systemen mit den Mitteln der Atomphysik beschrieben werden kann. Zum Beispiel. Oder gar quantenmechanisch, wer weiß.

    Soweit ich sehe, gibt es dafür aber nicht den kleinsten Beleg. Oder genügt hierfür etwa sein kurzer Blog-Beitrag, weil er es versäumt hat, dort die hierarchischen Wissenschaftsniveaus zu thematisieren?

    » Und dies [was P. Anderson 1972 in Science publiziert hat] liest sich doch ganz anders an als das, was Carroll da verkündet. «

    Ach was, erstens geht es in Carrolls Blog-Beitrag um etwas völlig anderes als in Andersons Science-Artikel, und zweitens dürfte Carroll mit dem, was Anderson 1972 geschrieben hat, sehr wohl einverstanden sein. Zum Beispiel mit dem hier:

    I think it will be accepted that all ordinary matter obeys simple electrodynamics and quantum theory, and that really covers most of what I shall discuss. (As I said, we must all start with reductionism, which I fully accept.)

    Wenn man Carrolls Aussagen tatsächlich widersprechen oder ad absurdum führen will, dann müsste man eigentlich behaupten, dass die fundamentalen Gesetze der Physik in komplexen Systemen keine Rolle (mehr) spielen.

  78. @Balanus: Pointe

    Du verpasst die Pointe, wenn du Anderson nur bis dahin liest, dass man mit einem Reduktionismus anfangen sollte. Womit soll man denn aufhören?

    Also auf mich wirkt das nicht wie gute wissenschaftliche Arbeit, bis zum ersten Satz zu lesen, der die eigene Sicht bestätigt (im Zweifelsfall nur die Überschrift), und dann den ganzen Rest zu ignorieren.

  79. @ Wolfgang Stegemann 03.06.2020, 20:09 Uhr

    Ich würde als „innewohnende Information“ z.B. den aus dem normalen Chemieunterricht stammenden, für chemische Prozesse wichtigen Fakt betrachten, dass die Zahl (Information) der in der äußersten „Elektronenhülle“ befindlichen „Elektronen“ von besonderer Relevanz für das chemische Veralten bei chemischen Prozessen ist.

    Ist die Schale vollständig (weitere Information), gibt es keine chemischen Prozesse (Edelgase, Gold…).

    Gesetzmäßigkeiten (Information) bestimmen, steuern z.B. das Verhalten beim Aufeinandertreffen von chemischen Komponenten. Vergleichbar damit, wie z.B. digitale Information gemäß der technischen Konzepte, das Verhalten eines Computers, Roboters steuert.

    Es sind naturgesetzliche Informationen. Einerseits „Prozess steuernde“, andererseits „beschreibende“ Informationen, die das Verhalten bei den Prozessen „erzwingen“, wie bei von Menschen geplanten technischen Prozessen.

    In diesem Sinne gibt es sehr viele z.B. Naturkonstante und mathematisch beschreibbare Zusammenhänge die das reale Geschehen z.B. in der Physik, Chemie …. abbilden können und natürlich auch real steuern.

    Diese „Objekte“ der Mathematik sind zweifellos auch Information. Diese Objekte „tragen“ von Beobachtern festgelegte „Bezeichner“.

    Mich persönlich stört es nicht, dass man dem Gesamtsystem aus Naturgesetzen, Konstanten, Prozessen, Informationen …. wie man vermutet hat, obwohl man es (wegen der offensichtlichen Transzendenz) nicht vollständig verstanden hat, den Bezeichner „Gott“ zugewiesen hat.

    Es wird stets aus dem „Pool aus Naturgesetzen geschöpft“, ob von Menschen wenn sie ein neues physikalisches/chemisches Verfahren aushecken, weil sie Geld verdienen wollen, oder wenn Google Computer, sagen wir einmal 20 Billionen neue Stellen von Pi berechnen.

    Aber auch, wenn mittels der Naturgesetze selbst, durch physikalische, chemische, biologische automatische ohne Zutun eines Menschen „schöpfende Maschinen“ am Werk waren, letztlich die „Welt“ entstanden ist, so bezeichnen diesen Sachverhalt manche Menschen eben mit „Gott“. Es ist nun einmal so.

    Für manche Theologen ist „Gott“ eben genau das wie es wirklich ist, praktisch eben eine „immer richtige“ Tautologie.

    Für mich als ehemaligen Elektroniker war, Information immer genau das Immaterielle (Sprache, abstrakte Daten….) die es mittels Materie zu übertragen, oder zu verarbeiten galt. Über Funk z.B. von Amerika nach Europa, womöglich über Satelliten im Weltraum, wurde z.B. ausschließlich Information übertragen, nicht das „kleinste Stäubchen“ Materie.

    Ob der Mensch „zielgerichtet“ entstand, hängt davon ab, ob die „Zufallsgeneratoren“ die die biologischen Variablen generiert haben, „systematisch und hoch exponentiell gearbeitet“ haben, also irgendwann zwingend die Menschen ermöglichenden „Genvariablen“ entstehen mussten.

  80. @H.Wied (Zitat):

    Die Gesetze der Logik , die Gesetze der Mathematik gelten immer, auch wenn keine Menschen mehr unseren Planeten bewohnen. Die Gesetze der Logik sind auch die Gesetze der Physik , der Mathematik, der Natur..

    Klar, aber Naturgesetze und die dazugehörige Mathematik sind Entdeckungen/Erfindungen des Menschen. Naturgesetze und Mathematik dienen der Beschreibung der Natur und für diese Beschreibung gibt es mehrere Beschreibungsmöglichkeiten, mehrere und verschiedene mathematische Ansätze dafür. Es sind Menschen, die diese Beschreibungen geschaffen haben. Die Mathematik und die Naturgesetze haben keine Eigenexistenz, genauso wie auch Information keine Eigenexistenz hat. Nur informationsverarbeitende Maschinen wie der Mensch eben können überhaupt Theorien über die Welt aufstellen. Ohne Menschen gibt es diese Theorien schlicht und einfach nicht.

  81. @Elektroniker:

    Immer wieder gibt es Formulierungen wie: Körper ist Geist, oder auch umgekehrt. Ich würde davon ausgehen, Körper als auch Geist gehören verschiedenen Kategorien an. Allein weil „Körper“ z.B. direkt „angreifbar“ und „sichtbar“ sind. Das ist der Geist nicht.

    Wenn sie mit Geist den Geist eines Menschen meinen, dann ist dieser Geist angreifbar, denn ohne Körper gibt es diesen Geist nicht. Wenn ein Mensch stirbt, stirbt auch sein Geist, denn der Geist eines Menschen ist im Körper beheimatet.
    Jeder menschliche Körper wiederum ist ein reales Ding unserer Welt. Als reales Ding unterliegt er den gleichen physikalischen Gesetzen wie alle anderen realen Dinge. Indirekt gilt das auch für den menschlichen Geist, denn ohne Körper gibt es ihn nicht. Wenn es etwas spezielles am menschlichen Geist gibt, dann das, dass er zu Erkenntnissen fähig ist. Ohne diese Erkenntnisse wüssten wir nicht einmal dass wir einen Körper oder auch einen Geist besitzen.

  82. Martin Holzherr,
    Frauen lieben Philosophie nicht besonders, weil sie mit den konkreten menschlichen Problemen hier und heute wenig zu tun haben.
    Das Leib-Seele -Problem stellt sich einer übergewichtigen Frau, die vor einem Café steht und die ein Stück Schwarzwälder Sahnetorte anlacht.
    Hört sie auf ihren Körper, dann lässt sie sich verführen. Hört sie auf ihren Geist (Verstand ist jetzt gemeint), dann verzichtet sie auf die Torte.
    Damit will ich zeigen, dass es tatsächlich diese Spaltung in Körper und Geist (das Ich ist gemeint) gibt.
    Der Sportler muss seinen inneren Schweinhund überwinden, wenn er vor sich selber bestehen will.
    Der Alkoholiker, der trocken ist, der braucht viel Willenskraft um beständig zu bleiben.
    Der Schüler, der eine gute Note will, der muss auf sein Fußballspiel verzichten.
    Die Spannung zwischen Körper und Geist (ganz allgemein) ist immer da, und es kommt auf die Harmonie zwischen beiden an, wenn man letzlich keine Depression bekommen will. Der Körper verzeiht keine Fehler, der Geist verzeiht auch keine Fehler.
    Deswegen gibt es die griechische Tragödie, genauso wie die Liebesromane.

  83. @ H.Wied

    “Damit will ich zeigen, dass es tatsächlich diese Spaltung in Körper und Geist (das Ich ist gemeint) gibt.”

    Das ließe sich schnell entkräften. Das Hirn ist zur Selbstreflexion befähigt. Da braucht es keine zwei Substanzen.

  84. @Wied

    „die Abwesenheit der Frauen hier ist kein Klischee“

    Könnte das nicht auch mit Ihren Beiträgen zusammenhängen?

  85. @Stephan // Pointen

    »Du verpasst die Pointe, wenn du Anderson nur bis dahin liest, dass man mit einem Reduktionismus anfangen sollte.«

    Welche Pointe?

    Die Pointe bei diesem Text von P.W. Anderson ist doch, aus meiner Sicht, dass er nicht als Zeuge der Anklage (gegen Carroll bzw. den Reduktionismus) taugt.

    Hier ein paar Zitate, die vor (!) der von mir oben zitierten Textpassage stehen:

    The reductionist hypothesis may still be a topic for controversy among philosophers, but among the great majority of active scientists I think it is accepted without question The workings of our minds and bodies, and of all the animate or inanimate matter of which we have any detailed knowledges are assumed to be controlled by the same set of fundamental laws, which except under certain extreme conditions we feel we know pretty well.

    Und auch das hier:

    The main fallacy in this kind of thinking [the only scientists who are studying anything really fundamental are those who are working on those laws] is that the reductionist hypothesis does not by any means imply a “constructionist” one: The ability to reduce everything to simple fundamental laws does not imply the ability to start from those laws and reconstruct the universe.

    Es bleibt also dabei, Carroll behauptet m. E. in seinem Blog-Beitrag nichts, woraus zu folgern wäre, dass »mit einem Schlag nicht nur die gesamten Geistes-, Kultur- und Sozialwissenschaften sowie die Psychologie, sondern gar Neurowissenschaften, Biologie und Chemie überflüssig« wären.

    Falls ich bei P.W. Anderson (oder auch bei Carroll) etwas Entscheidendes übersehen haben sollte, bitte ich um einen kleinen Hinweis—von wem auch immer.

  86. Mona,
    die Abwesenheit der Frauen kommt daher, dass die Blogmaster Männer sind. Die nüchterne Überschrift, “das kleine Einmaleins”
    das ist schon ein Gefühlskiller.
    Hätte er geschrieben; “Strapaziert die Orangenhaut mein Gefühlsleben”, dann wären viel mehr Beiträge gekommen.

    Männer sind nicht geeignet für die Philosophie von Leib und Seele. . Das Ergebnis sind nur viele Worte.

  87. @ Martin Holzherr 04.06.2020, 02:12 Uhr

    Es kommt auf die genaue „Deklaration“ von „Geist“ an.

    Gewisse Komponenten, von z.B. „Goethes Geist“, haben Goethe überlebt, obwohl er selber schon lange tot ist. Es ist sozusagen die besondere Anordnung der Buchstaben in Goethes Werken. Das Papier und die Druckerschwärze sind zwar vorhanden, aber es kommt normalerweise allein auf die „Information“ an, die mittels der üblichen sprachlichen Konzepte „abgebildet“ wird.

    Bemerkenswert ist meistens, dass die Information unabhängig von der zugehörigen Hardware z.B. transportiert, verarbeitet, umgewandelt weitergegeben werden kann.

    Beim Coronavirus kommt es ebenfalls auf die Anordnung der RNA Codeelemente an, die ist bemerkenswert und kann gefährlich sein. Die gleichen Codeelemente, aber etwas anders angeordnet, eine andere Information und der Virus wäre z.B. völlig harmlos. Auf die Information kommt es an.

    In meinem früheren Berufsleben ist es sehr oft darauf angekommen, die Information völlig vom System oder Datenträger getrennt zu sehen und in einem völlig anderen System zu verarbeiten, übertragen….

    Selbst wenn dass System stirbt oder verschrottet wird, Information kann z.B.im Netzwerk, weiter existieren. Egal ob Goethes Werke, seine Gene oder Computersoftware …

    Information ist flüchtig. Die „virale Information“, abgebildet auf RNA, nutzt Fettmoleküle sozusagen als Taxi, „reist“ von Mensch zu Mensch und macht ihren „Job“. Sie bleibt nicht auf einem Menschen „sitzen“ und stirbt eines Tages mit ihm. Information hat sich sozusagen emanzipiert, ist völlig unabhängig bis sie ein neues Opfer findet, setzt sich danach „ins Taxi“ und zieht einfach weiter, zum nächsten Job ….

  88. @Wied: Frauen und Philosophie

    Ihre Aussagen über Frauen und Philosophie hören sich für mich sehr stereotypisch und teils schon beleidigend an. Ich möchte Sie bitten, Ihren Standpunkt noch einmal zu überbedenken und vor allem hier im Diskussionsforum gemäßigter auszudrücken.

    Ich kann hier anführen, dass, menschlich gesehen, der Hauptgrund dafür, dass ich (als Student) in der Philosophie geblieben bin, die Lehre von Professorin Elke Brendel (inzwischen an der Universität Bonn) war. Die ist nicht nur Logikerin und Erkenntnistheoretikerin, sondern auch diplomierte Mathematikerin.

    Philosophie an den Universitäten ist v.a. in den höheren Etagen noch sehr männlich, ja, im Sinne von: Die allermeisten Philosophieprofessoren sind Männer. Es gibt aber auch Männer in Gebieten, die Sie vielleicht eher als weiblich ansehen würden (z.B. Ethik oder Lebensphilosophie) und umgekehrt Frauen in Gebieten, die Sie vielleicht eher als männlich ansehen würden (die genannten Beispiele Logik und Erkenntnistheorie).

    Falls Sie mal ansprechende, allgemeinverständliche Texte von einer Philosophin lesen wollen, dann empfehle ich Ihnen die (leider inzwischen eingestellte) Kolumne von Frau Dr. Stefanie Voigt.

  89. @all: Frauen/Männer

    Bei Telepolis, wo ich schreibe, bin ich mir ziemlich sicher, dass dort vor allem Männer in der zweiten Lebenshälfte lesen; hier auf Spektrum.de ist es vielleicht etwas ausgewogener, rechne ich aber auch noch mit so einem Unterschied.

    Für ernst gemeinte Vorschläge, die Texte für andere Gruppen zugänglicher zu machen, bin ich immer dankbar.

    P.S. Als ich meine frühere Kollegin Susanne Grassman zu einem Gastbeitrag über das Lernen einlud, erlaubten sich meinem Eindruck nach einige (männliche) Diskutanten grenzwertige Kritikpunkte, die sie sich bei mir wahrscheinlich nicht getraut hätten.

    Über solche Unterschiede sollte man mal nachdenken. Das lässt diese Männer jedenfalls in keinem guten Licht dastehen.

  90. Stefan Schleim,
    Ihre Kritik ist berechtigt, der Ton macht die Musik.
    Ich werde mich bemühen, feinfühlender zu sein.

    Zurück zum Thema.
    Beim Thema Körper – Seele geht es praktisch gesehen um das Thema Pflicht und Neigung, Gefühl und Verstand.
    Gehorchen wir unserem Körper oder gehorchen wir unserem Willen.
    Darum dreht es sich in der abendländischen Kultur und nicht nur dort.

    Sigmund Freud hat mit seinem Modell vom ES – Ich – ÜBERICH gute Vorarbeit geleistet. Man weiß, wie sich mit Arzneimittel das Denken und Fühlen beeinflussen lässt. Man weiß , wo in unserem Gehirn die Aktivitäten ablaufen, aber man weiß nicht, in welcher Form die Gedanken abgespeichert sind. Ja, man weiß nicht, was ein Gedanke ist und wie der physikalisch dargestellt werden kann. Von den bit der Informatiker mal abgesehen.

    Mir kam es darauf an, dass festgefügte Weltbild der Naturwissenschaftler, die keine Seele gelten lassen, wieder aufzurollen und dem Idealismus wieder als feste Größe ins Gedächtnis zu rufen.

  91. @Wied: Körper/Gefühl vs. Geist/Verstand

    Diese dualistische Betrachtungsweise überzeugt micht nicht. Wenn, wie ich schlussfolgere, Körper Geist ist, dann ist Körper sowohl Gefühl als auch Verstand.

    Freud war und ist als historische Persönlichkeit natürlich sehr relevant, doch bei der Unterscheidung von Es, Ich und Über-Ich sollte man an das denken, was ich im Artikel über Reifikation schrieb.

  92. Stephan Schleim,
    “dann ist Körper sowohl Gefühl als auch Verstand”
    das ist der Kernpunkt meiner Aussage.
    Die dualistische Sichtweise berücksichtigt nicht, dass Körper Sitz von Gefühl und Verstand ist , aber umgekehrt nicht, Gefühl und Verstand sind aber originär nicht nur eine Eigenschaft des Körpers sondern transzendent.
    Die Reifikation, die Vedinglichung von “Geistigem” ist eine sprachtliche Methode Dinge anschaulich zu machen, die sonst nicht verstanden werden.
    Sie ist die Ursache von Missverständnissen.

  93. Zu H. Wied:
    “dann ist Körper sowohl Gefühl als auch Verstand”
    Und wer fühlt da den Körper und den Verstand ? Mir fehlt hier die Definition was GEIST ist ? Der GEIST erkennt den Verstand und den Körper, der Geist erkennt sich SELBST. Er erkennt, dass dieses SELBST vom Zeitgeist geformt wird, von Religionen, Ideologien, von Manipulationen und Manipulierten. Gefühlen. All das ist ER nicht, sondern er nimmt das alles nur zur Kenntnis.(Prinzip der Leerheit im Buddhismus) Der Verstand ist nur ein Produkt de Egos. Wer ist es also in uns, der da den Körper und den Verstand erkennt ?

  94. @Querdenker: Welcher Geist?

    Nervenzellenreize laufen im Zentralnervensystem zusammen und lassen dort Wahrnehmungen entstehen. (Trotzdem sind diese Nervenreaktionen aufgrund der Qualitäten grundsätzlich nicht auf die neurowissenschaftliche/physikalische Ebene reduzierbar.)

    Wo ist das Problem? Wo fehlt hier ein Geist in der Gleichung?

  95. @Querdenker 04.06. 15:27

    „Wer ist es also in uns, der da den Körper und den Verstand erkennt?“

    Na wenn unsere Innenwelt eine Synthese unseres lokalen Gehirns mit dem kosmischen Bewusstsein ist, dann ist es der kosmische Anteil in uns.

    Dieser erkennt wohl nicht nur, sondern stellt überhaupt erst den Geistesraum zur Verfügung, in dem die konkreten Inhalte unseres Bewusstsein sozusagen aktuell eingetragen werden. Dazu gehört neben den Gedanken, dem Verstand und dem Gefühl vor allem die kompletten Wahrnehmungen über die momentane Außenwelt, in der wir uns gerade befinden.

    Falls dem so ist, müssen die Hirnforscher gar nicht entschlüsseln, wie die grundlegende Bewusstseinsfunktion mit den Nervenzellen konkret funktioniert, weil hier der kosmische Geist seine Unterstützung zur Verfügung stellt, und wesentliche Funktionen der lebenden Psyche übernimmt.

  96. Querdenker,
    die Definition von Geist fehlt …….
    Gehen Sie zu den Naturwissenschaften. Da gibt es Theorien mit festgelegten Definitionen. Mit einer Ausnahme, der Begriff der Energie. Das ist nur ein quantitativer Begriff, weil es zu seinem Wesen gehört, Unvergleichliches quantitiv vergleichbar zu machen.
    Wärmeenergie kann so mit Bewegungsenergie sogar mit Gravitationsenergie quantitativ verglichen werden.
    Der “Geist” der Geisteswissenschaften erfüllt die gleiche Aufgabe. Dieser Containerbegriff kann die Gedanken aufnehmen, den Sinn der Gedanken, Schriftenrollen der Vergangenheit, Glaubenswahrheiten der Religionen sogar die verborgenen Absichten eines Lügners.

    Deswegen lässt sich Geist nur definieren, wenn man weiß in welchem Zusammenhang er benützt werden soll.
    Der Geist, der zu Pfingsten über die Jünger gekommen ist, der meint eine göttliche Erscheinung und eine plötzliche Erkenntnis.

  97. @Balanus / 04.06.2020, 11:08 Uhr

    Es stimmt doch einfach nicht, dass alles, was wir im Alltag sehen, vollständig zu verstehen oder zu erklären ist nur durch ein paar Standard-Partikel, die sich nach den Regeln von Quantenmechanik plus Relativität verhalten.

    Im Alltag trifft man allenthalben auf thermophysikalische Phänomene, die sich mit den Regeln der Thermodynamik mehr oder weinger verstehen lassen. Die Hauptsätze der TD sind aber keineswegs Korollare aus QM oder RT.

    Im Alltag sehen wir, wie ein Küken aus seinem Ei schlüpft, aber niemals schlüpft es wieder hinein und verschliesst hinter sich die Eierschale. Das Schlüpfen ist irreversibel, und das ist eine neue Qualität dieses Prozesses, die sich nicht aus den Postulaten von QM oder RT verstehen und erklären lässt. Einverstanden?

  98. Das „Leib-Seele-Problem“ sollte, anders als in der Formulierung als „Körper-Geist-Problem“, eindeutig „dualistisch“ gesehen werden.

    Die Formulierung als „Körper-Geist-Problem“ macht eigentlich das „Elend“ der Philosophie, was die „Deklaration“ (im Sinne der Informatik) von Wortbedeutungen betrifft sichtbar. Die Philosophie ist oft gezwungen Wortbedeutungen festzulegen, obwohl die zur Bezeichnung anstehenden Sachverhalte womöglich kaum verstanden werden. (Und wenn es keine realistischen Bezeichnungen gibt, kann man die Sachverhalte auch nicht wirklich verstehen).

    Bei der Formulierung als „Leib-Seele-Problem“ verhält es sich anders, weil „Seele“ Informationscharakter hat und sozusagen im Sinne der Informatik korrekt „deklariert“ (mit einer halbwegs realistischen Bedeutung versehen) ist. Für „Geist“ kann man dies nicht behaupten.

    „Seele“ bezieht sich im Sinne der Theologie genau auf die Information, die eben nach dem Ende des „Leibes“ weiter existieren, jederzeit zumindest teilweise aktiviert werden kann. Dies gibt es eindeutig, Gene z.B. können dies, weil die vom Vater stammende Information z.B. auch noch nach seinem Tod aktiv bleiben kann und die „Menschwerdung“ seines Kindes steuert. So wie Computersoftware nach der Verschrottung des Systems weiter verwendet werden kann. Selbst wenn man „Seele“ eigentlich noch immer nicht vollständig versteht, sie gilt eben als „transzendent“ und das ist auch wissenschaftlich korrekt.

    Anders die Formulierung als „Körper-Geist-Problem“.
    Geist ist zu vage „deklariert“. Geist könnte z.B. nur die jeweilige Prozesssteuerung im Gehirn als Oberbegriff bezeichnen. Das bezieht sich nicht auf die Information die auf Datenträgern weiter existieren und neu aktiviert werden kann. Bei dieser Formulierung mit der zugrunde liegenden Bedeutung, würde tatsächlich auch der Geist mit dem Tod enden.

    Es gibt aber mit bestimmten Menschen verknüpfte Informationen (Gene, Meme, …) die nach dem Tod eines Menschen weiter existieren, so wie Computersoftware nach der Verschrottung des Computer weiter existiert. Seele ist so etwas wie der „Oberbegriff“ für diese besonderen Informationen beim Menschen.

  99. @Balanus: Erklärungen

    Anstatt so ins Leere zu philosophieren, schlage ich vor, dass wir uns weiter darüber unterhalten, sobald wir physikalische (oder wenigstens biologische) Erklärungen für konkrete Alltagsphänomene haben – oder zumindest in sichtbarer Nähe dieses Ziels sind.

  100. @Elektroniker

    „Das „Leib-Seele-Problem“ sollte, anders als in der Formulierung als „Körper-Geist-Problem“, eindeutig „dualistisch“ gesehen werden.“

    Wie Sie es auch benennen, bisher konnte weder die dualistische noch die monistische Betrachtungsweise das Problem eindeutig lösen.

    „Es gibt aber mit bestimmten Menschen verknüpfte Informationen (Gene, Meme, …) die nach dem Tod eines Menschen weiter existieren, so wie Computersoftware nach der Verschrottung des Computer weiter existiert. Seele ist so etwas wie der „Oberbegriff“ für diese besonderen Informationen beim Menschen.“

    Ähnliche Spekulationen gibt es immer wieder. So hat man in den letzten Jahren herausgefunden, dass Pflanzen durch chemische Stoffe miteinander kommunizieren können. In diesem Zusammenhang ist ein PDF der Uni München interessant, wo gefragt wird, ob die Seele des Menschen nicht in der DNA stecken könnte:
    https://epub.ub.uni-muenchen.de/71115/1/Piesbergen_Jakob_Steckt_die_Seele_in_der_DNA.pdf

  101. @Chrys // 04.06.2020, 23:46 Uhr

    »Einverstanden?«

    Ja, einverstanden!

    Aber ich schätze, auch Sean Carroll würde da nicht widersprechen wollen.

    Er würde vielleicht sagen, dass an all den genannten Phänomenen offenkundig nichts anderes beteiligt ist als das, von dem er in seinem Blog-Post spricht: die bekannten Partikel und physikalischen Grundkräfte oder Gesetzmäßigkeiten.

    Und dem würde ich halt auch zustimmen.

    Wenn wir in die Phänomene hineinzoomen, landen wir trivialerweise irgendwann bei den Atomen und elementaren Teilchen und Grundkräften. Vielleicht nicht real, aber immerhin haben wir tragfähige Vorstellungen/Modelle von den fundamentalen Kräften und Wechselwirkungen.

    Und von diesen grundlegenden Wechselwirkungen ausgehend ergibt sich zwangsläufig alles andere. Soweit ich das überblicke, müssen wir keine neuen Grundkräfte oder Teilchen postulieren, wenn wir das Verhalten komplexer Systeme erforschen.

  102. Zu St. Schleim
    “Nervenzellen laufen in Zentralnervensystem zusammen und lassen Wahrnehmungen entstehen….”
    Diese Wahrnehmungen bekommen doch von uns erst eine Bedeutung, wenn wir sie bewertet haben, also die subjektiven LebensErfahrungen, die wir als unser ICH ,unsere Persönlichkeit ansehen, diese Wahrnehmungen in ein gelerntes Muster einordnen kann. Die Altvorderen nannten diesen Bewertungsvorgang eben GEIST. Im übrigen habe ich diesen Begriff nicht ins Spiel gebracht und wir können uns gern auch diesem Thema auf neurowissenschaftlicher Sicht nähern. Dieser Buddhismus ist für mich, da die “moderne” Psychologie für mich das Wesen des Menschen nicht erfasst, ein brauchbarer Denkansatz .

  103. @Querdenker: Ich und so

    Ich, das ist wieder eine andere Reifikation. Und Bedeutung, ja klar, das ist eine ganz andere Kategorie.

    Worüber diskutieren wir eigentlich gerade?

  104. Balanesisch:

    Wenn wir in die Phänomene hineinzoomen, landen wir trivialerweise irgendwann bei den Atomen und elementaren Teilchen und Grundkräften.

    Wenn ich also meine Erfahrung von diesem Bildschirm unters Elektronenmikroskop lege (hä?), dann sehe ich Atome?

    Einfach köstlich, diese Kategorienfehler!

  105. @Chrys: »Im Alltag trifft man allenthalben auf thermophysikalische Phänomene, …«

    Ist doch wohl klar, was @Chrys damit meint.

  106. Stephan Schleim
    …worüber diskutieren wir gerade ….
    ganz modern nennt man das Qualia-Problem.

    Zu einer Lösung wird es nicht kommen, soll es auch nicht, denn solche Diskussionen schärfen die Sichtweise und jeder profitiert davon.
    Mich würde interessieren, leiten Sie solche blogs aus Freude am Diskutieren, auf der Suche nach der Wahrheit , oder schreiben Sie ein neues Lehrbuch.

  107. @ Mona 05.06.2020, 10:16 Uhr

    Dass es viele noch ungelöste Probleme im Zusammenhang „Monismus – Dualismus“ gibt scheint klar.

    Nur behaupteten „Hardcore Monisten“ dass Information (Software) nicht einmal von Hardware getrennt werden kann, dass jemand der derartiges behauptet, grundsätzlich ein Betrüger wäre, wie die Pfaffen, die nicht damit aufhören wollen ähnliches zu behaupten. Berufskollegen (Techniker) mussten sich (zumindest früher) mit diesem Problem herumschlagen.

    In der Technik sieht man dies völlig sachlich. Es hat sich als ungeheuer zweckmäßig erwiesen, in Hardware und Software zu trennen, und die Software getrennt zu bearbeiten, zumal man gelernt hat, dass dies sehr leicht möglich und keineswegs unmöglich ist, was ehemals behauptet wurde.

    Wenn eine große und schwere, sehr teure technische Maschine, eines z.B. deutschen Lieferanten in Bombay modifiziert werden soll um neue Anforderungen zu erfüllen, reicht es oft, einfach eine neue Software übers Internet „einzuspielen“. Man erspart sich sozusagen den teuren Transport nach Deutschland und einen aufwändigen Umbau. Allenfalls schickt man eine Kiste mit Bauteilen nach Bombay ein,Techniker baut sie ein und spielt die neue Software auf.

    Selbst in der Psychologie ist es so, dass letztlich versucht wird auf die Musterverarbeitung im Gehirn Einfluss zu nehmen um Verhaltensprobleme zu beheben. Man baut zumindest derzeit noch, kaum neue Hardware ein.

    Dass man mitunter auch in der Wirtschaft „monistisch“ vorgeht, den Kunden liebere eine neue Maschine andreht, Kassa machen will, wenn es die Konkurrenz zulässt, ist auch klar.

    Die Autowirtschaft geht angeblich dazu über, den Kunden neue Software zu verkaufen, damit er nicht auf die neumodischen Mätzchen die ein anständiges Auto heutzutage beherrscht, nicht verzichten muss.

    Natürlich wäre es ein Problem, wenn die Philosophie, womöglich gar die Religionen, gezwungen würden, jedes Jahr neue „Updates“ ihrer Ideologie herauszugeben um die neuesten Erkenntnisse zu berücksichtigen, wie Microsoft.

    Dann müssen sie halt einen flexiblen „Systembaukasten“ (wie die Autofirmen), womöglich kompatibel zu allen Ideologien/Religionen entwickeln. Wenn man „psychologisch“ oder „religiös“ aktuell unterwegs sein möchte, lässt man sich halt (möglichst in der Nacht) mit den neuesten Updates „berieseln“, so dass man immer korrekt und auf neuestem Stand ist, wie der Laptop.

    Die „Seele steckt“ auch in der DNA, sie steckt auch in den Gedanken die man sich bei der Erziehung, in der Schule oder sonst irgendwo aufgerissen hat, die Mikrobenwelt aus Bakterien und Viren nimmt Einfluss, wie auch das Wetter oder die Umgebung….

  108. @ Elektroniker

    Ja, Stephan Schleim hat hier schon einmal einen Aprilscherz geschrieben, mit Upload und so…

    https://scilogs.spektrum.de/menschen-bilder/gehirn-upload-teilweise-gegl-ckt/

    Ich glaube nach wie vor nicht daran. Der Mensch mit seinem Hirn dürfte um einiges komplexer sein. Bisher weiß man immer noch nicht, wie oder was Bewußtsein ist, geschweige denn, daß man es uploaden könnte. Das müßte man aber können, wenn der Geist die Software und das Hirn mit seinen Neuronen die Hardware wäre. Ist schon lustig, daß der Mensch sich immer mit dem aktuellen Stand der Technik vergleicht. Bei Jacques de Vaucanson sah das Tier noch sehr mechanisch aus, heute rennt der Mensch also als rotierende Festplatte mit Betriebssystem herum. Na von mir aus. Bei manchen Menschen im Hamsterrad scheint es ja auch so zu sein.

  109. Zu St. Schleim
    “Worüber diskutieren wir eigentlich jetzt gerade…”
    Antwort: “Du bist der Geist, den du begreifst” (FAUST/Goethe)
    Zu D. Hilsebein:
    Der Geist in meiner Ansicht nach nicht die Software sondern er kann die Software im Gehirn, sprich die Prägungen , erkennen.

  110. @Dietmar Hildebein (Zitat):

    Bisher weiß man immer noch nicht, wie oder was Bewußtsein ist,

    Was man nicht genau definiert kann man auch nicht erforschen.
    Für mich bedeutet Bewusstsein vor allem, dass man erlebt, dass man sich selbst erlebt und dass man reflektiert. Bis zu einem gewissen Grad können das auch Tiere.

  111. @ Dietmar Hilsebein 05.06.2020, 17:15 Uhr

    Dass die Komplexität ein Problem ist, das ist naheliegend. Aber sofern es die elektrische Informationsverarbeitung betrifft, wäre irgendwann einmal eine Simulation, zumindest in Bereichen denkbar.

    Ich meine, man kann davon ausgehen, dass „Information“ letztlich in das Gehirn „hineinkommt“, dort verarbeitet wird, und auch wieder mehr oder weniger modifiziert, wieder „herauskommt“. Dass Strukturen aus Neuronen(verbänden) derartiges bewirken können, scheint zumindest für bestimmte Berufsgruppen die mit ähnlichen Systemen (allerdings weitaus geringerer Komplexität) gearbeitet haben „irgendwie“ denkmöglich.

    Es kommt nicht auf das „uploaden“ an, sondern auf den offensichtlichen Informationstransfer der beobachtbar ist.

    Beim „Bewusstsein“ gibt es aber auch ein „Empfindungssystem“ und das scheint nicht einmal in den Grundlagen annähernd erforscht.

    Ich habe mir in meinem nun schon längeren Leben einige vage Gedanken für eine Erklärung, wie es vielleicht zu „Empfindungen“ kommen könnte, „aufgerissen“.

    Das „Empfindungsphänomen“ könnte vielleicht dann entstehen, wenn bestimmte elektrisch – chemische Signalkombinationen an bestimmten Stellen (z.B. nach Nadelstich) in Wechselwirkung treten. Es könnte dann entstehen, wenn auf die „Bindungsdynamik“ zwischen Atomen/Molekülen Einfluss genommen wird. Dabei könnten Elektronen sozusagen aus ihrer Bahn fliegen, frei werden und Einfluss auf das neuronale Geschehen nehmen. Vielleicht kommt es auch auf bestimmte Phasen- und Resonanzeffekte an.

    Das chemische Geschehen können Sie natürlich nicht „hochladen“.

    An Bewusstsein „abbildenden“ (Zwischen – End-)Schichten von Gehirnorganen könnten auch so etwas wie „Rechenergebnisse (Bewusstseinskomponenten) abgebildet“ werden, ähnlich wie die Bildpunkte auf der Netzhaut, die dort zu einem Bild emergieren, dass dann im neuronalen Netz ausgewertet wird. Es ist eine Art „innerer Bildschirm“ eine Art „Hirnkino“.

    „Geist“ hat eher nur „Prozesscharakter“. Die „Information“, die „Muster“, werden in der räumlichen Struktur (der Neuronen) „abgebildet“. Veränderungen abbildende „Muster“ haben, anders als Daten, gleichzeitig Programmcharakter. Das Gehirn, eine Musterverarbeitungsmaschine“ wird sozusagen immer klüger, weil es fast unaufhörlich neue Muster im bestehenden System „eingliedert“.

    Dass man versucht die Gehirnfunktionen mit aktuellen technischen Systemen zu vergleichen ist naheliegend. (Dies galt früher Psychologen als verrückt). Man versuchte z.B. immer eine Verbindung zur Boolschen Algebra herzustellen, weil damit die Brücke zur seriösen Wissenschaft, sogar zur Philosophie „gebaut“ würde.

    Ich finde, das ist W. McCulloch und A. Turing geglückt. E. Kandel hat den Beweis für die Wissensabbildung geliefert.

    Es hat überhaupt nichts mit Festplatten zu tun, die sind nur Metapher.

    Gene sind offensichtlich Information und „bleiben“ nach dem Tod messbar existent. Für mich sind die Werke Goethes jedenfalls auch Information und die existiert noch…. Goethe schon lange nicht mehr.

  112. Hallo Herr Schleim, ich bevorzuge folgende Sicht:

    Wenn man etwas Komplexes wie z.B. ein Lebewesen beobachten, beschreiben und verstehen will, ist es sinnvoll und zweckmäßig, das Ganze in Teile zu gliedern. Dabei bilden die Begriffe Leib (oder Körper oder Materielles) einerseits und Seele (oder Geist oder Immaterielles) andererseits einen guten Ansatz zur Gliederung. Diese dann getrennt zu beobachten usw. kann helfen, das Lebewesen und sein Verhalten besser zu verstehen und einzuschätzen.

    Getrennte Beobachtung usw. von Leib und Seele heißt aber nicht, dass sie auch getrennt sind oder unabhängig voneinander bestehen. Ein Leib ohne Seele ist ein Leichnam. Eine Seele ohne Leib kann in dieser Welt nicht bestehen. Ob das in einer jenseitigen Welt geht, ist eine Frage des Glaubens.

    Dass man noch nicht ganz versteht, wie ein Lebewesen aus Sinneseindrücken und Erfahrungen letztlich Entscheidungen trifft und Handlungen durchführt, ist nicht weiter schlimm. Offensichtlich können Lebewesen das. Manche sogar mit Vernunft.

  113. Zu Carroll und Reduktionismus:

    Bzgl. Carrolls Satz „Alles, was wir brauchen …“ liegen Sie auch aus meiner Sicht mit Ihrer Schlussfolgerung „Wenn das stimmte, wären …“ und dem „extremen Reduktionismus …“ daneben. Ich habe den Satz beispielhaft umformuliert mit Blick auf die Mathematik:

    “Alles, was wir brauchen, um alles zu erklären, was wir in der Mathematik sehen, sind eine Handvoll Axiome (als wahr angenommene Grundsätze) und die grundlegenden Regeln zum Folgern von wahren Aussagen / Sätzen aus (bekannten) wahren Aussagen / Sätzen.“

    Das ist etwas flapsig formuliert, stimmt aber im Wesentlichen. Und es führt NICHT dazu, dass die Mathematiker überflüssig würden. Die würden beim Versuch, eine neu formulierte Vermutung zu beweisen oder zu widerlegen, nie und nimmer bei den Axiomen beginnen sondern auf bekannten Aussagen / Sätzen aufsetzen.

  114. @Harald Andresen:

    Alles, was wir brauchen, um alles zu erklären, was wir in der Mathematik sehen, sind eine Handvoll Axiome (als wahr angenommene Grundsätze) und die grundlegenden Regeln zum Folgern von wahren Aussagen / Sätzen aus (bekannten) wahren Aussagen / Sätzen.“

    Sicher. Wobei Axiome+Logik ist nicht das was Mathematik interessant macht. Das zeigen gerade die Milleniums-Probleme. Die Riemannsche Vermutung beispielsweise hat nichts direkt mit Axiomen zu tun, sondern es ist eine Vermutung, deren Beweis wohl ganz verschiedene Gebiete der Mathematik miteinbeziehen muss. Es stimmt aber, dass Beweise letztlich bis zu Axiomen vordringen müssen, dass sie alles darauf zurückzuführen müssen. Doch das interessante an der Beweisführung ist nicht die Rückführung auf Axiome, sondern es ist, welche Mathematikgebiete herangezogen werden und welche neuen Wege der Beweis öffnet.

  115. @ Harald Andresen 05.06.2020, 22:17 Uhr

    Zitat:“Wenn man etwas Komplexes wie z.B. ein Lebewesen beobachten, beschreiben und verstehen will, ist es sinnvoll und zweckmäßig, das Ganze in Teile zu gliedern….“

    Da stimme ich Ihnen begeistert zu.

    Praktisch alles was man „verstehen“ will, wird gegliedert. In der Statik werden die Kräfte zerlegt in Komponenten um z.B. die Standfestigkeit eines Bauwerks zu berechnen. In der Elektrotechnik z.B. die Komponenten (Wirk- Blindanteil) des Wechselstromes. Bilder werden bei der Übertragung oder auch Speicherung in farbige „Bildpunkte“ zerlegt, und die werden auf „flächigen Strukturen“ (z.B. einer Leinwand, oder Bildschirm ….) gleichzeitig zur „optischen Anzeige“ gebracht, damit die einzelnen Bildpunkte wieder zu einem Bild emergieren können.

    Und ich behaupte, einzelne „Musterkomponenten“ können im Gehirn egal ob sie „Bilder“, „abstrakte Zusammenhänge“, ….. oder auch „Empfindungen“ abbilden, an so etwas wie „flächigen Bruchstellen“ (wie Leinwand) zum dynamischen Bewusstsein emergieren. Derartige “Bruchstellen” könnten im Gehirn an der Schnittstelle verschiedener Neuronentypen (Gehirnorgane) lokalisiert sein. (Wie es an der Netzhaut ja auch eindeutig realisiert ist).

    Die Neuronen mit ihrer Gatterfunktion (W. McCulloch) können Information aufspalten, was sie lt. obigem Denkansatz auch müssen, und danach bestimmte Komponenten selektiv zusammenführen, eben um das „vollständige Muster“ z.B. ein Feuerwehrauto zu erkennen.

    Beispiel: Aus nebeneinander (allenfalls hintereinander) auftretenden Bildpunkten (Musterkomponenten) im Auge, wird zusätzlich ermittelt, dass benachbarte Bildpunkte jeweils z.B. farblich oder hinsichtlich einer bestimmten „Krümmung“ gleich, oder musterabhängig verschieden sind. Mittels der Synapsen lernen die jeweiligen Neuronen für welches „Mikromuster“ sie jeweils zuständig sind. Die Information „spaltet“ sich auf.

    Danach werden die informationsabhängigen Signalmuster, ebenfalls wegen der Gatterfunktion der Neuronen, selektiv zusammengeführt. In einer Hierarchie werden Merkmale erkannt, z. B. Räder die „unten“ sind, letztlich wird aus der Auswertung der vielen Musteromponenten das gesamte Objekt „erkannt“.

    Beim Menschen kann z.B. das „Gesehene“ (dessen „Muster“) mit sprachlichen „Mustern“ interagieren, so dass die bekannten Effekte, z.B. dass ein Mensch „Sachverhalte“ sogar anderen Menschen erfolgreich mitteilen kann, entstehen.

    Die Kommissare im Krimi machen es genau so. Sie zerlegen die Situation in kleine „Puzzlestücke“, die führen sie danach zusammen, bis sie den Täter haben.

    Diese extrem komplexen „Aufspaltungs- und Zusammenführungsprozesse“ sind selbst mit nicht besonders intelligenten, aber dafür extrem vielen Gattern (realisiert als Neuronen) im Sinne McCullochs möglich.

    Neurone die wenige Inputsignale aufspalten müssen, sollten daher eher weniger Dendriten haben, Neuronenverbände die aus weiten Bereichen und jeweils sehr viele der Merkmale abbildenden Signale verknüpfen müssen, sollten entweder viele Dendriten haben, oder sehr viele entsprechend kaskadierte Neuronen.

    Ein Leib ohne Seele ist ein Leichnam. Das stimmt.

    Eine Seele ohne Leib (die das sozusagen „kommen“ sieht), „sucht“ oder „macht“ sich einfach selbst einen neuen Leib, dann kann sie in dieser Welt sehr wohl bestehen. Sie muss nur auf bestimmte „Organe“ (die ich hier aus vornehmen „Anstand“ nicht explizit erwähnen möchte), entsprechend “Druck” ausüben, dann klappt´s auch mit dem Weiterleben……., selbst ohne Glauben.

  116. @Harald Andresen / 05.06.2020, 22:21 Uhr

    »Das ist etwas flapsig formuliert, stimmt aber im Wesentlichen.«

    Ganz so einfach ist es dann doch nicht mit der Metamathematik.

    Ein formaler Kalkül heisst vollständig, wenn alle seine wahren Sätze auch mit seinen Ausdrucksmitteln beweisbar sind. Nun hat Gödel bekanntlich gezeigt, dass die Peano Arithmetik nicht vollständig ist. Das heisst, es existieren in der PA wahre Sätze, die in nicht aus ihren Axiomen formal herzuleiten sind. Weiters hat Tarski dann noch gezeigt, dass in der PA auch nicht formal entscheidbar ist, welche ihrer Formeln wahr sind (Tarskis Theorem von der Undefinierbarkeit der Wahrheit).

    Solche zunächst sehr abstrakt anmutenden Resultate haben durchaus philosoph. Relevanz, zumal in der strukturalist. Wissenschaftstheorie, weshalb man auch bei Stegmüller viel dazu findet.

    In der Konsequenz lässt für jede physikal. Objekttheorie (wie QM oder RT) etwas benennen, was sie garantiert nicht erklären kann, nämlich ihre eigene Metatheorie. Auch das ist jetzt etwas flapsig formuliert, doch dabei werden wir es hier belassen müssen.

    Mit Hinblick auf das Mind-Body Problem liesse sich aber noch sagen, für eine physikal. Theorie ist Mind naturgemäss stets metatheoretisch und nie objekttheoretisch, allenfalls Body kann als ein Gegenstand objekttheoret. Betrachtungen konstituiert werden. Was erweitert dann auch für Chemie und Biologie gilt, nicht nur für die Physik.

    Und wenn man dann Body physikalisch, chemisch oder biologisch inspiziert, findet man dabei auch nur “Teile, die einander stossen”, wie Leibniz es schon gesagt hat, aber eben keine Gedanken, keinen Mind.

  117. @ Elektroniker 06.06.2020, 08:56 Uhr

    Ein Leib ohne Seele ist ein Leichnam. Das stimmt.

    “Nicht notwendigerweise”. Im Buddhismus gibt es Wesen/Existenzformen, die ohne Bewusstsein sind (nur bestimmte körperliche Merkmale haben). Seele im klassische Sinn gibt’s im Buddhismus eh nicht. Das liegt daran, dass sie des Denkens (Philosophierens usw.) überdrüssig wurden und entsprechend reinkarnierten. Später kamen ihnen aber doch wieder Gedanken/Ideen und sie wurden normal (Leib mit “Seele” bzw. Bewusstsein) weitergeboren.

    Eine Seele ohne Leib (die das sozusagen „kommen“ sieht), „sucht“ oder „macht“ sich einfach selbst einen neuen Leib, dann kann sie in dieser Welt sehr wohl bestehen. Sie muss nur auf bestimmte „Organe“ (die ich hier aus vornehmen „Anstand“ nicht explizit erwähnen möchte), entsprechend “Druck” ausüben, dann klappt´s auch mit dem Weiterleben……., selbst ohne Glauben.

    Hier handelt es sich offenbar um den Sexualtrieb – das äußerst natürliche/normale Bedürfnis die Art zu erhalten (ein starker Überlebenswille).

    Im Buddhismus gibt es auch Wesen/Existenzformen, die reine Seele (bzw. reines Bewusstsein) sind – keinen Körper haben. Sie wurden des Körpers überdrüssig (da er nicht selten Probleme macht) – und respiritualisierten dann als purer “Geist” (wo ein Wille ist, ist auch ein Weg). Dann aber kamen ihnen doch wieder Vorstellungen i.V.m. angenehmer Körperlichkeit (vll. sexueller Art) und sie wurden entsprechend weitergeboren.

  118. @Axel Krüger

    „Das liegt daran, dass sie des Denkens (Philosophierens usw.) überdrüssig wurden und entsprechend reinkarnierten.“

    Und manche kommentieren zuweilen auch auf den Scilogs!

  119. @ Axel Krüger 06.06.2020, 16:49 Uhr

    Ich würde „Seele“ ganz cool, eben so „deklarieren“ (im Sinne der Informatiker), dass sie genau die Information ist, die nach dem Tod wirklich übrig bleibt. Das ist eine Tautologie und immer richtig.

    Dass zumindest genetische Information übrig bleibt, ist eher unbestritten, zumindest bei den Menschen die alltäglich mit Information zu tun haben. (Es dürfte noch mehr Information übrig bleiben, aber das ist nicht so offensichtlich begründbar). Diese Information kann die Entwicklung biologischer Systeme so steuern, dass sie mit oder ohne Bewusstsein ausgestattet sind.

    Und dies bewundere ich beim Buddhismus. Würden Buddhisten die “Deklaration” so wie oben angeführt akzeptieren, so würde der Glaube an die Seelenwanderung vermutlich der Realität besonders nahe kommen.

    Manche Wissenschaftler nehmen an, dass mobile Informationsträger ähnlich wie z.b. Viren, in bestehende DNA gesteuerte Systeme „eingebunden“ wurden, die die Mutationen (für die Darwinsche Auswahl) bewirkt haben, die für die Entwicklung aller biologischen Systeme bis hin zum Menschen notwendig waren und womöglich noch immer sind.

    Es mag zwar etwas übertrieben erscheinen, wenn religiöse Anführer indischer Religionen absolut keine Lebewesen umbringen wollen, weil sie Information (Seelen) vernichten könnten, die die Evolution später einmal vielleicht noch brauchen könnte. Sie fegen sogar mit einem kleinen Besen den Weg, damit keine Lebewesen zertreten werden können.

    Andererseits sind z.B. die Coronaviren gerade dabei eine gewisse Auswahl zu treffen, welche Menschen weiterleben sollen.

    Es fragt sich halt, ob wir uns das von der „Natur gefallen“ lassen sollen?

  120. @Elektroniker 06.06.2020, 08:56 Uhr

    🙂

    … und, wie Sie dargestellt haben, nach dem Zerlegen (und der Einzel-Analyse usw.) das Zusammenführen nicht vergessen.

    Ich hatte geschrieben „Eine Seele ohne Leib kann in dieser Welt nicht bestehen“ und dabei an die individuelle Seele gedacht. Ansonsten sehe ich das ähnlich wie Sie:

    Ja, es gibt ein Leben nach dem Tod, „selbst ohne Glauben“. Im biologischen Sinne, indem man Nachkommen hat, denn in denen lebt ein Teil von einem selbst weiter. Im übertragenen Sinne, indem sich andere an einen erinnern. Dabei sind Schriftsteller wie Goethe natürlich im Vorteil.

  121. @Chrys 06.06.2020, 12:00 Uhr

    Nun ja … auch für Herleitung und Beweis des Unvollständigkeitssatzes wird Herr Gödel die genannten „Zutaten“ wie Axiome, Regeln für Folgerungen und bekannte Sätze / Aussagen verwendet haben.

    Aber wie steht es um die Anwendung der Gödelschen Unvollständigkeitssätze auf Bereiche, die nicht zur Mathematik gehören?

    Etwas überspitzt: Der Gedanke, dass es im System „Lebewesen“ Aussagen geben kann, die innerhalb dieses Systems weder bewiesen noch widerlegt werden können, und dass es sich dabei z.B. um Aussagen über das Denken an sich handeln könnte, ist bestimmt schon gedacht worden. 😉

  122. Das Leib-Seele- oder Körper-Geist-Problem ist ein Produkt von Polarisierung und Ideologisierung im Verlauf der Kultur- und Sprachgeschichte. Auf einer Seite steht der physikalische oder materielle Körper und auf der anderen die immaterielle Spiritualität als Geist. Phantasie und Ideologie haben die Unvereinbarkeiten gedanklich und sprachlich verfestigt in der Form von Götterglaube oder Aberglaube einerseits und szientistische Naturwissenschaft oder Physikalismus andererseits.

    In der Wirklichkeit gibt es längst Brückenglieder zwischen den Polen, die uns als solche oft nicht bewusst werden. Der Körper ist nicht nur physikalisch, er ist als lebender Organismus vor allem biologisch. Der Geist ist nicht nur spirituell, er ist auch psychologisch und psychophysiologisch oder psychosomatisch. Es gibt begriffliche Lücken für den Übergang von einer Seite zur andern, weil wir zwar schon viel, aber immer noch zu wenig wissen über die Formen des Dazwischen.

    Das Körper-Geist-Problem zeigt sich auch als Übergang von Kausalität zu Motivation. Das lässt sich an banalen Beispielen zeigen: die Stellung des Wasserhahnes ist die kausale Ursache, ob kaltes oder heißes Wasser aus der Leitung sprudelt. Wenn ich die Hand unter den Wasserhahn halte, empfinde ich kaltes oder heißes Wasser auf Grund der Sinnesnerven in der Haut, deren Signale im Gehirn münden. Also verändere ich(!) motivational die Stellung des Wasserhahnes bis angenehm warmes Wasser auf die Haut trifft.

    In einer Zellmembran trifft ein Transmittermolekül auf ein Rezeptormolekül. Wenn die Moleküle “zusammenpassen”, dann wird das Rezeptormolekül verändert und sendet dadurch ein “Signal” in das Zellinnere bis zum genetischen Apparat, wodurch ein Gen exprimiert und ein Protein erzeugt wird, das wiederum eine Wirkung ausführt. Die Begriffe “Molekül” und “Signal” bezeichnen hier dasselbe, gehören aber unterschiedlichen Erklärungs- oder Funktionsebenen an. Ein Molekül ist ein Element der molekularbiologischen Erklärungsebene und ein Signal (Auslöser einer Information!) ist ein Element der kybernetischen Erklärungsebene, zu welcher weitere Elemente wie Filter, Vergleicher, Schablonen und Schalter gehören, die wir alle im Organismus wie im Gehirn in großer Zahl vorfinden.

    Das eigentliche Körper-Geist-Problem besteht darin, weitere Begriffe zu finden, die den Übergang von Körper zu Geist beschreiben, die also zwischen verschiedenen Erklärungs- oder Funktionsebenen übersetzen können. Ein Gehirn lässt sich nicht so einfach untersuchen wie ein Auto, wo jeder Fahrer weiß, was die Lenkung und die Bremsen bedeuten und bewirken, wie sie funktionieren, wie und wann sie bedient werden und welche Folgen sie haben oder haben können. Niemand käme auf die Idee, ein Auto dualistisch zu erklären. Das wird sich wohl auch bei selbstfahrenden Autos nicht ändern, es sei denn, eines käme an den Amazonas zu einem “primitiven” Volk.

    Was wir sehr wahrscheinlich nie wissen werden, das sind die sogenannten “Qualia” oder phänomenalen Bewusstseinsinhalte wie Farben, Töne und Gefühle. Wir wissen nicht, warum wir Farben als gelb oder blau sehen, obwohl wir wissen, wie sie biologisch zustande kommen. Das dürfte ein grundsätzliches, physikalisches Rätsel sein und bleiben.

  123. Harald Andresen,
    Teile, die nicht zur Mathematik gehören, die gibt es , aber über die lässt sich nicht sprechen. Die Grundlage der Mathematik ist die Logik. Die ist Grundlage der Mathematik. Die Logik ist auch die Grundlage des Sprechens und Schreibens.
    Was sich nicht sagen lässt, das sind die Gefühle, die lassen sich nur beschreiben aber nicht durch die Sprache abbilden. Höchstens durch Musik , Malerei und bildende Kunst.
    So gesehen ist Mensch nicht nur Naturgegenstand, sondern auch Kulturgegenstand.

  124. @Harald Andresen
    Wenn man etwas Komplexes wie z.B. ein Lebewesen beobachten, beschreiben und verstehen will, ist es sinnvoll und zweckmäßig, das Ganze in Teile zu gliedern.

    Das klingt vernünftig, aber wie kommen wir dazu, etwas als “Ganzes” zu sehen und zu verstehen? Warum oder woran erkennen wir ein Lebewesen, oder andere Dinge der Welt, als Einheit? Das ist nicht selbstverständlich, sondern beruht schon auf Vorerfahrungen und Vorannahmen.

    Wir müssen die Methoden und Funktionen der Wahrnehmung und des Erkennens erforschen, dann verstehen wir die Welt viel besser. Das Zerlegen setzt schon voraus, dass man etwas als zusammengesetzt erkannt hat! Beim Zerlegen gehen genau jene Eigenschaften verloren, die das Verständnis eines Dinges als Einheit ausmachen. Das Zerlegen, also das Prinzip “Erkenntnis durch schlachten”, kann nicht die Lösung bringen. Unbestritten und pragmatisch können damit aber singuläre Erkenntnisse gewonnen werden.

  125. @ anton reutlinger 07.06.2020, 11:08 Uhr

    Zitat: „Das Leib-Seele- oder Körper-Geist-Problem ist ein Produkt von Polarisierung und Ideologisierung im Verlauf der Kultur- und Sprachgeschichte.“

    Das stimmt offensichtlich, nur spielt der „pure Götterglaube“ so gut wie keine Rolle mehr. Er wurde sozusagen abgelöst, vom „Softwareglauben“. Und “ Hardware – Software“ ist nun einmal typisch traditionell „dualistisch“. Wobei eigentlich auch „Prozesse“ als eine 3. wichtige grundlegende Kategorie dazu gehören würden.

    Hardcore „Monisten“ haben übrigens ehemals sogar die Existenz von „Software“ bestritten, und Softwaretechniker für Betrüger gehalten, so wie auch die Pfaffen.

    „Gott“ ist letztlich „Bezeichner“ für alle „wirklichen“ Gesetzmäßigkeiten, alle Prozesse, jegliche Materie die im Zusammenhang mit der Welt stehen. Weil man nicht alles versteht, gilt „Gott“ als transzendent. Diese Aussage habe ich von einem Theologen vor einigen Jahrzehnten im Radio ungefähr so gehört und sie ist praktisch eine Tautologie, die nicht falsch sein kann. Ob einem dies reicht, oder man genaueres wissen möchte, ist eine andere Frage.

    Indische „Theologen“ scheinen mit ihrem Konzept der „Seelenwanderung“ der tatsächlichen Realität, wie sich die biologischen Systeme tatsächlich entwickelt haben, recht nahe gekommen zu sein. (Was ich in einem früheren Beitrag angedeutet habe).

    Bei den Autos spielt zunehmend die Software und eventuell das Aussehen eine entscheidende Rolle. Warum soll man dies nicht „dualistisch“ sehen?

  126. @ anton reutlinger 07.06.2020, 14:31 Uhr

    Beim Denkprozess erfolgt offensichtlich zunächst eine Zerlegung z.B. in Bildpunkte beim Auge und in so etwas wie „Mikromuster“ (Veränderungen wie z.B. Krümmungen…), die danach hierarchisch ausgewertet werden, von „Teilmusterkomponenten“ letztlich zu „Gesamtmuster“.

    Wir sind praktisch überzeugt, dass die Gesamtmuster „richtig“ sind. Das beruht offensichtlich auf Vorerfahrungen und Vorannahmen, wie Sie es beschreiben.

    Natürlich sind die Erfahrungen individuell und können auch falsch sein. Das Gehirn arbeitet offensichtlich nach dem Konzept einer Musterverarbeitungsmaschine, die die Realität nur annähernd abbildet. Es arbeitet mit Heuristiken und die sind nun einmal nicht immer absolut logisch und auch eindeutig wahr oder falsch.

    Wäre es anders, also „streng logisch“, dass System käme extrem oft auf Widersprüche und würde sich selber blockieren. Es wäre nicht existenzfähig.

  127. @Harald Andresen / 06.06.2020, 22:56 Uhr

    »Aber wie steht es um die Anwendung der Gödelschen Unvollständigkeitssätze auf Bereiche, die nicht zur Mathematik gehören?«

    Siehe etwa Douglas Hofstadter, Gödel, Escher, Bach (Basic Books, 1979, 1999). Er schlägt dort u.a. den Bogen von Gödel bis hin zum Geist-Körper Problem.

    Wenngleich das freilich keine `Anwendung’ des Theorems im üblichen Sinne ist und auch gar keine sein kann. Vielmehr geht es ihm um das Herausstellen eines gemeinsamen strukturellen Merkmals, das auch anderswo (Escher, Bach) noch von Belang ist — der Strange Loop.

  128. @ Elektroniker: Stellen Sie sich vor, ein Rad rollt den Berg hinunter. Wie würden Sie diesen Vorgang beschreiben? Etwa durch das Verhalten von Atomen und Molekülen des Rades? Nein, natürlich nicht, die Informationsdichte wäre viel zu gering. Aber nicht nur auf der beschreibungsebene verhält es sich so, sondern auch auf der Funktionsebene: Atome und Moleküle müssen mit dem Rad den berg hinunter, ob sie wollen oder nicht. Was bedeutet das? Atome und Moleküle sind zwar für das Rd konstitutiv, das Rad aber ist für Atome und Moleküle regulativ oder wie Eric Hoel, ein theoretischer Neurowissenschaftler sagte, das Makro schlägt das Mikro. Das gilt übrigens für das gesamte Universum. Ich habe versucht, dies auf den Organismus zu übertragen und ein Modell entwickelt, mit dem man Informationsverarbeitung zwischen den Regulationsebenen darstellen kann. Damit wird Krankheit etwa darstellbar als Chaos, in welchem zufallsbedingt neue, pathogene Strukturen entstehen, sowohl neuronal wie genetisch.

  129. @anton reutlinger 07.06.2020, 14:31 Uhr

    Upps, bei dem Versuch, das kurz und knapp und trotzdem treffend zu formulieren, habe ich den „Untersuchungsgegenstand“ selbst zerlegt. An Metzgerei oder Pathologie hatte ich aber nicht gedacht.

    Mir geht es um eine strukturierte Vorgehensweise und eine damit verbundene Gliederung. Dazu sollten unterschiedliche Aspekte der Untersuchung oder unterschiedliche Sichten auf den Untersuchungsgegenstand ebenso gehören wie (vermutete oder bereits erkannte) unterschiedliche Anteile des Untersuchungsgegenstands. Dass die getrennte Untersuchung von Anteilen nicht bedeutet, dass diese auch getrennt sind oder unabhängig voneinander bestehen, ist mir bewusst.

    Danke für den Hinweis.

  130. @ Wolfgang Stegemann 07.06.2020, 20:54 Uhr

    Zitat: „Ich habe versucht, dies auf den Organismus zu übertragen und ein Modell entwickelt, mit dem man Informationsverarbeitung zwischen den Regulationsebenen darstellen kann. Damit wird Krankheit etwa darstellbar als Chaos, in welchem zufallsbedingt neue, pathogene Strukturen entstehen, sowohl neuronal wie genetisch.“

    Ich fände es interessant, wenn Sie das näher erklären könnten. Möglichst anschaulich und aufbauend auf „Allgemeinwissen“ in Chemie.

    Der Sachverhalt scheint im „Grenzbereich“, wenn Information abgebildet auf elektrischen Signalen, auf elementare chemische Prozesse möglicherweise Einfluss nimmt und auch umgekehrt. Organische Strukturen können geschädigt bis zerstört, möglicherweise auch repariert werden.

    Die schweren, mit der Psychosomatik zusammenhängenden Probleme im Gesundheitsbereich, als vermutlich auch die Entstehung von Bewusstsein an der Schnittstelle „elektrisch“ arbeitendes neuronales Netz – Chemie, könnten besser verstanden und erklärt werden.

  131. @Wied: Qualia…

    …bzw. Quale (Einzahl) hört sich für mich irgendwie nach einer Verdinglichung an. Vielleicht lässt sich das Problem nicht lösen, weil es das gar nicht gibt?

    Und zu Ihrer Frage nach der Motivation: Mich hat halt anno 2007 jemand gefragt, ob ich das hier machen will, ich leide unter einem Schreibzwang und es macht hin und wieder auch Spaß. Die beiden jüngeren Einmaleins-Artikel waren tatsächlich aber auch als Sahnehäubchen für ein neues Buch gedacht. Darüber bei Gelegenheit einmal mehr.

  132. @Andresen: Carrol und so

    Da wollte ich Ihnen gerade schreiben, dass sich das nach einem guten Kompromissvorschlag anhört, und dann kommt noch dieser Folgekommentar über die Mathematiker…

    Ihr Gedankengang entbirgt eine gewisse Originalität. Ich vermute hier aber doch einen Haken in der Analogie beziehungsweise eine petitio principi. Denn dass beim Mathematiker aus den Axiomen und Regeln alles Weitere folgt, ist trivial. Ob das beim Physiker und den Alltagsphänomenen (einschließlich all ihrer kulturellen Facetten) auch der Fall ist, müsste aber erst gezeigt werden.

    Ich vermute hier eher einen Kategorienfehler (d.h. die Anwendung der Physik auf Bereiche, in denen sie Prinzipiell nichts aussagen kann). Nehmen wir einmal etwas wie die Frage, ob Berlin oder Bonn (im Jahr 2020) die Hauptstadt von Deutschland ist. Das sind Sachverhalte unseres Alltags, die die Leben von Millionen Menschen beeinflussen, für deren Verständnis uns die Physik aber nicht weiterhilft.

  133. Das Leib-Seele Problem ist letztlich eine Art Henne-Ei-Problem, hat also mit der Frage zu tun, was kommt zuerst, die physische oder die psychische Welt.
    Ein Philosoph, dessen Metier das (reine) Denken ist, der weiss natürlich, dass der Mensch und Philosoph, welcher nichts denken kann, gar nichts kann, dass jemand in dieser Situation (ohne Denkmöglichkeit) nicht einmal weiss, dass er existiert, was dann zu Descartes Aussage führt: “Ich denke, also bin ich.” Dieser Ansatz, der das Denken in den Mittelpunkt stellt, führt auch zwangslos zum Anthropzozentrismus, bei dem der Mensch im Mittelpunkt der Welt steht und ebenso zum Konstruktivisimus, einer Philosophierichtung, die gemäss dem deutschen Philosophen Markus Gabriel ansatzweise bereits seit Immanuel Kant existiert und welche behauptet, dass es keine sichere Erkenntnis einer äusseren Welt geben könne, denn unsere Sinne und unser Geist erzeugten die von uns erlebte Wirklichkeit und wir hätten keine Möglichkeit einer objektiven Weltbetrachtung, weil wir alles nur indirekt über Sinneserfahrungen und ihre Filterung/Verarbeitung angeboten bekämen.

    Ist das Hirn (unser Geist) die Wirklichkeit? Gibt es also keine externe Realität?
    Letztlich geht es hier um die Frage, ob es eine externe Realität überhaupt gibt. Dass konstruktive Prozesse, die unsere (erlebte) Wirklichkeit erschaffen im Geist und Hirn und selbst in einem künstlichen Hirn eines Roboters eine Rolle spielen, darüber gibt es keinen Zweifel. Doch dass diese Wahrnehmungs- und Erkennungsprozesse eine beliebige Welt erzeugen kann, die nichts mehr mit der äusseren Realität zu tun hat oder die sogar die Existenz einer äusseren Realität in Frage stellt, das anzunehmen bedeutet eine Grenze zu überschreiten: Die Grenze nämlich zwischen dem Glauben an eine Welt in der man sich behaupten muss und dem Glauben an eine Welt die man selbst konstruiert und in der man sogar selbst bestimmt was wahr und was falsch ist (fast wie Donald Duck (äh Donald Trump) oder Gott).

    Mit der zunehmenden Bedeutung der Lebenswissenschaften (Biologie, Evolution, Medizin), der Physik und den anderen Naturwissenschaften wird eine Sicht, die das Denken als Ursprung aller Dinge betrachtet und die den Dingen selbst keine Eigenexistenz zugesteht immer fragwürdiger.
    Denn die Lebenswissenschaften lehren uns, dass es Geschöpfe gibt, die einiges mit uns gemeinsam haben inklusive einfacher Formen des Denkens und die Physik und Kosmologie lehren uns sogar, dass es eine Entwicklung im ganzen Universum gibt, die trotz immer gleicher physikalischer Gesetze, Dinge schafft, die letztlich auch Lebewesen und den Menschen hervorgebracht haben können.

    Big History: Der Mensch als Tintenklecks im Universum
    Diese Betrachtung des Menschen von den Naturwissenschaften her (und nicht vom Mensch und seinem Denken her) hat inzwischen sogar die Geisteswissenschaften beeinflusst. In den Geschichtswissenschaften führte es zu dem was man Big History nennt. Gegenstand der Big History ist immer noch der Mensch und seine Geschichte. Doch Big History betrachtet die Menschheitsgeschichte nur als eine sehr kurze Phase in der Gesamtgeschichte des Universums und Big History vertritt implizit die Ansicht, dass man die Geschichte des Menschen nur verstehen kann wenn man die Einbettung dieser Geschichte in die Geschichte unseres Universums kennt.
    Nur wenige kennen den Begründer der Big History, David Christian der Macquarie Universität in Sydney. Viel mehr kennen aber Bill Gates, einen expliziten Förderer der Big History. Und fast so viel oder noch mehr kennen Yuval Harari, der Bücher wie “Eine kurze Geschichte der Menschheit” geschrieben hat und der darin genau diesen Ansatz der Big History wählt um nicht nur die historisch überlieferte Menschheitsgeschichte zu erzählen, sondern um die gesamte Spanne von der Entstehung des Homo sapiens vor ein paar hundertausend Jahren bis heute neu zu erklären. Obwohl Yuval Harari ein ausgebildeter Historiker ist, beschreibt er den Menschen sehr stark von der naturwissenschaftlichen Seite her und von behaupteten Kognitionsfortschritten gegenüber seinen Tiervorfahren, wie etwa der Fähigkeit zu fiktiven Denken und der Bereitschaft des Menschen sich an willkürliche Abmachungen zu halten wie etwa die, dass eine geprägte Münze einen bestimmten Wert darstelle.
    In Yuval Hararis Büchern ist der Mensch zwar ein Geschöpf mit völlig neuen Möglichkeiten des Denkens und Handelns, aber es sind nicht die einzelnen Menschen (etwa Napoleon), die das Entscheidende schaffen, sondern es sind die eingeborenen neuen (Denk-) Fähigkeiten des Menschen, die sein Schicksal bestimmen und ihn antreiben. Damit erklärt Harari den Menschen ähnlich wie der Physiker die Welt der Teilchen erklärt. Es sind Kräfte (angeborene, vererbte), die auf den Menschen einwirken und die in ihrem Zusammenwirken dann die landwirtschaftliche und später wissenschaftliche Revolution anstiessen. Der Einzelmensch ist in Hararis Weltentwurf vergleichbar mit dem Elementarteilchen des Physikers, denn er ist wie Elementarteilchen, Kräften unterworfen, die stärker sind als es selbst.

    Fazit: Die Naturwissenschaften und neue populäre Ansätze in den Geisteswissenschaften wie Big History sehen den Menschen und die Menschheitsgeschichte als zeitliches und materielles Partikel in einer sehr viel grösseren Welt. Der menschliche Geist in dieser naturwissenschaftlichen Sicht ein Produkt eines langen Prozesses und nicht etwa der Urheber der Welt. Während viele Philosophen mit der Idee liebäugelten, der Mensch sei eigentlich Gott, halten Naturwissenschaftler den Menschen für ein Produkt des Universums wobei gerade Yuval Harari mit seinem Buch “Homo Deus” auch die Vision aufleben lässt, dass der Mensch mithilfe von Wissenschaft und Technik selbst zum Gott werden kann.

  134. @Martin Holzherr

    Zur „Big History“: Populärwissenschaftliche Zusammenfassungen des Weltgeschehens gab es doch bereits in der Vergangenheit, sogar für Kinder. Ich weiß nicht, was ein dermaßen verengter Blick auf die Welt bringen soll. Wer sich näher mit sozioökonomischen Entwicklungen usw. befassen möchte, dem bleibt ein erweitertes Studium wohl nicht erspart.

  135. @Elektroniker: Hierzu in aller Kürze. Es handelt sich um ein systemtheoretisches, reduktionistisches Modell, das die einzelnen, in der Evolution entstandenen und aufeinander aufbauenden Regulationsebenen von der Zelle (Einzeller) zum Gewebe (Mehrzeller) über Organe (Organismen) bis zum Zentralnervensystem (z.B. Mensch) in ein kausales Verhältnis setzt, welches das „Ganze“ als Regulierendes gegenüber dem „Einzelnen“ sieht, wobei das Ganze immer repräsentiert wird durch die jeweils entwickeltste Ebene, beim Menschen also das Zentralnervensystem, das den gesamten Organismus steuert. Jede Ebene arbeitet emergent nach ihrer eigenen Logik, die Zelle etwa proteinbildend, das ZNS elektrisch. Bei der Informationsverarbeitung müssen elektrische Impulse in Richtung Organe, Gewebe oder Zellen jeweils systemintegrativ sein, ansonsten geraten sie in sog. Grenzzyklen, werden also krank. Bei der Entstehung von Krebs etwa werden nicht bestimmte Onkogene eingeschaltet (von wem auch), sondern auf der der Zelle übergeordneten kontrollierenden Ebene, der extrazellulären Matrix, entsteht „Unordnung“, was dazu führt, daß auf Zellebene Prozesse nicht mehr integrativ gesteuert werden, sondern rein zufällige Prozesse ablaufen, die etwa Onkogene aktivieren können, so sie vorhanden sind, oder auch nicht. Der zelluläre Selbstreparaturprozess wird außer Kraft gesetzt.
    Dieses Modell kann mit beliebigen Daten gefüllt werden.
    Bewußtsein scheint eine Eigenschaft elektrischer Informationsverarbeitung zu sein. Je differenzierter und verschalteter das ZNS, desto umfassender Bewußtsein. Die ständige und vielfältige Rückkoppelung von Daten durch spezielle Hirnregionen in Form von Bewertung empfinden wir als Emotionen und im Weiteren als Gefühle.
    Die Placeboforschung, die bisher nur Beschreibungen liefert, könnte künftig Erklärungen liefern sowie Strategien für eine gezielte Therapie. Wir könnten also lernen, mit dem Kopf unseren Körper zu heilen, wie es fernöstliche Methoden mit ihren speziellen Narrativen aus spirituellen Riten tun.

  136. @Martin Holzherr

    „Big History meint die Geschichte des ganzen Universums vom Big Bang bis heute.“

    Das habe ich schon verstanden! Ändert aber nichts an der Tatsache, dass man „die Geschichte des ganzen Universums vom Big Bang bis heute“ nicht in ein einziges Buch quetschen kann ohne das die wissenschaftliche Qualität der einzelnen Disziplinen leidet.
    Siehe dazu auch: https://re-visionen.net/rezension-david-christian-big-history/

  137. @Mona: hier der erste Abschnitts des Wikipedia-Artikels Zu Big History:

    Big History ist eine akademische Disziplin, die die Geschichte vom Urknall bis zur Gegenwart untersucht. Big History widersetzt sich der Spezialisierung und sucht nach universellen Mustern oder Trends. Sie untersucht lange Zeiträume unter Verwendung eines multidisziplinären Ansatzes, der auf der Kombination zahlreicher natur- und geisteswissenschaftlicher Disziplinen beruht,[1][2][3][4][5] und erforscht die menschliche Existenz im Kontext dieses Gesamtbildes.[6] Sie integriert Studien über den Kosmos, die Erde, das Leben und die Menschheit unter Verwendung empirischer Beweise zur Erforschung von Ursache-Wirkungs-Beziehungen,[7][8] und wird an Universitäten[9] und Grund- und Sekundarschulen[10][11] gelehrt, wobei oft webbasierte interaktive Präsentationen verwendet werden.

    Yuval Harari hat in seinem Buch „Eine kurze Geschichte der Menschheit“ ebenfalls diesen Ansatz gewählt. Für ihn ist die Geschichte der Menschheit nicht eine Sequenz von Weltereignissen, sondern eine Umsetzung und Realisation von kognitivem Potential, das der Mensch erstmals besitzt. Die Geschichte, die er schreibt umfasst mindestens die letzten 70‘000 Jahre und sie zielt nicht darauf eine Chronologie zu verfassen, sondern eher darauf darzustellen, welche mentalen Fähigkeiten und Zustände dem Menschen ermöglichen, die Welt 🌎 zu beherrschen und die Menschen in Gemeinschaften zu binden.

  138. @Mona: „Big History“ ist vor allem ein bestimmter Blick auf die Welt und die Geschichte. Ziel: den Graben zwischen geisteswissenschaftlicher Betrachtung und naturwissenschaftlicher Betrachtung zuschütten und die Menschheitsgeschichte als Teil der Naturgeschichte begreifen.
    Big History will ein kohärentes Gesamtbild der gesamten seit dem Urknall verflossenen Zeit erstellen.

  139. Ich stehe vermutlich mit meiner Vermutung ziemlich alleine da, aber „Qualia“ könnte ganz allgemein als „Phänomen“ dann auftreten, wenn irgend ein Einfluss auf die Elektronenbahnen und die Kohäsionskräfte zwischen chemisch verknüpften Molekülen, z.B. Farben, organischen Molekülverbänden…., z.B. durch Kräfte, Licht, chemische Prozesse…. bewirkt wird.

    Es können natürlich nicht nur „Farbempfindungen“ entstehen, sondern z.B. auch Empfindungen für Lust und Schmerz.

    Dieses Phänomen kann direkt, z.b. von der Sensorik über das neuronale Netz, oder bei Farb- oder Forminformationen auch visuell, sozusagen mittels „Lichtwellen“ übertragen werden. Im Bereich von „Bewusstseinsstrukturen“, z.B. Netzhaut oder andere Zwischen- bzw. Endstrukturen im Gehirn, können Qualia wieder reaktiviert werden.

    Demnach wären „Qualia“ im Sinne von C. I. Lewis die „erkennbaren Charaktere des Gegebenen, die wiedererkannt werden können, und deshalb so etwas wie Universalien sind“.

    „Qualia“ entsteht sozusagen bereits am Ort der Entstehung, z.B. das „Rot“ entsteht direkt beim Feuerwehrauto, dann gibt es so etwas wie eine „Übertragungsstrecke“ (Übertragungssystem auch für Qualia) z.B. zum Auge, die übertragenden Signale gelangen somit zum Betrachter. Am Feuerwehrauto selbst entstehen zwar auch „Qualia“, z.B. die die Farbe „Rot“ generierenden Prozesse w.o. erwähnt, aber sie werden nicht direkt von einem neuronalen System ausgewertet.

    Farben dürften bereits erstmals an den farbempfindlichen Strukturen an der Netzhaut auswertbare „Empfindungsprozesse“, generieren. Durch den Input könnten, wie am Entstehungsort, aus den molekularen Elektronenbahnen Elektronen „herausgeschleudert“ (allenfalls auch zugeführt) werden und im neuronalen System, z.B. hinsichtlich der Lokalität und des Zusammenhangs mit anderen Farbpunkten, ausgewertet werden.

    Die von den sensorischen Organen (z.B. Auge) einlangende und weitergegebene Information (normalerweise „Muster“) wird digital, also mit einer Kombination von Signalen auf mehreren Leitungen („Leitungsvielfach“) abgebildet und mittels „Gatter“ ausgewertet. Die einzelnen Bildpunkte emergieren zunächst zu Bildern, nach der weiteren Verarbeitung der Muster in neuronalen Gatteranordnungen (Neuronen) zu einem Gesamteindruck.

    Von der Netzhaut (Auge) ausgehend, werten die (synaptisch) verknüpften Neuronen(gatter) die Signale aus und erkennen einzelne „Muster“. Sie werden in weiteren Neuronen(gattern) ausgewertet, so dass insgesamt aus der Gesamtheit der Inputmuster und der gespeicherten Muster, aus den Bewusstseinskomponenten, letztlich das individuelle „Bewusstsein“ über z.B. das in einer Landschaft stehende Feuerwehrauto entsteht. Über weitere „Hierarchien“ dürfte letztlich das gesteuerte „gesamte Bewusstsein“ mit den allgemein bekannten Empfindungen entstehen.

    Ein Neuron entspricht ungefähr einem („qualifizierten“) UND Gatter und triggert, wenn möglichst gleichzeitig ausreichend viele Impuls von den Dendriten einlangen.

    Dies ist eine ziemlich „freie Interpretation“ des Konzeptes von W. McCulloch.

    Eine Art von „verdrahtungsprogrammierte Gatter Struktur“ (mittels Synapsen) erkennt „Muster“.
    Das Konzept dieser Synapsenbildung (Wissensabbildung) hat E. Kandel erklärt.

  140. @Mona: „Big History“ ist mehr als ein Buch. Es ist ein Projekt, das Projekt nämlich die Geschichte des Universums, unseres Planeten, der Lebewesen und der Menschheit zu einem einheitlichen Bild zusammenzufassen. Wenn David Christian die Voraussetzungen fehlen, das alleine überzeugend darzustellen, dann braucht es halt andere, die das noch besser können. Es stimmt aber, dass wohl die meisten von dieser Aufgabe überfordert sind. Auch Yuval Hariri verlässt die Wissenschaft, wenn er die kognitiven Fähigkeiten des Menschen und ihre Auswirkungen auf die Menschheitsgeschichte beschreibt. Hariri hat aber einen grossen Fanclub unter den Bildungsbürgern und er ist ein guter Schreiber und Rhetoriker, dem es gelingt ein konsistentes Erklärungsbild zu suggerieren und es durch sein grosses Detailwissen glaubhaft erscheinen zu lassen.

  141. @Martin Holzherr

    Ich habe mir den Wikipedia-Artikel durchgelesen, auf den Sie sich beziehen, und fand da Sätze wie diesen: „Große Historiker weisen darauf hin, dass dies das Studium auf die letzten 5.000 Jahre beschränkt und die viel längere Zeit, in der Menschen auf der Erde existierten, vernachlässigt.“ Das ist doch blanker Unsinn, anscheinend hat man in den USA noch nie davon gehört, dass man auch Vor- und Frühgeschichte sowie Archäologie usw. studieren kann. Außerdem scheint bei „Big History“ Geschichte sowieso nur ein Teilaspekt zu sein, deshalb hätte man das Ganze lieber „Studium Generale“ oder so ähnlich nennen sollen. Wobei natürlich nichts gegen eine erweiterte Allgemeinbildung einzuwenden ist, es scheint jedoch ein Trend in der heutigen Zeit zu sein, überall reinzuschnuppern und alles abkürzen zu wollen, um am Ende zu behaupten man hätte alles verstanden. Kein Wunder, dass die Lücken dann mit Spekulationen oder Fantasie gefüllt werden und Face-News ohne Ende verbreitet werden. Dazu passt auch, was Sie am Ende selbst schreiben: „Es stimmt aber, dass wohl die meisten von dieser Aufgabe überfordert sind. Auch Yuval Hariri verlässt die Wissenschaft, wenn er die kognitiven Fähigkeiten des Menschen und ihre Auswirkungen auf die Menschheitsgeschichte beschreibt. Hariri hat aber einen grossen Fanclub unter den Bildungsbürgern und er ist ein guter Schreiber und Rhetoriker, dem es gelingt ein konsistentes Erklärungsbild zu suggerieren und es durch sein grosses Detailwissen glaubhaft erscheinen zu lassen.“

  142. Auch wenn alles Denken auf dieser Welt auf den gleichen zellulären / neuronalen Vorgängen basiert, so denkt doch jeder seine eigenen Gedanken.
    Eine wunderbare Vielfalt!

  143. @Stephan / 07.06.2020, 22:09 Uhr / @Andresen: Carrol und so

    »Denn dass beim Mathematiker aus den Axiomen und Regeln alles Weitere folgt, ist trivial.«

    Na eben nicht »alles Weitere«, da ist eigentlich nichts trivial.

    Die in einem (korrekten) axiomat. Kalkül formal beweisbaren Aussagen sind stets auch wahr, doch die Umkehrung gilt i.a. nicht. Die Aussagenlogik wäre ein Beispiel, wo das gilt, denn alle wahren Ausdrücke lassen sich hier auch aus den Axiomen herleiten; die Aussagenlogik ist daher ein vollständiger Kalkül. Der Begrff `Wahrheit’ ist in diesem Fall auf den Begriff `Beweisbarkeit’ reduzierbar, doch darf man halt nicht denken, das sei immer so.

    `Wahrheit’ und `Beweisbarkeit’ sind tatsächlich kategoriell verschiedene Begriffe, denn erstere betrifft die Semantik, letztere hingegen die Syntax eines solchen Kalküls. Und die Semantik einer Sprache lässt sich i.a. eben nicht auf ihre Syntax reduzieren.

    Insofern passiert ein allfälliger Kategorienfehler nicht erst dort, wo man sich einer mathematischen Sprache zur Formulierung einer Theorie bedient, von der dann beispielsweise behauptet wird, dass aus deren Postulaten — zumindest im Prinzip — eine deduktiv-nomologische Erklärung für die Entstehung des subjektiven Farbeindrucks von Rot zu erhalten sei. Die Grenzen jeglicher DN-Erklärbarkeit sind bereits in der formalen Grammatik der Sprache angelegt, derer man sich zur Erklärung bedient.

  144. @Mona: Beim Big-History Ansatz geht es nicht darum neues geschichtliches Wissen zu gewinnen, sondern darum eine neue Sicht auf die Vergangenheit/Gegenwart und die zeitliche Entwicklung zu etablieren. In dieser neuen Sicht hat nicht nur die Menschheit eine Entwicklung durchgemacht, sondern auch die belebte Natur, unser Planet 🌎 und das gesamte Universum. Das stellt auch einen Zusammenhang mit der Philosophie her, denn auch in der Philosophie geht es weniger um den Erwerb neuen, gesicherten Wissens, sondern um eine bestimmte Sicht auf unsere Welt. Der Stoiker hat eine andere Sicht als der Epikureer, der ontologische Materialist eine andere als der, welcher dem Idealismus frönt.
    „Big History“ benötigt die Naturwissenschaften um die vormenschliche Geschichte einzubringen und sie will auch die menschliche Geschichte soweit als möglich naturwissenschaftlich erklären.

  145. @ Wolfgang Stegemann 08.06.2020, 17:53 Uhr

    Ihre Sichtweise scheint sehr interessant.

    Für die Entstehung von Bewusstsein dürften elektrische Signale allein nicht reichen, obwohl die elektrische Signal- (Informationsverarbeitung) Voraussetzung für ein bewertendes Bewusstsein sein dürfte.

    Wichtig für Bewusstsein, sollte die Empfindungsfähigkeit sein und dafür könnte es auf chemisch physikalische, womöglich auch Quanteneffekte ankommen.

    Dazu habe ich mir im laufe meines schon längeren Lebens 2 möglicherweise entscheidende, grundlegende, aber eher einfache Gedanken „aufgerissen“.

    Von meinem ersten Chemielehrer die Idee, dass die Elektronen in der äußersten Hülle (wegen der Kraftwirkungen) für die chemischen Bindungen sorgen und die können durch vielfältige äußere Einwirkungen verändert werden.

    Später, ich weiß nicht mehr von welchem Lehrer, habe ich erfahren, dass er eher nebenbei meinte, dass dynamische Resonanzeffekte, wie sehr häufig in der Technik, auch Kraftwirkungen auf molekularer Ebene immens verstärken können. Dies auch im Zusammenhang mit biologischen Systemen bedeutend sein könnte, womöglich auch beim Bewusstsein. Die „Kreativen“ unter uns hatten natürlich gleich an ganz „besondere Dynamiken“ im Zusammenhang mit den Zwischenmenschlichen Beziehungen gedacht, die durchaus „starke Empfindungen“ im Bewusstsein bewirken können.

    Da ich weder Chemiker noch Physiker bin, kann ich nicht sagen wie weit Sichtweisen wie vor rund 50 Jahren, heute noch halbwegs aktuell sind.

    Andererseits würden diese „uralten“ Gedanken im Zusammenhang mit Empfindungen, recht gut zu den Konzepten die die Musterverarbeitung im Gehirn betreffen passen. Durch äußere Einflüsse frei werdende Elektronen (oder „Löcher“ im Sinne der Halbleiterphysik), könnten in neuronalen Systemen bestens ausgewertet werden, was grundlegend auch für die Auswertung der „Bewusstseinskomponenten“ ist.

    Neuronen „können“ („elektrisch“ gesehen) nichts anderes, als halbwegs gute („qualifizierte“) UND Gatter und ausreichend komplex „kaskadierbar“ zu sein. Wären es UND Gatter im Sinne der Prädikatenlogik, wären sie völlig untauglich, weil das Information verarbeitende System an den kleinsten Widersprüchen, die es im Leben nun einmal gibt, scheitern würde. Es scheint recht robust bei der Bewältigung von Widersprüchen, allerdings ist dies auch der Grund warum 3 Juristen 4 verschiedene Rechtsmeinungen haben können.

  146. @Martin Holzherr

    Soweit ich gelesen habe ist das Projekt „Big History“ bestrebt, „die Geschichte des Universums, unseres Planeten, der Lebewesen und der Menschheit zu einem einheitlichen Bild zusammenzufassen“, um darin dann mithilfe von acht „Hauptschwellen“ eine „zunehmende Komplexität“ auszumachen. Das Modell dürfte Gläubigen bekannt vorkommen, weil es irgendwie an einen Schöpfungsmythos erinnert.

  147. @Mona: sie haben es erfasst: immer wieder wird nach dem grossen Letzten, der alles erklärenden Erklärung gesucht. Das tun sogar so säkulare Wissenschaftler wie die Physiker, welche mit ihrem Unterfangen der grossen Vereinheitlichung die ganze Welt auf ein einziges Urprinzip zurückführen wollen (und die dafür die grösste Maschine der Welt, den Large Hadron Collider) gebaut haben und das tut nun auch das Projekt „Big History“ welches die ganze Entwicklung des Universums bis zum Menschen als Überschreiten von Schwellen beschreiben ( es fragt sich nur ob es Schwellen vor dem Paradies oder vor der Selbstvernichtung sind). „Big History“ ist so gesehen vielleicht nur eine Neuauflage von Teilhard de Chardin’s Versuch einer Synthese von naturwissenschaftlicher Evolutionstheorie und christlicher Heilsgeschichte – nur diesmal viel säkularer ohne Gott.

  148. ZU St. Schleim:
    Also über die neurologisch Beschreibung des Überfalls eines Bankräubers auf eine Sparkasse muss ich doch etwas lächeln. Diese- von ihnen so beschriebene Szene- benötigt einen kausalen Zusammenhang- mit dem Gesellschaftssystem in dem es stattfindet. Räuber(Person) muss also über eine Pistole verfügen können bzw. muss wissen, das er dafür wenig bestraft wird bzw. entfliehen kann. Letztere Bedingungen waren z. Bsp. in der DDR nicht gegeben, was Banküberfälle in der Regel “sinnlos” machte. In anderen heutigen Systemen auf dieser Erde würde er dafür mit harten abschreckenden Strafen gezüchtigt. Dieses neurologische System-was so von ihnen beschrieben wurde- ist also sekundär nur ein ausführendes Organ eines tieferliegenden Systems, was diese Person erst zum Räuber macht-denn “Räuber ” und andere Ganoven werden nicht als solche geboren. Es sind also die Werte des Gesellschaftssystems, die solche neurologischen Muster ermöglichen ,prägen, die solche Egos produzieren. Auf Grund dieser Prägung findet dann auch eine entsprechende neurologische Verschaltung in dieser grauen Masse(Gehirn) statt.

  149. Teilhard de Chardin und die Überwindung des Geist-Materie-Dualismus
    Teilhard de Chardin war ein französischer Jesuit, der den Versuch einer Synthese von naturwissenschaftlicher Evolutionstheorie und christlicher Heilsgeschichte wagte.
    Bei Teilhard befindet sich das ganze Universum auf einem Evolutionspfad. Dieser führt vom Urstoff zur Biogenese und Biosphäre und durch das Auftreten des Geistes (der Noogenese) zur Noosphäre (Sphäre des Geistigen). Die Noosphäre ist zugleich das Ende der Kosmogenese, also das Omega auf das alles zustrebt.
    Fazit: Teilhard de Chardin überwindet in seiner Weltsicht den Geist-Materie/Dualismus indem er den Geist und die Sphäre des hyperpersonalen Geistigen (der Noosphäre) als Evolutionsstufe innerhalb der Kosmogenese betrachtet. Bei Teilhard entspringt zwar der Geist der materiellen Welt, aber dies tut er nicht zufällig sondern als Teil einer progressiven Geistwerdung des Kosmos

  150. @Martin Holzherr 10.06. 14:45

    „Teilhard de Chardin überwindet in seiner Weltsicht den Geist-Materie/Dualismus indem er den Geist und die Sphäre des hyperpersonalen Geistigen (der Noosphäre) als Evolutionsstufe innerhalb der Kosmogenese betrachtet. Bei Teilhard entspringt zwar der Geist der materiellen Welt, aber dies tut er nicht zufällig sondern als Teil einer progressiven Geistwerdung des Kosmos“

    Hört sich doch nicht schlecht an, finde ich. Wobei ich Geist auch vielen Tieren zusprechen würde, und dass Geist im Kosmos wohl auch von Anfang an dabei sein müsste. Wie soll er sonst in der Evolution sich selbst angebahnt haben. Zusätzlich in die Welt kamen nur die biologischen Lebewesen, die vom grundsätzlichem Angebot des ohnehin vorhandenen Weltengeistes Gebrauch machen konnten, und ihn in ihren Mikrokosmos aufnehmen konnten.

    Was jetzt Geist bzw. Bewusstsein ist, dass ist nochmal eine ganz andere Frage.

  151. @Tobias Jeckenburger (Zitat) :

    dass Geist im Kosmos wohl auch von Anfang an dabei sein müsste. Wie soll er sonst in der Evolution sich selbst angebahnt haben.

    Da könnten sie ebenso gut sagen: dass Leben im Kosmos wohl von Anfang an dabei sein müsste. Wie soll er sonst in der Evolution sich selbst angebahnt haben.
    Zudem: „Geist“ ist ein von Menschen geschaffenes Abstraktum. Um es konkret zu machen muss man vom Geist/von Gedanken eines Menschen wie Tobias Jeckenburger sprechen oder von den Gedanken, die gerade ein Eisbär hat, wenn er eine Robbe erspäht.

    Und nicht vergessen: Teilhard de Chardin ist ein Theologe und seine Sicht ist eine theologische in der die Welt auf einen finalen Zustand zustrebt.

    Die Physik und die Evolution lehrt uns etwas anderes: Es gelten jederzeit die gleichen Gesetze, nur befindet sich die Welt oder befinden sich bestimmte Teile der Welt in einem anderen Zustand wo andere Dinge möglich werden. Seit es Lebewesen gibt oder sogar schon vorher, gab und gibt es auch die Evolution. Viren, Bakterien, ja die gesamte belebte Welt evolviert heute genau so wie zu Beginn des Lebens. Nur macht es (vor allem für uns Menschen) einen Unterschied aus, ob ein Virus evolviert oder ob das Gehirn evolviert und aus Protomenschen den Neandertaler oder Homo sapiens macht.

    Was die Physik betrifft: schon die Physik des Kleinsten, die Quantenphysik also ermöglicht die Bildung von neuen Produkten mit völlig neuen Eigenschaften, mit Eigenschaften, die sich nur schwer durch die Eigenschaften der Ausgangsstoffe erklären lassen. Wasser beispielsweise ist völlig verschieden von Sauerstoff oder Wasserstoff, besteht aber nur aus Wasserstoff- und Sauerstoffatomen.
    Auch unser Geist ist etwas Neues, aber etwas, das aus bereits vorhandenen Elementen zusammengeschustert wurde. Nur tauchen dabei neue Phänomene auf, die ebenso wenig oder ebenso schwer mit den Bestandteilen erklärbar sind wie es Wasser in Bezug auf seine Ausgangsstoffe ist,

  152. @Tobias Jeckenburger (Ergänzung): eine Evolution zum Höheren gibt es nur aus unserer menschlichen Sicht. Weil wir uns selbst an die Spitze der Evolution gesetzt haben. Aber eine solche Hierarchie gibt es nicht ohne willkürliche Festlegung und ohne recht willkürliche Bewertung. Das gilt ganz generell. Sie können auch Sterne, Galaxien und andere kosmische Objekte nicht einfach als besser oder höher einstufen. Warum etwa soll unsere Sonne besser sein als ein roter Zwerg ? Etwa, weil unsere Sonne lebensfreundlichere Bedingungen schafft als ein roter Zwerg wie es Alpha Centauri ist (rote Zwerge haben regelmässig massive Strahlungsausbrüche, die wohl alles Leben auslöschen). Schon wieder ist eine solche Bewertung anthropozentrisch.

  153. @ Martin Holzherr und alle die über „Information“ im Zusammenhang mit dem Dualismus Stellung nehmen wollen.

    Zitat aus Wikipedia: „….Information ist die Teilmenge von Wissen, die von einer bestimmten Person oder Gruppe in einer konkreten Situation benötigt wird und häufig nicht explizit vorhanden ist…“

    Das ist schon eine recht weitgehender Versuch Teilaspekte von „Information“ zu beschreiben.

    Mir ist der Versuch aber noch weitaus zu wenig weitgehend. Ganz einfach, nicht nur Personen, auch Tiere möchten z.B. wissen wo es Futter gibt, Computer werden „angehalten“ neue Stellen von Pi zu berechnen, oder nach anderen Wissen (Mustern) zu scannen. Muster die noch kein Mensch kennt, die aber bedeuten können, dass man in einer Handelsfirma z.B. 10% mehr Umsatz machen kann.

    Der Sachverhalt (nachdem gesucht wird) kann (muss aber
    noch
    nicht einmal) vorhanden sein (er könnte z.B. in der Zukunft liegen).

    Es reicht, wenn „irgend etwas“ keine „Hardware“ oder kein „Prozess“ ist. Beide, auch Information selbst, können mit Information beschrieben werden, Prozesse zusätzlich von Information „gesteuert“ werden.

    „Wissen“ benötigt einen „Träger“, Information nicht, sie ist implizit immer vorhanden.

  154. @Elektroniker (Zitat):

    Wissen“ benötigt einen „Träger“, Information nicht, sie ist implizit immer vorhanden.

    Damit widersprechen sie dem Wikipedia-Zitat, dass sie selbst als Einleitung gewählt haben. Es, das Wikipedia-Zitat, bedeutet: Ohne Interpretation gibt es keine Information. Das galt schon immer und gilt auch heute bei so aktuellen Themen wie der künstlichen Intelligenz. Dort verbessert man die Erkennungsleistung durch einen „reward“, eine Belohnung, wenn man ein bestimmtes Ziel erreicht oder umgekehrt werden Fehler beseitigt indem man Korrekturen an den „Perzeptoren“ vornimmt, die falsche Beiträge zur Erkennungsleistung beitragen. Mit andern Worten: was Information ist wird durch Erreichen oder Nicht-Erreichen eines Erkennungsziels bestimmt und ohne solch ein Ziel ist es schwierig zu sagen was Information ist. Vielleicht gibt es auch dann Information in Form von Struktur. Diese Struktur wird bei unüberwachtem Lernen in der künstlichen Intelligenz versucht zu erkennen. Doch reine Strukturerkennung genügt nicht um die Welt zu verstehen. Denn was sinnvolle und bedeutungsvolle Struktur ist lässt sich nicht objektiv bestimmen.

    Für Lebewesen ist Information was ihnen beim Leben und Überleben hilft.

  155. Martin Holzherr,
    eine gute Aussage “Für Lebewesen ist Information was ihnen beim Leben und Überleben hilft.”

    Und damit sind wir wieder bei Gott und der Religion. Das Volk der Juden hat überlebt, weil es an Gott glaubt.
    Und jetzt wage ich mal die Aussage: “Gott ist Information”.

  156. @ Martin Holzherr 10.06.2020, 23:41 Uhr

    Das ganze (nachstehende) Zitat scheint ein Widerspruch, wenn man Wissen gewinnen will, aus Information, die explizit nicht vorhanden ist. Aus „nichts“ kann man eben „nichts“ gewinnen.

    Zitat aus Wikipedia: „….Information ist die Teilmenge von Wissen, die von einer bestimmten Person oder Gruppe in einer konkreten Situation benötigt wird und häufig nicht explizit vorhanden ist…“

    Die „Leute“ von Wikipedia sind sozusagen meine „Zeugen“, die meinen dass es doch (auch) „geht“.

    Allerspätestens dann, wenn man etwas vorher (noch) nicht „vorhandenes“ z.B. als eine bestimmte Variable oder einem Objekt, in einem Computerprogramm „deklariert“, dann existiert es auch, der Computer kann es verknüpfen, verarbeiten….

    Information sollte natürlich sinnvoll sein und Nutzen bringen. Muss sie aber nicht.

    Nachrichtentechniker/Informatiker bekommen z.B. von anderen Fachgebieten Datenträger mit aufgezeichneten Signalen und sollen ermitteln ob die Signale „sinnloses Rauschen“, oder „sinnvolle Information“ abbilden.

    Für die Auswerter ist es in jedem Fall Information, auch wenn ein sinnvoll codierter informeller Inhalt nicht nachweisbar ist, oder selbst dann wenn jemand absichtlich „Rauschen“ produziert hat, die (zufällig) als sinnvolle Information „erkannt“ wird.

    Früher wurde z.B. von „Hardcore Monisten“ und von den „Steuereintreibern“ (die immer Betrug wittern, was für die Betroffenen noch schlimmer war) die reale Existenz von Software (im Sinne der Informatik) bestritten.

    Software kann nur gemeinsam mit einer „Maschine“ existieren und nur mit der Maschine verknüpft „verkauft“ werden, niemals getrennt, weil dies eben unmöglich und daher Betrug wäre. So wie es keinen „Körper“ mit einer abtrennbaren „Seele“ geben könne.

    Der Ankauf von neuer Software war sozusagen „Privatvergnügen“ der Unternehmer, konnte zeitweise nicht von der Steuer abgesetzt werden. Die Produzenten von Software galten als Betrüger, wie die Pfaffen, weil sie etwas verkaufen wollten was es unmöglich geben kann.

    Steuerbehörden wollten als „Wert von Software“ nur den lächerlich geringen Wert des Datenträgers akzeptieren. Natürlich hat das Hardwarefirmen gefallen, weil ihnen externe Softwareerzeuger nicht das Geschäft verderben konnte und die Kunden fest abgezockt werden konnten.

    Bei diesem absurden „Background“ sollten Sie sich nicht wundern, warum ich mich noch immer so für die Eigenständigkeit von Information einsetze, und auch dafür, dass sie sozusagen aus dem Nichts „geschaffen“ werden kann.

    Die schlauen Finanzbehörden haben ehemals auch nach den Rohstoffen gefragt, die es außer den Algorithmen auch nicht gab. Diese Algorithmen waren auch Information und die konnte es auch nicht geben, für die war alles nur Schwindel und Betrug.

  157. @ H.Wied 11.06.2020, 09:02 Uhr

    Zitat: „Und jetzt wage ich mal die Aussage: “Gott ist Information”“.

    Nicht nur Information.

    „Gott“ ist letztlich „Bezeichner“ für alle „wirklichen“ Gesetzmäßigkeiten (Information), alle Prozesse, jegliche Materie die im Zusammenhang mit der Entstehung und mit dem „Lauf der Welt“ stehen.

    Weil man nicht alles versteht, gilt „Gott“ als transzendent.

    Diese Aussage habe ich von einem Theologen vor einigen Jahrzehnten im Radio ungefähr so gehört und sie ist praktisch eine Tautologie, die immer „Wahr“ ist.

    Ob einem dies reicht, oder man genaueres wissen möchte, ist eine andere Frage.

  158. Elektroniker,
    Menschen, die an Gott glauben sind keine Logiker. Wir lassen dabei außer acht, dass Menschen Gotteserfahrungen machen und dieser transzendente Gott dann zu einem persönlichen Gott wird, mit dem man sprechen kann.

  159. @Martin Holzherr 10.06. 17:54 / 18:10

    Ich denke schon, dass der Mensch eine neue Kategorie im Kosmos aufmacht. Als Kulturwesen, dass u.a. Wissenschaft betreiben kann, und (hoffentlich) dabei ist, eine weltweite funktionierende Organisation aufzubauen, verursachen wir gerade ein neues Zeitalter, das Anthropozän. Neben den aktuellen Zerstörungen, die ich als Kinderkrankheiten einstufen würde, können wir die Natur in einen Naturgarten verwandeln, und haben das Potential die Galaxis zu besiedeln. Und wir können uns unseres eigenen Geistes bewusst sein.

    Inwieweit wir unser eigenes Bewusstsein darüber hinaus mit einer Teilhabe am kosmischen Geist verbinden, ist sicher auch eine Frage der persönlichen Erfahrung. Spirituelle Erfahrungen legen dies näher, und Theologen wie Teilhard de Chardin interessiert das Thema auch.

    Biologische Evolution ist eine Entwicklung von Biochemie, und die chemischen Grundlagen gab es auch schon in den Staubwolken, aus denen sich überhaupt erst die Sterne und Planeten gebildet haben. So stelle ich mir das auch mit dem Geist als eigene Kategorie vor: Es gab ihn schon immer, aber erst in uns Menschen wird er richtig aktiv, und unterstützt uns in unserem Leben und bei den großen Projekten Naturgarten und Besiedlung der Galaxis.

    Aber auch die Dinosaurier hatten wohl eine eigene psychische Existenz, die für sich schon einiges an Sinn mit sich brachte. Deren persönliches Leben war vermutlich auch schon eine Art Spielwiese des Geistes. Wer selber Haustiere hat und sich mit diesen intensiv beschäftigt, der merkt dann meistens auch, das deren geistige Präsenz und Existenz ein relevanter Teil des (Geistes-)Lebens ist. Wir sind nicht so viel mehr als das, nur eben größer und vor allem in der Lage, Kultur zu praktizieren. Die Kulturfähigkeit ist unsere eigentliche Innovation, persönlichen Geist in einer lebendigen Innenwelt, den gibts auch bei Tieren schon.

    Ich glaube nicht, das nur der Mensch Sinn macht, und dass wir der Omegapunkt der Evolution sein müssen. Aber wir spielen vermutlich im Kosmos eine wichtige Rolle, und wenn es nur die Verbreitung des Lebens durch Raumfahrt betrifft. Darüber hinaus kann die Faszination unseres eigenes Leben für uns selbst erheblich sein, so wie es im Leben von Tieren auch schon zu beobachten ist.

  160. @Tobias Jeckenburger
    Meinen sie mit Kosmos wirklich den ganzen Kosmos?
    Auf der Erde ist der Mensch sicherlich etwas besonders, aber im gesamten Kosmos? Ich denke da wäre ein wenig Demut angemessen.
    Es gibt keinen Grund anzunehmen, dass es so etwas wie Mensch nur auf der Erde geben könnte. Dafür ist der Kosmos viel zu groß etc.

  161. Tobias Jeckenburger,
    wir sind im Weltall nicht nur durch die großen Entfernungen von anderen Lebensformen entfernt, sondern auch durch die Zeit.
    Die Sterne die wir sehen, die gibt es oft gar nicht mehr, und die Lebensformen auch nicht mehr.
    Von einem gemeinsamen Universum zu sprechen, das ist aus diesem Grunde nicht möglich.

  162. Zu H. Wied:
    Widerspruch. Dass, was T. Jeckenburger hier meint mit “kosmischen Bewusstsein ” ist zwar Marke Eigenbau aber, wenn wir es aus hinduistischer Sicht (Brahman) betrachten : ohne Anfang und ohne Ende. Der Kosmos ist sozusagen zeitlos wie das “Bewusstsein” ,was ihn im Gewahrsein betrachtet zeitlos ist. Im Gegensatz zum Christentum gibt es wohl keinen Schöpfer, keinen Anfang. Der Atman geht sozusagen auf in das Brahman, dem höchsten SELBST. Das “kosmische Bewusstsein ” ist für mich also mehr ein religiöser Begriff ohne wissenschaftlichen Anspruch und damit zum Gebrauch oder Missbrauch freigegeben für alle Gurus dieser Welt .
    Ansonsten habe ich, ohne durch die Blume zu sprechen, noch eine Frage hinsichtlich ihrer Anmerkung “Das Volk der Juden hat überlebt”. Was wäre so schlimm gewesen, wenn es sich nicht mit anderen Stämmen und Völkern in der Geschichte vermischt hätte wie die Arier mit den Indern, die Germanen mit den Kelten, die Angelsachsen mit den Briten bzw. die Vermischung der Völker aus dem alten Europa in den USA ? Warum wollten die Juden diesen humanistischen Weg der Völkerverschmelzungen nicht gehen ?

  163. Das Leib-Seele Problem ließe sich auflösen, wenn man den kartesischen Satz umdreht in: Ich bin, also denke ich.

  164. @Einer / H.Wied

    Was die Besiedlung angeht, da dachte ich sicher nicht an den ganzen Kosmos, sondern an unsere Galaxie, die Milchstraße. Und auch hier sind wir wohl kaum die einzigen intelligenten Kulturwesen, da könnten sogar jede Menge dabei sein, die uns hunderte von Millionen Jahren voraus sind. Da wäre dann echt Demut angesagt.

    Aber wer weiß, vielleicht sind wir ja in der Milchstraße die Ersten? Das wäre ein Grund dafür, dass sich bei uns noch keiner vorgestellt hat.

    Die Entfernungen innerhalb der Milchstraße sind vergleichsweise gering, bis zum Zentrum sind es gerade mal ca. 25.000 Lichtjahre. Das ist auch ein Zeitraum, der bezogen auf geologische und astronomische Zeiträume ziemlich kurz ist. Fast alle Sterne aus der Milchstraße, die wir sehen können, existieren auch heute noch. Wenn es um andere Galaxien geht, da geht es dann erst in die Millionen Lichtjahre und auch in die Milliarden.

    Unsere Erde hat nun schon 4,5 Milliarden Jahre astronomisch stabile Zeiten erlebt, nur deswegen gibt es hier Mehrzeller und letztlich den Menschen. Einzeller sind recht schnell nach ein paar hundert Millionen Jahren nach der Entstehung der Erde entstanden, aber komplexeres Leben erst nach ca. 3,5 Milliarden Jahren.

    Vermutlich haben die allermeisten prinzipiell geeigneten Planeten nicht eine so lange stabile Zeit, dass sich da Mehrzeller entwickeln konnten. Das wären wohl die Kandidaten, die einen Export unserer Biosphäre gebrauchen könnten. Und falls sich Leben überhaupt doch nicht so schnell wie bei uns bildet, könnten wir auch sehr viele Planeten besiedeln, wo es zwar lebensfreundliche Bedingungen gibt, das Leben dort aber noch gar nicht entstanden ist.

    Selbst wenn es noch weitere 10.000 Zivilisationen in der Milchstraße gibt, kämen bei schätzungsweise 100 Millionen besiedelbaren Planeten immer noch 10.000 Besiedlungskandidaten pro Zivilisation, falls man sich das hier friedlich aufteilt.

    Vor kurzem habe ich einen Beitrag gelesen, dass man gar nicht so weit fliegen muss, um die Galaxie zu besiedeln. Die Sterne kreisen alle um das Zentrum unser Galaxie, fliegen dabei aber auch recht wild durcheinander. Man muss also mit den Raumschiffen nicht die ganze Galaxie durchqueren. Wenn man sich vielleicht 10 Millionen Jahre Zeit damit lässt, kommen jede Menge Sterne mitsamt ihrer Planeten irgendwann auf vielleicht nur ein paar Lichtjahre Abstand an unserer Sonne vorbei. Wenn wir diese dann anfliegen und Besiedeln, nehmen die uns dann auf ihrer weiteren Reise quer durch die Galaxie mit, und bilden eine Basis für unsere weitere Verbreitung.

    Fazit: Da gibt es ganz konkret was zu besiedeln, und wir haben das Zeugs dazu, das auch zu realisieren. So hoch sind wir entwickelt. Wir haben zwar über hunderttausende von Jahren nur in Kleingruppen gelebt, aber uns dabei weiterentwickelt. Und dann mit dem Aufkommen des Ackerbaus haben wir es gewagt, immer größere Gesellschaften zu organisieren. Inzwischen sind wir dabei, eine gemeinsame Weltkultur hinzubekommen. Wenn das gelingt, steht der Raumfahrt nichts Prinzipielles mehr im Wege, würde ich sagen.

  165. Tobias Jeckenburger,
    die Idee von der Besiedelung des Universums ist gut. Aber…..und hier kommt das große Aber……gesetzt den Fall …..unsere Gene, Keime, Bakterien, Viren sind für die anderen Biowelten tödlich. Das Corona Virus könnte ja auch so ein Wanderer zwischen den Welten werden.
    Bleiben wir lieber zu Hause !

    Querdenker,
    Unsere Erde ist seit etwa 3 Milliarden Jahren belebt.
    Seit etwa 3 Tausend Jahren lassen die Juden als Volk von sich hören.
    Das ist ein Millionstel der biologischen Zeitrechnung.
    Warten wir doch ab, was da noch kommt.

  166. @Martin Holzherr

    „Big History“ ist so gesehen vielleicht nur eine Neuauflage von Teilhard de Chardin’s Versuch einer Synthese von naturwissenschaftlicher Evolutionstheorie und christlicher Heilsgeschichte – nur diesmal viel säkularer ohne Gott.

    Der Gedanke kam mir tatsächlich auch. Soweit ich irgendwo gelesen habe lässt man die Idee eines Schöpfergottes beiseite, weil die modernen Naturwissenschaften keinen direkten Beweis für einen Gott finden konnten.

    Bei Teilhard befindet sich das ganze Universum auf einem Evolutionspfad. Dieser führt vom Urstoff zur Biogenese und Biosphäre und durch das Auftreten des Geistes (der Noogenese) zur Noosphäre (Sphäre des Geistigen). Die Noosphäre ist zugleich das Ende der Kosmogenese, also das Omega auf das alles zustrebt.

    Bei dieser Sichtweise geht es nicht ohne Gott. Teilhard de Chardin zitiert ja auch ausdrücklich die Offenbarung des Johannes (Offb 21,6), wo Gott spricht: „Ich bin das Alpha und das Omega, der Erste und der Letzte, der Anfang und das Ende.“ Auf Wikipedia heißt es dazu: „Teilhard de Chardin sieht Leben und Kosmos in einer von Gott bewirkten kreativen Bewegung, die noch nicht an ihr Ziel gelangt ist. Kennzeichen dieser Bewegung ist die ständige Zunahme von Organisiertheit und organischer Einheit. Das Streben in diese Richtung, also der Motor der Evolution, ist für Teilhard die Liebe. Diese Liebe, die das letzte Ziel, „die organische Einheit alles Seienden, bereits handelnd und leidend vorwegnimmt“, war für Teilhard im Herzen eines Menschen vollkommen verwirklicht: in Jesus Christus. So nennt er Christus mit einem biblischen Hoheitstitel (Offb. 21,6) das Omega oder den Punkt Omega, das heißt Ziel, Richtung und Motor der Evolution.“ Somit ist für Teilhard de Chardin der Mensch nicht die Krone der Schöpfung, da die nächste Stufe der Evolution noch nicht erreicht ist. Um den Dualismus von Geist und Materie zu überwinden musste Teilhard de Chardin von einer stufenweisen Geistwerdung des Kosmos ausgehen, ansonsten hätte er Glaube und Wissenschaft kaum vereinen können.

  167. @H.Wied Konfrontation von Biosphären

    Wenn eine Biosphäre eines Exoplaneten unabhängig entstanden ist, wird sie in der Tat mit unserer recht unverträglich sein. Wenn es da Pflanzen gäbe, könnten Tiere von der Erde die wohl nicht essen können. Die DNA-Codes und die speziellen Proteine würden unverdaulich sein. Planeten, auf denen es schon Mehrzeller gibt, wären also zur Besiedlung komplett ungeeignet. Aber diese Planeten sind wohl vergleichsweise sehr selten, bei uns auf der Erde hat es ja ca. 3 Milliarden Jahre gedauert, bis sich aus Einzellern komplexe Mehrzeller entwickelten.

    Die meisten Exoplaneten mit Leben hätten also nur Mikroben. Wenn wir die jetzt mit unserer exportierten Biosphäre konfrontieren, würden voraussichtlich die jeweiligen Biosphären zunächst parallel koexistieren, bis sich eine von beiden durchsetzt und sich die andere in Nischen zurückzieht. Dass daraus dann Krankheitserreger entstehen, ist nicht zu erwarten, eine fremde Biologie ist dafür wohl viel zu unspezifisch. Aber das wird man dann ja sehen.

    Sollte sich auf dem Mars, dem Jupitermond Europa oder dem Saturnmond Enceladus unabhängig von uns hier eigenständig Leben gebildet haben, könnten wir ja sogar die Kompatibilität schon mal testen, noch lange bevor wir zu den Sternen aufbrechen werden. Solche Experimente sollte man dann aber sicherheitshalber auf Raumstationen im Orbit um die Sonne machen, nicht auf der Erde.

    Was auf jeden Fall bleibt, das sind Exoplaneten, auf denen zwar lebensfreundliche Bedingungen herrschen, aber die Zeit zur selbstständigen Entstehung von Leben noch nicht ausreichte.

    Wie auch bei den Planeten mit nur Mikroben, müsste man sowieso in 3 Schritten besiedeln. Zunächst müsste man entsprechend effektive Algen aussetzen, die einen gewissen Sauerstoffanteil in den Meeren und der Atmosphäre produzieren. Erst wenn der bei vielleicht ein paar Prozent liegt, kann man dann mit der ganzen Palette von Mehrzellern nachziehen. Und erst ab 10% Sauerstoffgehalt wären die Bedingungen für Menschen und die meisten größeren Tiere erträglich.

    So schwierig das auch ist, und so fürchterlich teuer, bewohnbare Planeten sind so superschön zu bewohnen, da lohnt sich jeder Aufwand. Im Vergleich zu einem eigenständigem Leben auf Raumstationen im Orbit um die Erde oder die Sonne sind bewohnbare Planeten die Superjuwelen im Kosmos.

    Und ganz nebenbei: ich frage mich schon länger, was wir Menschen auf der Erde dann machen sollen, wenn wir mal eine wirklich gemeinsame Kultur, eine friedliche weltweite Organisation, eine komplett naturverträgliche Lebensweise sowie entsprechende KI haben, die die meiste Arbeit macht. Schon vor lauter Langeweile fällt mir hier nur entsprechend richtig aufwändige astronomische Forschung und letztlich die Vorbereitung von Reisen zu den Sternen ein.

  168. @Mona

    „Somit ist für Teilhard de Chardin der Mensch nicht die Krone der Schöpfung, da die nächste Stufe der Evolution noch nicht erreicht ist. Um den Dualismus von Geist und Materie zu überwinden musste Teilhard de Chardin von einer stufenweisen Geistwerdung des Kosmos ausgehen, ansonsten hätte er Glaube und Wissenschaft kaum vereinen können.“

    Eine Weiterentwicklung des Menschen? Wohin? Hier fehlt doch nur Kultur. Der Mensch hat nach hunderttausenden von Jahren Evolution in Kleingruppen als Jäger und Sammler doch alles drauf, was er braucht.

    Die Geistwerdung des Menschen im spirituellen Sinn ist uns selbst doch schon möglich. Hier braucht es doch nur die entsprechenden spirituellen Erlebnisse, Zeit sich damit zu beschäftigen und einen kulturellen Rahmen, dass man mit dem Spirituellen erstens mit klar kommt ohne dabei verrückt zu werden, und zweitens eine Gesellschaft, die einen weder für verrückt noch für geistig abtrünnig erklärt.

    In modernen Gesellschaften mit wirklicher Religionsfreiheit und einer Psychiatrie, die nur Leute behandeln will, die mit ihrem Leben wirklich nicht alleine klar kommen, ist das doch kein Problem.

    In diesem Sinne mag Jesus Christus als Vorbild helfen, aber inspirieren genügt, die biologische Grundlage scheint doch im wesentlichen gegeben zu sein. Hier fehlt doch nur Engagement und Kultur.

  169. @Querdenker 11.06. 20:52

    „Warum wollten die Juden diesen humanistischen Weg der Völkerverschmelzungen nicht gehen?“

    Ja haben sie doch. Da sind immer wieder welche zum Judentum konvertiert und umgekehrt. Das Ergebnis war doch gerade, dass die europäischen Juden, die hier ermordet wurden, dem Durchschnittseuropäer wesentlich ähnlich waren, und kaum wie Orientale aussahen. Genauso haben die Katholiken ständig mit Eintritten und Austritten zu tun, und sind doch seit 1700 Jahren sogar die selbe Amtskirche geblieben.

    Das man den Juden immer wieder Schwierigkeiten gemacht hat, und sie sogar bezichtigt hat, eine seperate Menschenrasse zu sein, liegt doch daran, das sie die meiste Zeit als religiöse Minderheit koexistieren mussten. Auch Katholiken haben in der jüngeren Geschichte schon Schwierigkeiten bekommen, wenn sie mal auch nur in der Minderheit waren. Im Sozialismus, vor allem auch in China, mussten sie vorsichtig sein, und auch aktuell in streng muslimischen Ländern. Genauso werden Katholiken in den USA zwar nicht verfolgt, aber doch von vielen Evangelikalen etc. doch nicht gerade hoch angesehen.

    Genau so wie Katholiken, die nicht ihren Glauben aufgegeben haben, wenn sie irgendwo in die Minderheit gerieten, haben auch die Juden ihren Glauben wenn immer irgendmöglich beibehalten. Das ist doch recht normal, würde ich sagen. Und dass Juden dem Humanismus entgegenstehen, das wäre mir auch neu.

  170. @Tobias Jeckenburger

    „Eine Weiterentwicklung des Menschen? Wohin? Hier fehlt doch nur Kultur. Der Mensch hat nach hunderttausenden von Jahren Evolution in Kleingruppen als Jäger und Sammler doch alles drauf, was er braucht.“

    Laut Teilhard de Chardin strebt die Evolution in eine höhere Richtung. Zu diesem Zweck führte er das Konzept des Punktes Omega ein, dessen leuchtendes Zentrum alles zu sich heraufzieht. „Teilhard de Chardin sieht Leben und Kosmos in einer von Gott bewirkten kreativen Bewegung, die noch nicht an ihr Ziel gelangt ist. Kennzeichen dieser Bewegung ist die ständige Zunahme von Organisiertheit und organischer Einheit. Das Streben in diese Richtung, also der Motor der Evolution, ist für Teilhard die Liebe. Diese Liebe, die das letzte Ziel, „die organische Einheit alles Seienden, bereits handelnd und leidend vorwegnimmt“, war für Teilhard im Herzen eines Menschen vollkommen verwirklicht: in Jesus Christus. So nennt er Christus mit einem biblischen Hoheitstitel (Offb. 21,6) das Omega oder den Punkt Omega, das heißt Ziel, Richtung und Motor der Evolution.“
    Quelle: https://de.wikipedia.org/wiki/Omegapunkt

    Dazu sei anzumerken, dass sich Teilhard de Chardin mit seinem Konzept des „kosmischen Christus“ wohl von Rudolf Steiner inspirieren ließ, was zum Teil auch die Probleme erklärt mit denen sich Teilhard seitens der Kirche konfrontiert sah. Eine Akzeptanz für diese Sichtweise findet sich deshalb eher im Bereich der Esoterik.

  171. @Mona

    „Laut Teilhard de Chardin strebt die Evolution in eine höhere Richtung….Diese Liebe, die das letzte Ziel, „die organische Einheit alles Seienden, bereits handelnd und leidend vorwegnimmt“, war für Teilhard im Herzen eines Menschen vollkommen verwirklicht: in Jesus Christus.“

    Ich würde das jetzt hier aber so verstehen, dass es in der weiteren Entwicklung des Menschen nicht mehr um die biologische Evolution für ein weiter verbessertes menschliches Genom geht, sondern um kulturelle Fortschritte.

    Wenn ich mir Jesus Christus als tatsächlichen Sohn Gottes vorstelle, dessen halbes Genom direkt von Gott maßgeschneidert war, bekomme ich ein gewisses Grausen. Wenn dann andersherum der normale damalige Erdenmensch nie die Chance hatte, sich geistig vernünftig zu entwickeln, und wir noch Jahrmillionen der biologischen Evolution brauchen, bis wir ein Omega-Mensch werden können, dann hätten wir ja noch sehr sehr harte Zeiten vor uns. So wie man weiß, braucht biologische Evolution Mutation und vor allem Selektion, also die Vernichtung von minderwertigen Individuen wie Sozialhilfeempfänger wie auch die Vernichtung ganzer minderwertiger Völker wie die Afrikaner.

    Eine kulturelle Evolution läuft dagegen hauptsächlich über den Faktor Qualität und Überzeugung, und in einem Klima der Freiheit, der Offenheit und dem Austausch von Gedanken. Der Austausch von Personen mit vermeintlich minderwertiger DNA ist dann im wesentlichen unnötig. Der Jesus Christus braucht in diesem Zusammenhang auch keine Extrasupergene, sondern nur richtig viel göttliche Inspiration und Unterstützung sowie einen praktischen Verstand, da was Zielführendes draus zu machen.

    Hier geht es wohl vor allem um die Forstsetzung der kulturellen Evolution. Was hier an wissenschaftlichem und kulturellem Fortschritt sowie mit entsprechender Bildung noch ansteht, ist eine ganze Menge. Und auch die Bildung der Herzen der Menschen kann noch Fortschritte machen, wobei ich echt denke, dass hier auch nur kulturelle Evolution das überhaupt kann. Verbesserte Biologie kann ich mir hier nicht als das wesentliche Element vorstellen, das kann doch nur ein Randthema sein.

    Diese „organische Einheit alles Seienden“ möge denn auch kulturell erzielbar sein, insbesondere in einem Kosmos, der sowieso schon die ganze Zeit von Geist durchsetzt ist, mit Lebewesen, deren lokales Bewusstsein sowieso eine Synthese der lokalen Gehirns mit kosmischem Geist ausmacht. Das muss man ja nur noch kultivieren, dann geht doch alles. Und Individuen können und müssen wir dabei bleiben, nur das Miteinander muss vernünftig laufen.

    In begrenzten Gruppen können wir ja schon ganz gut. Eigentlich müssen wir in erster Linie die gesamte Menschheit als eine – unsere – Gruppe begreifen, und uns unserer gemeinsamen Verantwortung für den ganzen Planeten bewusst werden. Wenn wir das geschafft haben, ist hier wohl zunächst einmal die Kuh vom Eis, und Entspannung angesagt.

  172. @Tobias Jeckenburger

    „So wie man weiß, braucht biologische Evolution Mutation und vor allem Selektion, also die Vernichtung von minderwertigen Individuen wie Sozialhilfeempfänger wie auch die Vernichtung ganzer minderwertiger Völker wie die Afrikaner.“

    Wie kommen Sie jetzt auf so etwas? Falls Sie sich nicht als Rassist oder ähnliches outen wollen, dann müssen Sie zumindest etwas gründlich missverstanden haben.
    Und ja, Evolution ist ein sehr langer Prozess und zieht sich über viele Generationen hin. Wenn Teilhard de Chardin von Liebe als Motor der Evolution spricht, dann meint er sicher nicht die Vernichtung von „minderwertigen Individuen“. Liebe meint jene zu akzeptieren und ihnen zu helfen, die das aus eigener Kraft nicht können. Dazu könnte eine „kulturelle Evolution“ durchaus hilfreich sein, die dem Menschen hilft sich geistig weiterzuentwickeln.
    Im Gegensatz zu Teilhard de Chardin ging Darwin nicht von einer zielgerichteten Evolution aus. Seine Theorie besteht aus mehreren Annahmen: Reproduktion, Variation, Selektion und Vererbung.

  173. @ Tobias Jeckenburger und alle am Thema Bewusstsein Interessierten

    Mir scheint, Sie haben eine sehr „weite“ Sicht von „Bewusstsein“. Sie sprechen auch von „Weltbewusstsein“, kosmischen Bewusstsein“…. Was insofern interessant ist, weil man Bewusstsein üblicher Weise eher nur den Menschen, aber eher weniger den Tieren zuspricht.

    Ich meine das Problem ist, dass man nicht so wirklich weiß, wie es zu „Bewusstsein“ kommt, was ist wesentlich für Bewusstsein?

    Ich möchte versuchen, dieses Problem hier aus Sicht der Elektronik/Informatik anzugehen.

    Ich meine, Voraussetzung für „Bewusstsein“ sind 2 wesentlichen Komponenten.

    1. Auf „flächigen“ Strukturen bekommen die „einlangenden Information“ einen örtlichen und zeitlichen Zusammenhang, danach werden die „Informationen“ (z.B. visuelle Infos auf elektrischen Signalen „abgebildet“) weiter geführt und ausgewertet, als Beispiel die Augennetzhaut.

    2. Einer „chemischen Schnittstelle“, an der „Qualia“ auftreten können und die mit den flächigen Strukturen verknüpft sind. Vielfältige „Qualia“ („Phänomene“ wie Farben, aber auch Lust oder Schmerz….) entstehen meiner Meinung nach, wenn auf die „Elektronenbahnen“ zwischen den Atom- bzw. Molekülverbänden Einfluss genommen, sie sozusagen „moduliert“ werden, demnach eine bestimmte „übertragbare Dynamik“ entsteht.

    In meinem Beitrag „Elektroniker vom 08.06.2020, 18:33 Uhr“ bin ich näher darauf eingegangen.

    Diese wesentlichen 2 (Grund)Komponenten können mehrfach und auf vielfältige Weise strukturell angeordnet und verknüpft sein und zusammenwirken. Für einen einzelnen Menschen scheint eigentlich naheliegend wie es zu Bewusstsein kommt, auch wenn es noch nicht vollständig verstanden wird.

    Aber letztlich ist dem „Bewusstsein ähnliches“, wegen der systematischen Vernetzung zwischen allen lebenden, allenfalls allen „Qualia“ auswertenden Systemen möglich. Sowohl mittels elektrischer als auch chemischer Prozessketten, die mit stark unterschiedlicher „Intensität“ auftreten.

    Die „Abbildung von Information“ erfolgt als Kombination von z.B. Bildpunkten, Schaltprozessen einzelner Neuronen …. die auf flächigen Strukturen (z.B. Netzhaut) jeweils annähernd gleichzeitig auftreten und so einen örtlichen und zeitlichen Zusammenhang bekommen. (Bildpunkte „emergieren“ zu Bildern). Hauptsächlich erfolgen viele zum „Bewusstsein“ (im „Hauptsinn“) gehörenden Auswerteprozesse elektrisch und auf neuronalen Strukturen.

    Eine nur „elektrische Auswertung“ dürfte aber nicht zwingend sein.

    Es ist im Prinzip davon abgängig, wie die „Abgrenzungen“ der verwendeten „Begriffe“, verlaufen, wie diese Begriffe „deklariert“ sind. Dass es hier Probleme gibt ist klar, man versteht zu wenig was „Bewusstsein“ wirklich ist, daher sind die Begriffe sehr vage.

    Aber was wäre, wenn man auch chemische Auswerteprozesse (nicht nur neuronale) dazunimmt?

    An den Darmschleimhäuten als „Trennschicht“ erfolgen wichtige Stoffwechsel Prozesse, die Informationsverarbeitung ist eher sekundär, an der Netzhaut erfolgen auch chemische Prozesse, „wichtig für das System“ ist aber die Informationsverarbeitung und die Entstehung elektrischer die Information abbildenden Signale.

    Das Geschehen an „flachen Schichten“ ist höchst vielfältig und von größter Bedeutung. Bei der Haut, bei Pflanzen, bei p-n Übergängen bei Halbleitern, bei Osmose und Diffusion, ….

    Würde man dies konsequent weiterdenken, hätte letztlich eine rot angestrichene Bank so etwas wie „Bewusstsein“. Allerdings praktisch kein neuronales „Auswertesystem“.

    Auf der flächigen Struktur entstehen zwar auch die Qualia z.B. „rot“. Qualia entstehen praktisch immer wenn auf die „Elektronenbahnen“ zwischen den Atom- bzw. Molekülverbänden modulierend Einfluss genommen wird, behaupte ich einmal. Abhängig von den chemischen Prozessen, wird das darunter liegende Holz mehr oder weniger schnell „vermodern“. Bedeutsam wäre hier die Chemie, kaum die Informationsverarbeitung.

    Dies würde bedeuten, „Qualia“ entstehen oft, weil sie bei chemischen Prozessen als Phänomen auftreten. „Bewusstsein“ hauptsächlich dann, wenn sie auf neuronale („modulierte“ elektrische Signale „verarbeitende“) Prozesse, z.B. im Gehirn einwirken.

  174. @Mona

    Ich meine ja gerade, dass es eben keiner wesentlichen Weiterentwicklung der Biologie des Menschen bedarf. Wir haben von der Biologie her schon alles drauf, was wichtig ist.

    Die Konsequenz daraus, dass man eine wesentliche genetische Weiterentwicklung des Menschen für nötig hält, wäre in der Tat nicht unbedingt, dass man minderwertige Individuen vernichtet. Da habe ich mich wohl übertrieben ausgedrückt.

    Aber die höherwertigen Individuen müssten dann schon mehr Fortpflanzungserfolg haben als die weniger „Geistesgeeigneten“, wie soll sonst eine Verbesserung der genetischen Grundlage funktionieren? Soweit zumindest die gängige Vorstellung über die biologische Evolution.

    Wie dem auch sei, ich setze hier alles auf eine kulturell vermittelte Weiterentwicklung von Mensch und Gesellschaft. Das kann auf jeden Fall funktionieren, und vor allem viel schneller. Erstmal auf eine genetische Weiterentwicklung des Menschen zu warten, da würde ich schlicht zu sagen, dass wir dafür keine Zeit mehr haben.

  175. @Tobias Jeckenburger

    „Aber die höherwertigen Individuen müssten dann schon mehr Fortpflanzungserfolg haben als die weniger „Geistesgeeigneten“, wie soll sonst eine Verbesserung der genetischen Grundlage funktionieren? Soweit zumindest die gängige Vorstellung über die biologische Evolution.“

    Das stimmt so nicht! Wie ich oben bereits schrieb ging Charles Darwin, im Gegensatz zum gläubigen Katholiken Teilhard de Chardin, nicht von einer zielgerichteten Evolution aus.
    Darwins Theorie ist heutzutage allgemein anerkannt und dient als Grundlage für die Synthetische Evolutionstheorie, diese vereint seine Erkenntnisse mit neuen Forschungserkenntnissen aus Genetik, Ökologie Zoologie usw..

  176. @ Mona 14.06.2020, 16:23 Uhr

    Zitat: “Das stimmt so nicht! Wie ich oben bereits schrieb ging Charles Darwin, im Gegensatz zum gläubigen Katholiken Teilhard de Chardin, nicht von einer zielgerichteten Evolution aus.“

    Ob die Evolution zielgerichtet ist oder nicht, hängt vom grundsätzlichen Algorithmus des „Zufallsgenerator System“ ab, das die „Genvariablen“ des gesamten Genpools „generiert“.

    Man könnte z.b. davon ausgehen, dass der gesamte Gencode (bei entsprechender Kodierung) als „sehr große Zahl“ interpretiert werden könnte, die letztlich immer mehr und systematisch in „Richtung unendlich“ anwachsen könnte. In diesem Fall würden eher neue (und „erfolgreiche“) Variable eingebunden, das System wäre eher positiv „zielgerichtet“.

    Es könnten aber auch immer mehr „alte“ (früher schon existierende) Variable, nochmals generiert werden und keine neuen Variablen. Dann könnte die Entwicklung auf schlechtem Niveau „stagnieren“. Dies wäre eher kein besonders „intelligenter Algorithmus“, fast schon nur ein dummer Zufall….

  177. @Elektroniker, Mona, Thomas Jeckenburger:
    Evolution bedeutet Reproduktion mit Verbesserungspotenzial für die nächste Generation im Sinne einer besseren Adaption an Umweltbedingungen. Die Umweltbedingungen umfassen neben den physikalischen/biologischen Faktoren (z.b. saisonale Temperaturschwankungen oder Fressfeinde) auch die selbstgeschaffene Umwelt einer Art. Für den einzelnen Menschen etwa ist es vorteilhaft, sprechen zu können, denn ohne zu sprechen kann er nur eingeschränkt mit anderen interagieren.

    Evolution hat aber für die Zukunft des Homo sapiens keine Bedeutung mehr, denn die biologische Evolution ist viel langsamer als die kulturell/technisch/wissenschaftliche Entwicklung, die der Mensch selber in Gang gesetzt hat. Wenn sich der Mensch oder bestimmte Menschen in Zukunft biologisch ändern, dann weil sie selbst Hand an sich oder ihre Nachfahren gelegt haben und sie ihr Erbgut gemäss Wünschen „editiert“ haben.

    Der Mensch könnte aber auch eine Welt von autonomen technischen Geschöpfen schaffen, die schliesslich ihre Weiterentwicklung selbst in die Hand nehmen.
    Neben der Biosphäre könnte es also irgendwann eine vom Menschen unabhängige Technosphöre geben – und Geschöpfe der Technosphöre könnten das gesamte Universum „erobern“ und das weit besser als der Mensch, weil für sie der Weltraum eine „nette“, angenehme Umgebung sein könnte, etwas was für den Menschen nie zutreffen wird.

  178. @Martin Holzherr

    „Evolution hat aber für die Zukunft des Homo sapiens keine Bedeutung mehr, denn die biologische Evolution ist viel langsamer als die kulturell/technisch/wissenschaftliche Entwicklung, die der Mensch selber in Gang gesetzt hat. Wenn sich der Mensch oder bestimmte Menschen in Zukunft biologisch ändern, dann weil sie selbst Hand an sich oder ihre Nachfahren gelegt haben und sie ihr Erbgut gemäss Wünschen „editiert“ haben.“

    So läuft das nicht! Der Mensch ist ein biologisches Wesen, das von seiner Umwelt abhängig ist. Nicht umsonst sind wir bestrebt den menschengemachten Klimawandel aufzuhalten, weil wir sonst mit gravierenden Umweltproblemen zu kämpfen haben und die weltweite Nahrungsmittelversorgung in Gefahr ist. Ob es da hilft, das Erbgut der Menschen zu „editieren“ ziehe ich in Zweifel. Und was Sie sonst noch schreiben ist pure Science-Fiction.

  179. Ist die Evolution zielgerichtet ?
    Zumindest wissen wir, dass es einen Anfang gab , denBig Bang, und dass es ein Ende geben wird, das Aufblähen der Sonne zu einem roten Riesen , der dann die Erde verbrennt.
    Und wenn wir die Begriffe “Anfang und Ende” auf die Spezies Mensch anwenden, dann sind wir bei der Entwicklung der Fauna und Flora und dabei stellen wir das gleiche fest. Es hatte einen Anfang und es wird ein Ende geben.
    Was jetzt dazwischen liegt, das gilt es sinnvoll zu gestalten, wir haben ja Geist und Urteilskraft. Woher kommt der Geist und woher kommt die Urteilskraft ?
    Damit sind wir beim Kern des Themas, kann man Mensch nur als Naturwesen betrachten oder ist da mehr ?
    Phantasie und Kunst kann man nicht naturwissenschaftlich erklären.

  180. @ Mona 16.06.2020, 09:27 Uhr

    Zitat: „So läuft das nicht! Der Mensch ist ein biologisches Wesen, das von seiner Umwelt abhängig ist. Nicht umsonst sind wir bestrebt den menschengemachten Klimawandel aufzuhalten, weil wir sonst mit gravierenden Umweltproblemen zu kämpfen haben und die weltweite Nahrungsmittelversorgung in Gefahr ist. Ob es da hilft, das Erbgut der Menschen zu „editieren“ ziehe ich in Zweifel. Und was Sie sonst noch schreiben ist pure Science-Fiction.“

    Würde es ungebremst immer mehr Menschen auf der Welt geben, so würde dies irgendwann zwingend auf Umwelt und Klima negativen Einfluss nehmen. Ich persönlich hätte aber nicht geglaubt dass dies schon in näherer Zukunft sein könnte.

    Nicht nur das Erbgut der Menschen wird zunehmend „editiert“ werden, sondern die gesamte Biosphäre könnte genetisch so „verbessert“ werden, dass die Systeme möglichst Umwelt- und Klima resistent werden. Z.B. warten Forstwissenschaftler dringend auf Forschungsaufträge um „klimafeste Wälder“ zu züchten. Fast die gesamte Wissenschaft will auf dem „Klimazug“ aufspringen, um Beiträge zur Lösung der auftretenden Probleme liefern zu dürfen und das finde ich gut.

    Selbstverständlich wird sich auch die Technik weiter entwickeln. Allerdings sehe ich dort eher keinen automatischen „Antriebsmotor“ wie es das „Lustprinzip“ (als „Empfindungsphänomen“) in der Biologie ist. Ob man ähnlich starke „Antriebskonzepte“ als Motivator für technische Systeme findet ist fraglich, könnte mir aber so etwas ähnliches wie „künstliche Geld gierige Systeme“ vorstellen.

    Andererseits wird man als Mensch das aber auch verhindern wollen, weil wir kaum davon abhängig werden wollen, dass autonome technische Systeme uns Menschen, sozusagen aus ethischen Gründen, freiwillig eine „Nische zum Überleben“ überlassen werden.

  181. @Mona/Martin Holzherr Biologische Evolution

    Der Mensch als Kulturwesen muss viel mehr auf seine Umwelt achten, weil er in modernen Zeiten einen so großen Einfluss auf seine Umwelt hat. Aber wir können angesichts des Klimawandels nicht darauf warten, dass wir uns in einer zukünftigen biologischen Evolution zu liebevolleren Geschöpfen entwickelt haben. Wir haben keine 50 Jahre mehr Zeit hier gegen den Klimawandel effektiv vorzugehen. Das geht nur durch kulturelle Evolution bzw. durch Kultur und Politik.

    Die Idee vom Omegamenschen, der wie Jesus Christus ganz von Liebe bestimmt ist, erscheint mir jetzt aber auch als Science-Fiction. Mag sein, dass hier auch die Biologische Evolution über die nächsten 500.000 Jahre dann doch den Menschen nochmal entscheidend umbaut. Warum nicht.

    Aber das hilft uns im 21. Jahrhundert kein Stückchen weiter. Wir können einen Naturgarten aus der Erde machen, und wirklich friedlich und gut organisiert miteinander umgehen, insbesondere was Geld und Arbeit angeht. Dafür läuft uns hier jetzt schon die Zeit davon: Klimakrise, Ressourcenverbrauch und auch weiterhin die Bedrohung durch die Existenz von Atomwaffen sind akut zu entschärfen. Und jetzt auch noch die Coronagegenmaßnahmen, die eine Wirtschaftskrise nach sich ziehen wird, die die Welt wohl noch nicht gesehen hat. Die Herausforderungen sind groß, aber im Prinzip kann der Mensch doch alles, was er braucht, um hier erfolgreich zu sein.

    Die Geschwindigkeit, in der sich der Mensch kulturell bedingt eine andere Haltung zulegen kann, und anfängt die anstehenden Probleme zu erfassen, und am Ende auch zu lösen, überrascht immer wieder. In 20 oder 40 Jahren kann sich eine moderne Gesellschaft grundlegend wandeln. Die Geschwindigkeit der biologisch-genetischen Evolution ist 1000 mal langsamer, mindestens.

    Was die Editierung von Erbgut angeht, so frage ich mich, ob das denn hier eine Rolle spielt. Erbkrankheiten in der Keimbahn auszuschalten und ein paar spezielle Wünsche an das zukünftige Kind sind denkbar. Aber einen neuen Menschen macht das noch nicht.

    Was die Technosphäre angeht, so reicht es ja nicht, dass sich diese rein technisch bilden könnte. Sollte es nicht gelingen, dass diese KI-Systeme eine eigene Innenwelt mit einer Lebensqualität in sich selbst entwickeln, wird diese auch keine eigene Motivation entwickeln können. Sie muss sich dann wohl weiterhin auf die Unterstützung des Menschen beschränken. Da macht es dann keinen Sinn, nur mit Geräten die Galaxis zu besiedeln, das wäre ja nur ein seelenloser Schrottplatz. Da muss der Mensch und Vertreter seiner ganzen Biosphäre schon mit umziehen, wenn die Besiedlung von geeigneten Exoplaneten Sinn machen soll.

  182. @ Elektroniker: Die Natur kennt keine Algorithmen, jedenfalls keine, die wir kennen würden. Diese bräuchten konkrete Handlungsanweisungen. Genetische Algorithmen kennen wir aus Technik und Informatik und dienen dort unter Anwendung von Mutation und Selektion der Lösung von Problemen. Sie beschreiben aber nicht biologische Evolution. Dazu sind sie nicht in der Lage, da epigenetische Prozesse nicht berücksichtigt werden könnten.

  183. @ Tobias Jeckenburger: “Was die Technosphäre angeht, so reicht es ja nicht, dass sich diese rein technisch bilden könnte. Sollte es nicht gelingen, dass diese KI-Systeme eine eigene Innenwelt mit einer Lebensqualität in sich selbst entwickeln, wird diese auch keine eigene Motivation entwickeln können. Sie muss sich dann wohl weiterhin auf die Unterstützung des Menschen beschränken.”
    Ich bin sicher, eines Tages wird man das menschliche Hirn nachbauen können, über diverse Rückkoppelungsschleifen wird man Emotionen, Gefühle und damit auch Motivation induzieren können, aus mechanischen Robotern werden dann Maschinen, die uns zum Verwechseln ähnlich sind. Vielleicht sind sie dann “die Krone der Schöpfung” 🙂

  184. Warum bringen sie das Beispiel mit dem Storch?

    Wenn ich einem Teil der Bevölkerung ein gesundheitsschädliches Gift gebe, und dem andere nicht, so wird sich das statistisch kausal anhand der gezählten Todesfälle und der ärztlichen Diagnosen nachweisen lassen, oder nicht?

    Wozu ist die Wissenschaft der Statistik sonst gut?

    Wenn die Austria Statistik sauber arbeitet, wovon ich ausgehe, weiß ich woran ich bin. Wenn die Statistiken nach drei Monaten abgebrochen werden, sehe ich leider nur den abgeschnittenen Teil des Bildes.

  185. @Stephan Schleim –
    (3) Kausale Irrelevanz: Kausale Erklärungen sind überbewertet und für den Fortschritt vieler Wissenschaftszweige von geringerer Relevanz.

    Das möchte ich ergänzen.
    Weil – nicht zu wissen, warum jemand was tut, mag ja noch angehen, aber diese Ursache für nicht Relevant zu erklären geht ein wenig zu weit.

    “Es ist eine Eigenschaft unserer Alltagssprache, mit einem Ausdruck wie “die Entscheidung, X zu tun” einen Sachverhalt ausdrücken und in einen Sinnzusammenhang stellen zu können, der tatsächlich stattgefunden und zudem eine erklärende Funktion hat.”
    Es ging aber auch nicht nur um die Kausalität sondern die Tücken der Erklärung, nicht wahr – und das im Zusammenhang mit “Entscheidung”?

    Wenn ich Bewusstsein auf “Emergenz”, also reine Struktureigenschaften reduziere, zeigt sich, dass es immer wieder um “Differenzierung” geht. Erst wenn etwas “völlig ausdifferenziert” ist, emergiert da etwas “übergeordnetes”.
    Dieser Zellhaufen, der unser Hirn darstellt, ist nicht nur differenziert, er tut es auch: das Differenzieren.

    Ich bin auf Umwegen auf diesen (bzw. folgenden) Gedanken gestoßen, es ging um: Diskriminierung, das Gegenteil, bzw. die automatische Folge. (Genauer das Toleranz-Paradoxon.)
    Eine – jede! – Unterscheidung bedingt, dass dadurch, das etwas abgegrenzt wird, immer auch etwas ausgegrenzt wird. Ob sich das zu einer “Diskriminierung” im gesellschaftlichen Sinne auswirkt ist zweitrangig, ich möchte erstmal auf die schnöde Tatsache hinaus.
    Punkt ist, dass das halt immer geschieht.
    Jede Handlung hat Konsequenzen – aber Gedanken haben erst dann “richtige” Konsequenzen, wenn sie an den Punkt gelangen, wo eine Entscheidung getroffen wird – und “Sprache” findet dazwischen statt. Es ist Handlung und Gedanke gleichzeitig.
    (Zum Beispiel die Wörter für Zahlen – eine drei schließt alle anderen Zahlen aus. Für Mathematik ist das sehr praktisch, für andere Informationsübertragungen unpraktisch.)

    Worauf ich hinaus will, ist, dass eine “kausale Erklärung” zwei große Vorteile hat: sie stimmt (wenn sie stimmt) und sie schließt nicht von sich aus den Rest der Realität aus, dazu ist eine weitere Entscheidung nötig – nie daran zu Zweifeln.

    Wissen ist keine Entscheidung, das wird geschafft, nicht erschaffen.
    Aber im Glauben wirken ganz andere Kräfte…

    Solange ich eine Umgebung für die Kausalität habe, die sich ändern kann oder mir nicht kenntlich sein muss, findet auch kein “Ausschluss” statt, ist es nur eine Abgrenzung. (Definition, Differenzierung.) Die der Orientierung dienlich sein kann – aber nicht muss.

  186. Erst ging die Vorschau nicht und dann bin ich zu früh auf den falschen Knopf gekommen – ich bitte auch die Rechtschreibfehler zu entschuldigen, die drin geblieben sind.
    ——————
    Das mit der “Umgebung der Kausalität” ist zu dünn, ich möchte ergänzen –
    Auch wenn wir die Leib-Seele Dualität los geworden sind, bleiben da Glaube, bzw. Lüge, die uns kümmern. Und da finde ich es wesentlich, diese Unterscheidung beim (bzw. nach dem) Differenzieren zu machen.

    Kausalitäten trauen zu können, bzw. nicht pauschal, bzw. ungerechtfertigt abzuwerten ist wichtig, um einer Lüge auf die Spur zu kommen. Orientierung schützt.

    Und für den Fortschritt relevant ist, wie klar ein Punkt abgegrenzt ist, nicht, wie vehement er verteidigt wird.

    Last, but not least fällt die Aussage “Weiß ich nicht” ja auch unter “kausale Begründung”. Und diese frei abgeben zu können ist sehr relevant und bringt durchaus weiter.

    Zum “Diskriminieren” noch – bei einer Definition ist es in Ordnung, dass sie etwas ausschließt, aber bei den alltäglichen Differenzierungen, dem normalem Unterscheiden, kommen wir halt ins Trudeln. Wir diskriminieren, was uns fremd ist. Glauben daran und legen uns damit fest. Aber es ist eigentlich auch “was neurologisches”, i.S.v. der Punkt, wo wir unsere Welt gestalten, durch die Entscheidungen, die wir mehr oder weniger bewusst treffen, die uns dann wiederum formen.

    Einerseits bringt uns dieses Unterscheiden können weiter, anderseits legen wir uns mit dem, was wir Ausschließen, fest. (Die Emergenz differenziert sich…)

    Es spricht aber nichts dagegen, sich zu schützen, also abzugrenzen, wo es nötig ist – und anderes immer feiner zu differenzieren.
    Sobald wir schnallen, dass das unser eigener Job ist und nicht der externer Autoritäten.
    ———————-
    Damit meine erste Fassung nicht so durcheinander rüber kommt, möchte ich “den damaligen Gedankengang” kurz skizzieren: es ging um das Toleranz-Paradoxon und Diskriminierung – und erst von da zum Differenzieren.
    Denn über meine entwicklungsgeschichtliche Affinität (also parallel) kam ich zur Emergenz, dem sich differenzierenden Bewusstsein – und den abgeschlossenen Stufen, einer “natürlichen Diskriminierung”.
    So konnte ich meine Position zur “Toleranz” klären – man sollte Diskriminierung (falsche Werte) diskriminieren können, wenn man sauber differenzieren, bzw. werten oder sich orientieren will.

    Das hat jetzt bei der Übertragung etwas geholpert, Entschuldigung.
    Aber so viel zu “kausal”.

Schreibe einen Kommentar